1. Trang chủ
  2. » Lịch sử lớp 12

Tạp chí Epsilon số 6

177 26 0

Đang tải... (xem toàn văn)

Tài liệu hạn chế xem trước, để xem đầy đủ mời bạn chọn Tải xuống

THÔNG TIN TÀI LIỆU

Nội dung

Trong khi đó người Hy Lạp (Archimede nói riêng) đã dùng cả các phép chứng minh dựa trên những tương tự một cách máy móc, song những phép chứng minh ấy bị coi là không chặt chẽ, chỉ có tí[r]

(1)(2)

Chủ biên:

TRẦN NAM DŨNG

Biên tập viên:

VÕ QUỐC BÁ CẨN

TRẦN QUANG HÙNG

NGUYỄN VĂN HUYỆN

(3)

Ban Biên tập Epsilon

Cuối số Tạp chí Epsilon đến tay bạn Số năm

Bắt đầu từ ngày 13/2/2015, đến hẹn lại lên, vào ngày 13 tháng chẵn, Epsilon-tạp chí online người yêu toán lại mắt hàng trăm, hàng ngàn độc giả

Và để có đặn với nội dung ngày phong phú hình thức ngày đẹp nỗ lực tập thể: từ người viết đến biên tập viên Tất làm việc tinh thần tự nguyện mong muốn đóng góp cho cộng đồng

Đặc biệt, dù hoàn toàn dựa tinh thần tự ngun, khơng có quyền lợi vật chất, ràng buộc pháp lý tất người làm việc với tinh thần trách nhiệm cao, có đêm phải thức trắng để hoàn tất viết hay hoàn chỉnh phần biên tập

Qua số báo, Epsilon dần có thêm nhiều tác giả hơn, nhiều cộng tác viên nhiều độc giả Đội ngũ biên tập bổ sung số lượng nâng cao chất lượng, vừa đảm bảo công tác biên tập tiến độ, vừa chủ động tạo nguồn dồi cho số báo

Chúng ta qua năm đầy khó khăn thật tự hào Có năm đầu tiên, có nghĩa có năm thứ 2, thứ

Và để Epsilon tiếp nối, nguồn lượng lớn ủng hộ độc giả Những góp ý, bình luận, đặt hàng từ phía độc giả động lực cho Epsilon tiếp tục ấn hành Đặc biệt, ban biên tập ln chờ đón viết từ phía độc giả Chúng tin viết bạn chắn làm tăng thêm phong phú tạp chí, nội dung đề tài lẫn phong cách hành văn

Đi nhiều người ta xa

(4)

Ban Biên tập Epsilon

Lời ngỏ cho Epsilon số

Ngơ Quang Hưng

Tính lý hàm độc tài

Lý Ngọc Tuệ

Xấp xỉ Diophantine với độ đo - Định lý Khintchine 21

Đàm Thanh Sơn

Hình học rối lượng tử 33

I I Blekman, A D Myshkis, Ya G Panovko

Logic toán học ứng dụng 39

Desmond MacHale

Cuộc đời nghiệp George Boole - Sự khởi đầu cho kỷ nguyên kỹ thuật số 55

Bình Nguyễn

Nhận dạng chó mèo 59

Đặng Nguyễn Đức Tiến

Bài toán cân tiền 67

Nguyễn Tiến Dũng

Xung quanh tốn hình học kỳ thi VMO 2014 77

Nguyễn Ngọc Giang

Mối liên hệ Euclide, Afin Xạ ảnh qua toán sách "Các phương pháp giải toán kỳ thi Olympic" 89

Trần Quang Hùng

(5)

Trần Minh Hiền

Thuật tốn tham lam xây dựng cấu hình tổ hợp 109

Lưu Bá Thắng

Định đề Bertrand 125

Kiều Đình Minh

Chuỗi điều hòa 133

Yimin Ge

Số dư củaA:axCBx 149

Trần Nam Dũng

Bài toán hay lời giải đẹp 153

Trần Nam Dũng

Các vấn đề cổ điển đại 157

Ban Biên tập Epsilon

(6)(7)

Ngô Quang Hưng (Đại học Buffalo, Mỹ)

Bài viết chứng minh định lý kinh điển Kinh Tế học, gọi định lý bất khả thi Arrow Định lý có nhiều chứng minh ngắn gọn, khoảng nửa trang Nhưng chọn đường tương đối dài để đến kết luận Đích đến, người ta thường nói, không thú vị đường

1 Định lý bất khả thi Arrow

Marquis de Condorcetlà triết gia, nhà toán học, nhà khoa học trị người Pháp sống kỷ 18 Năm 1785, ông viết “Essay on the Application of Analysis to the Probability of Majority Decisions” có ảnh hưởng sâu rộng đếnlý thuyết chọn lựa xã hội, kinh tế học, đến thuật toán xếp hạng quảng cáo mạng Condorcet người mang (tính chặt chẽ của) Tốn học vào nghiên cứu khoa học xã hội Ông tham gia cách mạng Pháp, viết vài sách bất hủ ủng hộ cho tinh thần Khai Sáng Ông bị bắt giam gần năm, tù Nhiều khả tự uống thuốc độc

Ông khám phá ra“nghịch lý Condorcet” Đại để nghịch lý sau Giả sử nhà nước cần đầu tư vào ngành giao thông (GT), y tế (YT), giáo dục (GD) Nhà nước làm trưng cầu dân ý Mỗi người dân bỏ phiếu xếp hạng riêng tầm quan trọng ba ngành Ví dụ, anh A bảo tơi nghĩ GD trước, đến GT, đến YT Anh B chọn YT>GD>GT, vân vân Thì khả sau xảy ra: đa số người xếp GD YT, đa số xếp YT GT,đa số xếp GT GD Đó tính phi lý chọn lựa xã hội Khi biết nghịch lý Condorcet rồi, đọc thống kê xã hội cẩn thận Obama với McCain cãi nhau, lôi thống kê Một ông bảo phải đầu tư đa số dân chúng ủng hộ kia, McCain bảo Chúng ta nên nghĩ đến khả vô lý chọn lựa xã hội Có khả Obama lẫn McCain đúng, vô lý

Đến năm 1950,Kenneth Arrow(giải Nobel kinh tế 1972) viết báo tiếng luật bầu cử [1], ơng chứng minh định lý ta gọi làđịnh lý bất khả thi Arrow1. Định lý Arrow nói “hàm độc tài” luật bầu cử có tính “duy lý” tuyệt đối Để phát biểu định lý này, ta định nghĩa “độc tài”, “duy lý”

Để đơn giản (nhưng không tính tổng qt) ta giả sử xã hội có chọn lựa A-B-C cần xếp hạng bầu cử (GD-YT-GT, anh Ba-anh Tư-anh Sáu, bánh mì-sữa-bia) Mỗi phiếu bầu gồm ba đề mục Đề mục thứ xếp hạng A B B A Đề mục thứ hai xếp hạng cặp B, C; đề mục thứ ba xếp hạng cặp A C Nếu anh xếp hạng vòng tròn (A B, B C, C A) anh bị chập cheng, khơng cho bầu Nói cách khác, ta giả sử tất phiếu bầu hợp lệ, nghĩa khơng phiếu xếp hạng vịng trịn

1

(8)

Sau có tất phiếu bầu xã hội dựa mộtluật bầu cửđể cóxếp hạng tồn xã hội ba A, B, C, nghĩa định xem xã hội thích chọn lựa A B, B C, C A Luật bầu cử phải thỏa mãn số tiên đề định:

1 Tính độc lập chọn lựa không liên quan2(IIA): việc xã hội xếp hạng A B hay B A độc lập với việc người xếp C cao thấp

2 Tính trí(cịn gọi làhiệu suất Pareto): người thích A B xã hội phải chọn A B

3 Tính lý: xã hội khơng thể xếp hạng quẩn quanh theo vịng trịn (A B, B C, và C A)

4 Không độc tài: xếp hạng xã hội giống hệt xếp hạng anh Tám Tàng mà khơng đếm xỉa đến phần lại xã hội

Arrow chứng minh khơng có luật bầu cử thỏa bốn điều kiện trên, tất phiếu cá nhân hợp lệ (Bài báo Arrow dài dòng văn tự Với giả thuyết, tiên đề, ông lại đá sang triết lý vài kết trước đó.) Định lý Arrow thật định lý mang tính tổ hợp, có cácchứng minh tổ hợp ngắn gọn

Phần mô tả khơng đủ cụ thể mặt Tốn học để ta chứng minh Một kỹ quan trọng mà người làm Tốn ứng dụng cần có khả “Tốn học hoá” đối tượng nghiên cứu ngành ứng dụng Bước “Tốn học hố” vấn đề đơi quan trọng không bước giải vấn đề

Một mơ hình Tốn học vấn đề chọn lựa xã hội sau Giả sử cónphiếu bầu Phiếu bầu thứiđược đại diện ba.xi; yi; zi/2 f 1; 1g3;trong

xi D (

1 phiếui chọnA > B phiếui chọnB > A

yi D (

1 phiếui chọnB > C phiếui chọnC > B

zi D (

1 phiếui chọnC > A phiếui chọnA > C

(Lý ta chọnf 1; 1gthay vìf0; 1g,ftrue;falsegsẽ trở nên rõ ràng đây.) Định nghĩa hàmNAEW f 1; 1g3 ! f0; 1gnhư sau3:NAE.a; b; c/D0nếu nếuaDb Dc Khi đó, phiếu.xi; yi; zi/là phiếu hợp lệ nếuNAE.xi; yi; zi/D1 Vớinphiếu bầu ta có ba vectors xD xi/niD1,yD.yi/niD1,zD.zi/niD1 f 1; 1gn Bây ta mơ hình xem bốn tính chất phát biểu mặt toán học nào:

2Independence of irrelevant alternatives 3NAE

(9)

1 Tính chất IIA nói chọn lựa xã hội đúc kết ba hàm số f; g; h W f 1; 1gn ! f 1; 1g,

f x/ D (

1 xã hội chọnA > B xã hội chọnB > A

g.y/ D (

1 xã hội chọnB > C xã hội chọnC > B

h.z/ D (

1 xã hội chọnC > A xã hội chọnA > C

2 Tính trí nói với x f 1; 1g, f x; : : : ; x/ D x, g.x; : : : ; x/ D x

h.x; : : : ; x/Dx

3 Tính lý nói khơng tồn phiếu hợp lệ.x;y;z/mà lại cho chọn lựa xã hội khơng hợp lệf x/Dg.y/Dh.z/

4 Tính khơng độc tài mơ hình hố sau Vớii 2Œn, gọiDicti hàm độc tài4thứi, trả phiếu bầu củaxi, nghĩa làDicti.x/Dxi Tớnh khụng c ti núi rngf; g; hÔDicti, vi mii Œn

Từ tính trí tính lý suy rằngf g h Ta lập luận sau Xét ba x,yD x, vàzD.f x/; ; f x// Do tính trí ta cóh.z/Df x/ Như chọn lựa xã hội có tính lý thỡg.y/Ôf x/; ngha lg x/D f x/vi mix Tương tự ta có

g x/D h.x/với mọix Do đóf h Lập luận đối xứng dẫn đếnf g h

Tóm lại, chọn lựa tồn xã hội mô tả bằngmộthàm sốf W f 1; 1gn ! f 1; 1g. Và ta cần tìm hàmf cho

Nếu.x;y;z/là phiếu hợp lệ thìNAE.f x/; f y/; f z//D1 (Đây tính lý cha chn la xó hi.)

f ÔDicti vi mọii 2Œn (Hàm chọn lựa xã hội không nên hàm độc tài.)

Định lý Arrow nói khơng có hàmf thoả hai tính chất lúc (Lưu ý tất hàm độc tài hàm lý!) Chúng ta chứng minh định lý Arrow phân tích Fourier hàm nhị phân Chứng minh làphát kiến tuyệt vờicủa Gil Kalai [2] Việc nghiên cứu hàm nhị phân (còn gọi hàm Bool5) đề tài quan trọng lý thuyết máy tính Nó quan trọng cách hiển nhiên máy tính xử lý bít nhị phânf0; 1g Nhưng cụ thể hơn, mơn giải tích hàm nhị phân cóứng dụng cụ thểtrong lý tuyết tính tốn đại Phương pháp giải tích để nghiên cứu hàm nhị phân ta tìm cách viết chúng thành tổ hợp tuyến tính hàm đơn giản Để làm điều ta cần biến đổi Fourier rời rạc (DFT) Để mô tảDFTmột cách tổng quát ta cần lý thuyết biểu diễn nhóm Do ta bắt đầu với lý thuyết biểu diễn

4

Dictator

5

(10)

2 Sơ lược lý tuyết biểu diễn nhóm

Lý thuyết biểu diễn nhómcho phép ta nghiên cứu nhóm (trong đại số trừu tượng) dùng đại số tuyến tính (Đại số tuyến tính vạn tuế!) Bằng cách này, số vấn đề, đặc tính nhóm trừu tượng giải tìm hiểu dùng cơng cụ đại số tuyến tính Từ góc nhìn tổ hợp, “Nhóm đối xứng” Bruce Sagan thú vị [3]

Trước hết ta định nghĩabiểu diễn ma trận6của nhóm Một biểu diễn ma trậnnchiều nhómGlà phép đồng cấu7 WG!GLn.F/trong đóFlà trường đại số, ví dụ trường số phức, cịnGLn.F/là nhóm tuyến tính tổng qt8bậcntrên trườngF Tổng qt hơn,

ta khơng thiết phải biểu diễn nhóm ma trận GọiV khơng gian vector có số chiều hữu hạn GọiGL.V /là nhóm biến đổi tuyến tính trênV (nghĩa làGL.V /là tập hợp ánh xạ tuyến tính khả nghịch) Một phép biểu diễn nhómGtrên khơng gianV phép đồng cấu

WG !GL.V /:

Nói cách khác, biểu diễn luật gán: ta gán cho phần tửgcủa nhómG ánh xạ

.g/2 GL.V /sao cho phép gán tương thích với hoạt động nhómG Nếu có vector sở củaV ta dễ dàng chuyểnthành phép biểu diễn ma trận (Trong trường hợp đó, phần tửg 2Gsẽ có tương ứng ma trận khả nghịch.g/.)

Ví dụ Xét nhómG DZn Ánh xạ WG !GLm.R/gán phần tửk2 f0; : : : ; n 1gmột

ma trận.k/khả nghịchmm, cho, với mọij; k 2Zn, ta có j Ck/D.j /.k/:

(Đây định nghĩa phép đồng cấu.) Do đó,.0/phải ma trận đơn vị Và, với mọikta có

.k/D.1/k Nghĩa là, sau chọn ma trận khả nghịch.1/sao cho.1/nD.0/DIm (nếu được) phần cịn lại củalà hồn tồn xác định.

Trong phần cịn lại này, để đơn giản vấn đề ta xétV khơng gian tuyến tínhm

chiều trường số phức (hiểu làV DCm) Như hầu hết đối tượng trừu tượng khác

tốn học, ta tìm cách chia phép biểu diễn nhóm thành thành phần nhỏ hơn, “tối giản” Từ đó, ta nghiên cứu cấu trúc lớn cấu trúc tối giản, với hy vọng nhiều câu hỏi dễ trả lời

Một không gian conW củaV gọi làG-bất biếnnếu phần tử gủaGtương ứng với ánh xạ từW vàoW Cụ thể hơn, với mọiw2W vàg 2G ta có.g/w2 W thìW gọi làG-bất biến Tất nhiên, nếuW làG-bất biến ánh xạ thu hẹp củatrênW biểu diễn củaG

Hai không gian bất biến tầm thường làV vàfE0g Nếu ngồi hai khơng gian tầm thường ra,V

khơng cịn khơng gian conG-bất biến khác, thìđược gọi mộtbiểu diễn tối giảncủaG NếuV tổng trực tiếp9của hai không gian conG-bất biếnW

1vàW2, ký hiệu làV DW1˚W2, phép biểu diễntrênV tổng trực tiếp của1và2, viết D1˚2, đó1 và2

6

Matrix representation

7

Homomorphism

8

General linear group

9

(11)

là thu hẹp củatrênW1vàW2, theo thứ tự Nếu phép biểu diễn tối giản tổng trực tiếp phép biểu diễn khác, ngoại trừ tổng tầm thườngV ˚ f EOg

Ví dụ Xét nhómGDZ2n, ánh xạ WG !GL2.C/định nghĩa sau:

.0/ D 0 DI2 1/ D

e i=n 0 e i=n

.k/ D 1/k; k 2Z2n:

(Lưu ý rằngi là số phức và.1/2n D.0/như ý.) Trong ví dụ thìV DC2 Xét

W1 D

a

W a2C

W2 D

0 b

W b2 C

:

Dễ thấyW1 là khơng gianG-bất biến, với mọik2 Z2nvà mọiwD

a

2W1, ta có

.k/wD

ek i=n 0

0 ek i=n a

D

aek i=n

2W1:

Tương tự,W2cũng bất biến Ngoài raC2 DW1˚W2 a b D a C b

với vector

a b

2C2 Vì thế,khơng phải biểu diễn tối giản Hai thu hẹp12của định nghĩa sau:

1.0/ D

1 0

2.0/ D

0 0

1.k/D

e i k=n 0

2.k/D

0 0 e i k=n

là biểu biễn tối giản có số chiều bằng1.

Đến ta có đủ kiến thức để phát biểu định lý đơn giản quan trọng lý thuyết biểu diễn: định lý Maschke Định lý tương tự định lý “phân tích thừa số nguyên tố”, định lý đại số đa thức tích đơn thức tuyến tính Heinrich Maschke (1853–1908) nhà Tốn học người Đức Ông theo học người khổng lồ Weierstrass, Kummer Kronecker Tốt nghiệp năm 1880, không tìm vị trí Đức, ơng di cư sang Mỹ, nhận vị trí khoa Tốn mở Đại Học Chicago, nơi nhà Toán học lừng danh Việt Nam làm việc; anh chuyên gia lý thuyết biểu diễn

(12)

Chứng minh. Ta cần chứng minh rằng, nếuchưa tối giản thìD1˚2 và1; có số chiều nhỏ Giả sửW V không gian conG-bất biến không tầm thường Ta chứng minh tồn tạiW? choW ˚W? DV vàW?cũngG-bất biến Gọif;glà dạng Hermit10tuỳ ý không gianV, dễ chứng minh dạng song tuyến sau sau đây

hv;wi D jGj

X

g2G

f.g/v; g/wg

làG-bất biến:hv;wi D h.g/v; g/wi,8g 2G;v;w V Từ đó, không gian trực giao W? củaW định nghĩa theo dạng song tuyến này11 làG-bất biến

Như vậy, ta nghiên cứu phép biểu diễn dùng phép biểu diễn “đơn giản hơn” chút Tuy nhiên, phép biểu diễn đối tượng phức tạp để mơ tả (nó phép đồng cấu thỏa mãn số tính chất đại số, xem ma trận ta chọn trước hệ sở không gianV) Thậm chí, có phép biểu diễn (khơng đẳng cấu12với nhau) ta khơng biết Có thể có vơ hạn phép biểu diễn không? Làm để phân loại chúng?

Để phân loại phép biểu diễn, có cách để loại bỏ đa số thơng tin phép biểu diễn, giữ lại vài số! Các số chứa nhiều thông tin phép biểu diễn, ta dùng chúng để phân loại phép biểu diễn Kết định lý đẹp đại số

Các số “kỳ diệu” chứa hàm gọi làhàm đặc trưng13của phép biểu diễn Hàm đặc trưngcủa phép biểu diễntrên nhómGlà hàmWG!Cđịnh nghĩa sau

.g/Dtrace g//:

(Nhớ rằng.g/là tốn tử tuyến tính khả nghịch khơng gian phứcV, cáivết14trace g//

của.g/là tổng trị đặc trưng Hàm đặc trưngcũng vector mà tọa độ đánh sổ thành viên nhómG

Ví dụ Lại xét nhómG DZnvà biểu diễnWG !GLm.C/ Gọilà hàm đặc trưng của

biểu diễn này, thì

.0/Dtrace 0//Dtrace.Im/Dm:

Và với mọik2Znta có

.k/Dtrace k//Dtrace 1/k/D m X

iD1

ki

trong đó i là trị đặc trưng của.1/ Xét vector đặc trưngv Cmtương ứng vớii, thì

.1/vDiv Do đó,.1/nvDinv Mà.1/n DIm Do đó,i là bậcncủa

1.

10Hermitian form

11W?WD fvj hv;wi D0;8w2Wg

12

Isomorphic

13

Character

14

(13)

Tổng quát hơn, định nghĩa số chiều hàm đặc trưng số chiều không gianV phép biểu diễn Định nghĩa tích vơ hướng Hermit hàm đặc trưng sau:

h; 0i D jGj

X

g2G

.g/0.g/: (2.1)

(alà liên hợp phức củaa.) Hàm đặc trưng phép biểu diễn chứacực kỳ nhiềuthông tin phép biểu diễn Sau vài kết luận quan trọng:

1/chính số chiều phép biểu diễn (Lưu ý rằng1là phần tử đơn vị nhómG NếuG DZnthì1là số ngun0.)

g/D.hgh 1/, với phần tửg; h2G, nghĩa hàm đặc trưng có giá trị lớp liên hợp15 của nhóm.

g 1/D.g/

Hàm đặc trưng của˚0là tổngC0của hàm đặc trưng thành phần; 0. GọiN D jGj, và1; 2; : : : đại diện lớp đẳng cấu phép biểu diễn

tối giản trênG, gọii hàm đặc trưng củai Ta có:

Các vectorsi vng góc với có chiều dài đơn vị Gọiclà tổng số lớp liên hợp nhóm GọiC khơng gian vector hàm f W G ! Csao cho f có giá trị lớp liên hợp củaG Ta có, hàm đặc trưngi tạo thành hệ sở trực chuẩn củaC (Ta dùng tính chất để nói thêm biến đổi Fourier rời rạc nhóm Abel đề mục tới.)

Tổng số lớp đẳng cấu phép biểu diễn tối giản với tổng số lớp liên hợp nhómG Gọir tổng số

Gọidi số chiều củai, ta códi chia hết choN,

N Dd12C Cdr2:

Một hàm đặc trưng nhóm biểu diễn (theo cách nhất) thành tổ hợp tuyến tính hàm đặc trưng tối giản

Hai phép biểu diễn có hàm đặc trưng giống đẳng cấu với

Một hàm đặc trưnglà tối giản có chiều dài đơn vị (nghĩa làh; i D1) NếuGlà nhóm Abel biểu diễn tối giản có số chiều bằng1 Có hệ tuyệt đẹp lý thuyết biểu diễn nhóm khiG nhóm hốn vị củanphần tử Gọiflà số cácstandard Young tableauxdạng Ta có:

X

`n

.f/2 DnŠ

Định lý chứng minh bằnggiải thuật Robinson-Schensted 15

(14)

3 Biến đổi Fourier rời rạc

Tôi học biến đổi Fourier rời rạc (DFT) lần vào khoảng năm 1993 Học xong thấy hoang mang, theo kiểu: lấy vector này, tính tốn này, hệ số kia, khơng hiểu ý tưởng nằm sau cơng thức Sau khám phá làDFTchẳng qua phép thay đổi sở khơng gian tuyến tính Từ thấy thứ rõ ràng, dễ hiểu hẳn

3.1 Biểu diễn nhóm Abel hữu hạn

Các biểu diễn tối giản nhóm Abel biểu diễn với số chiều Nếu nhóm cónphần tử cónhàm đặc trưng trực giao Nhóm tuần tồn nhóm Abel Nhóm tuần hồnZncó đúngnhàm đặc trưng tối giảna; a2 Zn Mỗi hàm đặc trưnga vector trường vector phứcCn, định nghĩa làa.b/ D !nab, đó!nD e2 i=n nguyên thuỷ Các hàm đặc trưng hệ trực chuẩn theo tích Hermit (2.1):

ha; bi D

1 n

X

c2Zn

a.c/b.c/Dab:

(ab D1nuaDbv0nuaÔb.)

nh lý nhóm Abel hữu hạn nói nhóm Abel hữu hạnG viết dạng tổng trực tiếp nhóm tuần hồn:GŠZm1˚ ˚Zmk Các biểu diễn tối giản nhómGlàtăng sờcủa biểu diễn tối giản nhóm tuần hồnZmi Các hàm đặc trưng tối giản nhómGlà tích tăng sờ hàm đặc trưng tối giản nhóm tuần hoàn

Zmi Với phần tửaD.a1; ; ak/2Zm1 ˚ ˚Zmk, ta có hàm đặc trưng tối giản

acủa nhómGđịnh nghĩa sau: với “tọa độ"bD.b1; ; bk/2Zm1˚ ˚Zmk

a.b/D

k Y

iD1

!miaibi D!m1a1b1 !mkakbk:

Một trường hợp đặc biệt nhóm Abel hữu hạn quan trọng khoa học Máy Tính nói chung làGDZn2 D f0; 1g

n

Mỗi phần tử củaGlà đỉnh khối lập phươngn-chiều, phép gán thật16vàonbiến nhị phân, tập con S Œn đóS tập tọa độ bằng1của phần tử Ở đây, nhómGcóN D2nphần tử, thếN

hàm đặc trưng Do!2 D 1, với cặpa;b2Z2nta cóa.b/D 1/ab

Thay dùnga;bđể đánh số hàm đặc trưng tọa độ chúng, ta dùng tập conA; B Œnđể đánh số, đóA D fi jai D 1g, vàB D fi jbi D 1g Theo cách này, hàm đặc trưng định nghĩa bằngA.B/D 1/jA\Bj Còn dùng b2 Zn2 làm tham số thìA.b/D 1/

P

i2Abi Bạn nên làm quen với việc chuyển qua lại giữa tập vectors củaZn2

Lý quan tâm đến hàm đặc trưng tối giản nhómZn2 sau Ta cần nghiên cứu hàm nhị phân gồmnbiến nhị phân kiểu nhưf W f0; 1gn! f0; 1g Mỗi hàm loại xem vector không gianf0; 1gN Dĩ nhiên, chúng vectors không gianRN vàCN Như phân tích trên, hàm đặc trưng tối giản sở trực

16

(15)

chuẩn khơng gianCN Vì thế, hàm nhị phânnbiến bất kỳ, viết thành vector không gianCN, tổ hợp tuyến tính hàm đặc trưng tối giản

Thay làm việc khơng gian vectorCN, cách tương đương làm việc khơng gian (tuyến tính) hàm sốf W f0; 1gn !C Hàmf W f0; 1gn ! Cbất kỳ

đều biểu diễn dạng

f y/D X SŒn

O

fSS.y/D X

SŒn O

fS 1/

P

i2Syi:

Để đámyi số mũ khó chịu Chúng ta đổi biến Đặtxi D 1/yi Nghĩa

yi D0(FALSE) thìxi D1, cịnyi D1(TRUE) thìxi D Thì ta có phát biểu sau đây:

Bổ đề Mọi hàmf W f 1; 1gn !

Cđều viết dạng f x/D X

SŒn O

fSS.x/;

trong (lạm dụng ký hiệu chút)S W f 1; 1gn! f 1; 1glà hàm đơn thức17

S.x/DS.x1; ; xn/D Y

i2S

xi:

Đám hàmS gọi làhệ sở đơn thứccủa hàmf W f 1; 1gn!C Nhớ hệ

cơ sở đơn thức hệ sở trực chuẩn không gian hàmf W f 1; 1gn!C Trong

đó, “tích vô hướng" hai hàmf; gbất kỳ định nghĩa hf; gi D

2n X

x2f 1;1gn

f x/g.x/DE

x

h

f x/g.x/i;

trong trị kỳ vọng vế phải tính phân bố vectorsx2 f 1; 1gn Chúng ta thấy hiểu tích vơ hướng hai hàm trị kỳ vọng tích hữu dụng sau

3.2 Biến đổi Fourier rời rạc

Ý tưởng biến đổi Fourier rời rạc phát biểu đại số tuyến tính: vector khơng gian vector tổ hợp tuyến tính hệ sở khơng gian (Xem thêmbài Terry Taogiới thiệu biến đổi Fourier nói chung, sách tuyệt vời anh Vũ Hà Văn Terry Tao có ứng dụng giải tích đồng điều tốn tổ hợp [4])

Trong ngữ cảnh chúng ta, hàmf W f 1; 1gn!

Rđều tổ hợp tuyến tính hàm

đơn thức:

f x/D X SŒn

O

fSS.x/;

Tổ hợp Các hệ sốfOS D hf; Sigọi cáchệ số Fouriercủaf Chúng số thực vìf vàS vectors thực Từ trở có thểm làm việc ln khơng gianRN thay vìCN khơng cần liên hợp tính tích vơ hướng hai vectors Hệ sở đơn thứcS gọi làhệ sở Fourier.

Hai đẳng thức biến đổi Fourier 17

(16)

đẳng thức Plancherel

E

x Œf

x/g.x/D hf; gi D X SŒn

O

fSgOS DNh Of ;gOi

trường hợp đặc biệt làđẳng thức Paserval

E

x

f2.x/D hf; fi D X SŒn

O

fS2:

Dễ chứng minh hai đẳng thức từ định nghĩa tính trực chuẩn cácS (Đơi khi, để đảm bảo tính đối xứng người ta định nghĩa tích vơ hướng chia cho bậc hai củaN.)

4 Luật bầu cử biến đổi Fourier cho hàm nhị phân

4.1 Luật bầu cử nói chung

Trong trường hợp hàm nhị phânf W f 1; 1gn ! f 1; 1gthìf2.x/D1với mọix, đẳng thức Parseval nói

X

SŒn O

fS2 D1:

Một hàm nhị phân “luật” bầu cử Cónphiếu bầuxi cho hai ứng cử viên1và Hàm

f trả người thắng cử Sau số hàm (luật) bầu cử hay thấy thực tế:

Majnlà hàm bầu đa số, định nghĩa vớinlẻ, trả về1nếu đa số “phiếu” là1, trả 1nếu đa số phiếu

Dicti hàm độc tài (đã định nghĩa), trả phiếu bầu củaxi, nghĩa Dicti.x/Dxi Const1 Const hàm số (hay hàm “đảng cử, dân bầu"), trả giá trị

đảng cử1hoặc

Ta định nghĩa số hàm khác hàm chẵn lẻ, hàm “electoral college” (như luật bầu cử Mỹ), vân vân Xembài nàycủa Ryan O’Donnell để thêm số ví dụ

Với luật bầu cử định, muốn biết nhiều thuộc tính

Nó có thiên vị khơng?Thiên vị hiểu sau, ta lấy bộnphiếu bầu ngẫu nhiên xác suất mà kết là1hoặc 1khác cỡ Một luật bầu “công bằng” hai xác suất Do đó, ta định nghĩa thiên vị hàmf

P

x Œf x/D1 Px Œf x/D 1DEx Œf x/ :

Đến ta thấy phân tích Fourier có lợi Do hàm; D1, độ thiên vị củaf

Ex Œf x/DEx Œf x/;.x/D Of;:

(17)

Ảnh hưởng phiếu sao?Nếu Tám Tàng đổi phiếu từ 1sang1thì kết bị đổi nào? Với phiếux, gọix˚i phiếu mà ta đổi phiếuxi lại Thì tầm ảnh hưởng (Influence) phiếu thứi kết định nghĩa

Infi.f /WDP

xf x/Ôf x

i/:

Trong lý thuyết chọn lựa xã hội tầm ảnh hưởng gọi làchỉ số sức mạnh Banzhafhoặc số Banzhaf-Penrose index Chỉ số có ảnh hưởng vài phiên tòa bầu cử

Bài tập Chứng minh rằngInfi.f /DPi2SfOS2:

Dễ thấy tầm ảnh hưởng hàm đảng cử là0, tầm ảnh hưởng hàm độc tài

0cho tất trừ anh độc tài có ảnh hưởng bằng1 Tầm ảnh hưởng hàm đa số cơng chút Dùngxấp xỉ Stirlingta tính khoảng

q n

Ảnh hưởng nhiễu sao?Khi ghi lại triệu phiếu bầu xác suất mà phiếu bị ghi sai không bỏ qua Gọi xác suất chẳng hạn Giả sử ta lấy phiếu bầuxhoàn toàn ngẫu nhiên Gọiylà phiếu đạt cách lật phiếuxi với xác suất Dễ thấy, với mọii,

E

xŒxiyiD1 2:

Do cặp.x;y/được gọi là.1 2/-correlated Độ ổn định nhiễucủaf tại.1 2/

được định nghĩa

Stab1 2.f /D Ex;y

.1 2/ cor

Œf x/f y/DE

xŒf x/Df y/ Exf x/Ôf y/:

Ngc li:

E

xf

x/Df y/D

2 C

2Stab1 2.f /:

Bài tập Chứng minh rằng:

Stab1 2.f /D X SŒn

.1 2/jSjfOS2:

Độ ổn định nhiễu hàm đảng cử là1, hàm độc tài thứi là1 Độ ổn định nhiễu hàm đa số thú vị Có thể chứng minh điều sau đây:

lim

n!1Stab1 2.Majn/D1

2

arccos.1 2/:

Nếu ta dùng xấp xỉ arccos.1 2/2p(khá tốt khinhỏ) ta thấy nhiễu

(18)

4.2 Chứng minh định lý Arrow

Giả sử tồn hàmf cho, với phiếu hợp lệ nào.x;y;z/nào, chọn lựa xã hội

.f x/; f y/; f z//cũng lý Ta chứng minh rằngf phải hàm độc tài

Với cá nhâni, chọn ba xi; yi; zi/ngẫu nhiên từ ba hợp lệ.1; 1; 1/,

.1; 1; 1/, 1; 1; 1/, 1; 1; 1/, 1; 1; 1/, 1; 1; 1/ Ta có ba vectors

.x;y;z/hợp lệ Xác suất mà.f x/; f y/; f z//là lý phải bằng1 Khai triển Fourier hàmNAElà

NAE.a; b; c/D

4 4ab 4bc 4ca:

Như vậy, xác suất mà.f x/; f y/; f z//là lý

E

x;y;zŒ

NAE.f x/; f y/; f z/D

4

4EŒf x/f y

4EŒf y/f z

4EŒf z/f x/ :

Dox;y;zcó vai trị nhau, ta kết luận

E

x;y;zŒNAE.f x/; f y/; f z/D

3

3

4EŒf x/f y/ :

Nhớ trị kỳ vọng tính từ cách lấy ba lýx;y;znhư mô tả Từ dễ thấy rằngx;ylà cặp 1=3/-correlated Do

EŒf x/f y/DStab 1=3.f /D X

SŒn

1=3/jSjfOS2:

Để cho gọn, ta định nghĩaWk.f /DPjSjDkjfO

S Nhớ P

SfO

S D1, P

kWk.f /D1 Do đó, xác suất mà.f x/; f y/; f z//là lý

3 4 n X

kD0

1=3/kWk.f /D

3

3

4W0.f /C

4W1.f /

36W2.f /C

Xác suất W1.f / D Wk.f / D với k Ô Nhng

W1.f /D1nu v ch nuf DDicti hoặcf D Dicti vớii Nhưngf phải thỏa tính trí, đóf DDicti Và tính lý độc tài

Chứng minh định lý Arrow phương pháp không vui Chứng minh cũ Arrow không cho biết xác suất phí lý chọn lựa xã hội Phân tích Fourier cho biết, ta dùng hàm đa số, khintiến đến vô xác suất có chọn lựa xã hội lý

3 4

arccos 1=3/

:912:

(19)

Tài liệu tham khảo

[1] ARROW, K J A Difficulty in the Concept of Social Welfare Journal of Political Economy 58(1950), 328

[2] KALAI, G A Fourier-theoretic perspective on the Condorcet paradox and Arrow’s theorem Adv in Appl Math 29, (2002), 412–426.

[3] SAGAN, B E The symmetric group, second ed., vol 203 ofGraduate Texts in Mathematics.

Springer-Verlag, New York, 2001 Representations, combinatorial algorithms, and symmetric functions

(20)(21)

ĐỊNH LÝ KHINTCHINE

Lý Ngọc Tuệ

(Đại học Brandeis, Massachusetts, Mỹ)

1 Giới thiệu

Trong phần [5], có câu trả lời cho câu hỏi khả xấp xỉ số thực số hữu tỉ qua Định lý Dirichlet:

Định lý 1.1(Dirichlet 1948). Với số vô tỉx 2RXQ, tồn vô số số hữu tỉ p

q 2Qsao cho:

ˇ ˇ ˇ ˇ

x p q

ˇ ˇ ˇ ˇ

<

q2; (1.1)

và mở rộng kết không gian véc tơRntrong phần [6] Mặt khác, Định lý Dirichlet chứng minh tối ưu qua tồn số/véc tơ xấp xỉ Nói cách khác, với số vơ tỉx, ta tìm vơ số nghiệm hữu tỉ p

q cho bất đẳng thức (1.1) Tuy nhiên,

Định lý Dirichlet xét chung tất số vô tỉ, xét riêng biệt số vơ tỉx hàm xấp xỉ

q2 khơng phải tối ưu Chẳng hạn cácsố LiouvilleLđược định nghĩa sau: số vô tỉxđược gọi số Liouvillex Lnếu với mọin 1, tồn số hữu tỉ p

q 2Qsao cho:

ˇ ˇ ˇ ˇ

x p q

ˇ ˇ ˇ ˇ

<

qn: (1.2)

Vào năm 1844, nhà toán học Joseph Liouville chứng minh tậpLkhơng rỗng, ví dụ số siêu việt (transcendental numbers) Lưu ý nhưx 2Llà số Liouville với mọin1, bất đẳng thức (1.2) có vơ số nghiệm p

q 2Q

Trong phần này, đưa thêm yếu tố độ đo vào vấn đề khả xấp xỉ số thực số hữu tỉ Nói cách cụ thể hơn, ta thay “Với số" Định lý Dirichlet “Với hầu hết số (theo độ đo Lebesgue)" ta thay hàm số xấp xỉ

q2 hàm số nào? Câu hỏi A Y Khintchine trả lời hoàn toàn vào năm 1924 [1] mở rộng cho không gian véc tơRnvào năm 1926 [2] Kết sau A V Groshev chứng minh cho không gian ma trậnMm;n.R/vào năm 1938

(22)

một số -xấp xỉ được( -approximable) tồn vô số số hữu tỉ p

q Qvớiq > 0sao

cho:

ˇ ˇ ˇ ˇ

x p q

ˇ ˇ ˇ ˇ

< q/

q : (1.3)

Tập số -xấp xỉ được ký hiệu làWA /

Định lý Dirichlet trênRcó thể viết lại theo ký hiệu sau:

Nếu q/D

q thìWA /DR:

Định lý Khintchine cho tập số thựcRđược phát biểu sau:

Định lý 1.2(Khintchine 1924). Giả sử như là hàm xấp xỉ choq q/là hàm không tăng, ký hiệu.E/là độ đo Lebesgue tậpE.

(i) Nếu chuỗi X

nD1

n/hội tụ thì.WA // D0.

(ii) Nếu chuỗi X

nD1

n/phân kỳ thì.RXWA //D0.

Nói cách khác, chuỗi X

nD1

hội tụ với hầu hết tất số thực, bất đẳng thức (1.3)

có vơ số nghiệm hữu tỉ; cịn chuỗi phân kỳ, với hầu hết tất số thực, bất đẳng thức (1.3) có hữu hạn nghiệm hữu tỉ

Lưu ý 1.3. Các kết dạng Định lý 1.2 lý thuyết xấp xỉ Diophantine thường gọi cácĐịnh luật 0-1

Lưu ý 1.4. Một số hệ trực tiếp thú vị Định lý Khintchine sau:

(i) Tập số xấp xỉ kémBAcó độ đo Lebesgue

(ii) Với hầu hết số thựcx, bất phương trình: ˇ

ˇ ˇ ˇ

x p q

ˇ ˇ ˇ ˇ

< q2logq có vơ số nghiệm hữu tỉ

(iii) Với > 0bất kỳ với hầu hết số thựcx, bất phương trình: ˇ

ˇ ˇ ˇ

x p q

ˇ ˇ ˇ ˇ

< q2.logq/1C có hữu hạn nghiệm hữu tỉ

(23)

2 Phần hội tụ Định lý Khintchine

Có nhiều tài liệu tham khảo cho phần Ở giới thiệu số định nghĩa tính chất để dẫn đến Bổ đề Borel-Cantelli

2.1 Độ đo Lebesgue

Độ đo Lebesgue không gianRnlà mở rộng khái niệm độ dài (nD1), diện tích (nD2), thể tích (n3) TrênR, đoạn thẳng.a; b/là thước đo để đo độ dài tập hợp, độ đo đoạn.a; b/được định nghĩa bởi: a; b//WDb a Độ đo (ngoài) tập

E Rbất kỳ xây dựng cách sử dụng số đếm đoạn thẳng để phủ lên tập E với tổng độ dài nhỏ tốt:

.E/WDinf ( 1

X

iD1

.Ii/WE [

iD1

Ii vớiIi đoạn thẳng )

:

Một tập conE Rđược gọi mộttập đo đượcnếu với mọiAR: A/D.A\E/C.A\.RXE//:

Tập tập đo tạo thành một-đại sốthỏa mãn tính chất sau: (1) Tập rỗng;vàRlà tập đo

( 2) Nếu nhưE tập đo phần bùRXE tập đo

( 3) Nếu nhưA1; A2; :::là tập đo [

iD1

Ai tập đo

Độ đo Lebesgue tập đo thỏa mãn tính chất sau:

(M0) ;/D0

(M1) Nếu nhưAB tập đo thì.A/.B/

(M2) Nếu nhưA1; A2; :::là dãy tập đo rời cặp thì:

1 [

iD1

Ai !

D X

iD1

.Ai/:

Lưu ý 2.1. Áp dụng Tiên đề chọn (Axiom of Choice), ta xây dựng tập củaR khơng đo chẳng hạn tập Vitaly tập Bernstein Tập không đo dẫn đến số nghịch lý nghịch lý Banach-Tarski Mặt khác, ta bỏ Tiên đề chọn, Solovay [4] chứng minh tồn mơ hình tập số thực mà đó, tập tập đo

Lưu ý 2.2. Ta nói tính chất P thỏa mãn với hầu hết số x, tập fx R W xkhơng thỏa mãn tính chấtPgcó độ đo Lebesgue

(24)

Bài tập 2.4. Chứng minh nếuE Rlà tập đến thì.E/D0

Bài tập 2.5. Tìm độ đo Lebesgue tập Cantor:

C WD (

x 2Œ0; 1Wx D X

iD1

ai

3i; f0; 2g )

:

Bài tập 2.6. Tìm độ đo Lebesgue tập LiouvilleL

2.2 Tập limsup Bổ đề Borel-Cantelli

Gọi X không gian Một họB tập củaX thỏa mãn tính chất tương tự (1)–( 3) gọi một-đại số tập đo trên X Một hàm không âm

W B ! R0 thỏa mãn tính chất M(0)–M(2) gọi độ đo trênX Bộ

.X;B; /được gọi mộtkhông gian đo.

Nếu như0 < X / <1, ta thay độ đobằng độ đo0.E/D E/

.X /

0.X /D1. Không gian đo.X;B; /với.X /D1được gọi mộtkhông gian xác suất Trong lý thuyết xác suất, tập đo đượcE 2Btương ứng với kiện, độ đo.E/củaE tương ứng với xác suất để kiệnE xảy

Với dãy tập conE1; E2; :::củaX, ta định nghĩa tập limsup dãy sau:

lim sup n!1

En WD \

nD1 [

kDn

Ek D˚x 2X W có vơ số i’s chox2 Ei :

Nói cách khác, E D lim sup n!1

En kiện "có vơ số kiện En xảy ra" Bổ đề Borel–Cantelli phát biểu sau:

Bổ đề 2.7(Bổ đề Borel-Cantelli). Cho.X;B; /là không gian xác suất, vàE1; E2; :::;2B là kiện Nếu chuỗi

1 X

nD1

.En/hội tụ, thì:

lim sup n!1

En

D0:

Bài tập 2.8. Chứng minh Bổ đề 2.7

Bài tập 2.9. Tìm phản ví dụ cho mệnh đề đảo Bổ đề 2.7

Trở lại với Định lý Khintchine, để áp dụng Bổ đề Borel-Cantelli, ta cần có khơng gian xác suất, biểu diễn tập có số -xấp xỉ đượcWA /dưới dạng tập limsup

(25)

Áp dụng tập trên, để chứng minh Định lý Khintchine, ta cần tập trung vào số nằm đoạnŒ0; 1/ Giới hạn độ đo Lebesgue đoạnŒ0; 1/sẽ cho ta không gian xác suất Đặt

EnD

x 2RW

ˇ ˇ ˇx

p n

ˇ ˇ ˇ<

n/ n

;

ta biểu diễn tậpWA /dưới dạng tập limsup sau: WA /Dlim sup

n!1

En:

Bài tập 2.11. Tìm độ đo Lebesgue củaEn\Œ0; 1(tập bao gồm hữu hạn đoạn thẳng)

Bài tập 2.12. Chứng minh chuỗi X

nD1

n/hội tụ chuỗi X

nD1

.En\Œ0; 1//hội tụ

Áp dụng Bài tập 2.10 Bổ đề Borel-Cantelli, ta có phần hội tụ Định lý Khintchine

3 Phần phân kỳ Định lý Khintchine

Để chứng minh phần phân kỳ Định lý 1.2, sử dụng công cụ tốt cho xấp xỉ mà ta có tập số thực: liên phân số

3.1 Một số điều Liên phân số

Nhắc lại phần [5], gọi liên phân số hữu hạn có độ dài.nC1/là biểu thức có dạng:

ŒaoIa1; :::; anWDa0C

1 a1C

1 : ::C

an với dãy số thực hữu hạna02 R; a1; :::; an2 RX f0g

Khia0 2Z; a1; :::; an2N, ta gọi biểu thức mộtliên phân số đơn hữu hạn Với dãy a0 Z; a1; a2; :::2 N, dãy liên phân số đơn hữu hạnŒa0Ia1; :::; anlà dãy hội tụ

n! 1, ta ký hiệu giới hạn là:

Œa0Ia1; :::WD lim

n!1Œa0Ia1; :::; an:

Mỗi số vô tỉx2RXQcó thể biểu diễn dạng liên phân số đơn vô hạn: x DŒa0Ia1; ::::

Với cách biểu diễn trên, phân số hội tụ thứncủaxlà:

pn

qn

D pn.x/

qn.x/

(26)

với pn

qn

ở dạng tối giản

ĐặtxnC1DŒanC1IanC2; :::, có công thức sau:

Bổ đề 3.1. Với mọin0:

x D xnC1pnCpn

xnC1qnCqn

:

Tuy trơng phức tạp, biểu diễn số thực liên phân số có tính chất sau giống với biểu diễn số hệ thập phân:

Bổ đề 3.2. Giả sử nhưx DŒa0Ia1; :::y DŒb0Ib1; :::.

pn

qn

< y < pnCpn qnCqn

khi khib0 Da0; :::; bnDan:

Hay nói cách khác, tập số có biểu diễn liên phân số có phần đầu cố định

J.a0; a1; :::; an/WD fy DŒa0Ia1; :::; an; rWr R; r 1g

là đoạn thẳng:

J.a0; a1; :::; an/D

pn

qn

;pnCpn qnCqn

;

J.a0; :::; an/\J.b0; :::; bn/Ô ;khi v ch khib0 Da0; :::; bnDan

Bài tập 3.3. Chứng minh Bổ đề 3.2

Bổ đề 3.4. Với ký hiệu trên, với mọia0 2Z; a1; :::; an; k2 N:

3k2 <

.J.a0; :::; an; k//

.J.a0; :::; an//

< k2: Chứng minh. Áp dụng Bổ đề 3.1 3.2, đẳng thức:

pnqn pn 1qnD 1/n; ta có được:

.J.a0; :::; an//D ˇ ˇ ˇ ˇ pn qn

pnCpn

qnCqn ˇ ˇ ˇ ˇ

D

qn.qnCqn 1/ D

q2 n

1Cqn

qn :

.J.a0; :::; an; k//D ˇ ˇ ˇ ˇ

pnkCpn

qnkCqn

pn.kC1/Cpn

qn.kC1/Cqn ˇ ˇ ˇ ˇ

D

.qnkCqn 1/.qn.kC1/Cqn 1/

D

q2 nk2

1Cqn

kqn

1C

k C qn

kqn :

Lưu ý qn

kqk

qn

qk

(27)

3.2 Định luật 0-1 Liên phân số

Vận dụng số tính chất liên phân số trên, chứng minh Định luật 0-1 Liên phân số Định luật bổ đề quan trọng dùng để chứng minh phần phân kỳ Định lý Khintchine

Định lý 3.5. Giả sử nhưf n/là hàm số dương bất kỳ, đặt:

FnD fx DŒa0Ia1; :::Wanf n/g:

(i) Nếu chuỗi X

nD1

1

f n/ hội tụ thì

lim sup n!1

Fn

D0.

(ii) Nếu chuỗi X

nD1

1

f n/ phân kỳ thì

RXlim sup

n!1

Fn

D0.

Chứng minh. Chúng ta chứng minh phần phân kỳ, phần hội tụ để dành làm tập dành cho độc giả Lưu ý trường hợp, giới hạn sốx 2Œ0; 1/ Xét dãya1; :::; amCnbất kỳ cho với1i n:

amCi f mCi /: Ta có được:

0

@

[

kf mCnC1/

J.0; a1; :::; amCn; k/

AD

X

kf mCnC1/

.J.0; a1; :::; amCn/ (Bổ đề 3.2)

>

3.J.0; a1; :::; amCn/

X

kf mCnC1/

1

k2 (Bổ đề 3.9)

>

3.J.0; a1; :::; amCn/

Z

f mCnC1/C1

dx x2

D

3.f mCnC1/C1/.J.0; a1; :::; amCn/:

Mặt khác,

J.0; a1; :::; amCn/D [

kD1

J.0; a1; :::; amCn; k/:

Từ ta suy ra:

0

@

[

k<f mCnC1/

J.0; a1; :::; amCn; k/

A<

1

3.f mCnC1/C1/

.J.0; a1; :::; amCn/:

Đặt

Em;n D

[

.a1;:::;amCn/2NmCn amCi<f mCi /;1in

(28)

bất đẳng thức cho ta:

.Em;nC1/ <

1

3.f mCnC1/C1/

.Em;n/:

Lần lượt áp dụng bất đẳng thức này, ta có được:

.Em;n/ < Em;1 n Y

iD2

1

3.f mCi /C1/

< Em;1exp n X

iD2

1

3.f mCi /C1/

!

.1Cx < ex/

Bài tập 3.6. Chứng minh chuỗi X

nD1

1

f n/ phân kỳ (! 1) dẫn đến chuỗi

n X

iD2

1

3.f mCi /C1/

phân kỳ (! 1) với mọim

Vì vậy, vế phải hội tụ khintiến đếnC1, với mọim; n, ta có được:

.Em;n/D0:

Lưu ý rằngŒ0; 1/X [

nDmC1

Fn D [

nD1

Em;n,

Œ0; 1/Xlim sup m!1

Fm

D Œ0; 1/X \

mD1 [

nDm

Fn !

D

1 [

mD0

Œ0; 1/X [

nDmC1

Fn !!

D

1 [

mD0 [

nD1

Em;n !

X

mD0 X

nD1

.Em;n/D0:

Bài tập 3.7. Chứng minh rằng:

0

@ [

kf nC1/

J.a0; :::; an; k/

A<

4

f nC1/.J.a0; :::; an//:

Bài tập 3.8. Áp dụng tập 3.7 để chứng minh

.FnC1\Œ0; 1// <

4 f nC1/:

(29)

3.3 Chứng minh phần phân kỳ Định lý Khintchine

Bổ đề 3.9. Tồn số tuyệt đốiC > 0sao cho với hầu hết sốxDŒa0Ia1; :::, tồn tạiN > 0sao cho với mọinN,

qn < eC n:

Chứng minh. Đặt

En.g/D

[

a1a2:::ang

J.0; a1; :::; an/:

.J.0; a1; :::; an// D ˇ ˇ ˇ ˇ pn qn

pnCpn

qnCqn ˇ ˇ ˇ ˇD

1 qn.qnCqn 1/

< q2

n

< a1a2:::an/2

ta có cận cho độ đo củaEn.g/:

.En.g// <

X

a1a2:::ang

1 a1a2:::an/2

:

Tích bên tay phải bị chặn tích phân sau:

n Y

iD1

1 a2i D

n Y

iD1

1C

ai

1

ai.aiC1/

2n

n Y

iD1

1 ai.ai C1/

D2n

n Y

iD1

Z aiC1

ai

dxi

xi2

D2n

Z a1C1

a1

Z a2C1

a2

: : :

Z anC1

an

dx1dx2: : : dxn

x12x22: : : x2 n

GọiIn.g/là tích phân:

Z Z

: : :

Z dx

1dx2: : : dxn

x12x22: : : xn2 miền:

x1; :::; xn1; x1x2: : : xng; ta có được:

.En.g// < 2nIn.g/:

Bài tập 3.10. Chứng minh khig 1,In.g/D1

Bài tập 3.11. Chứng minh khig > 1,

In.g/D

1 g

n X

iD0

(30)

Đặtg DeAn, với sốA > 1bất kì, ta có được:

En eAn

< en.ln2 A/

n X

iD0

.An/i i Š

< en.ln2 A/n.An/

n

nŠ < Ben.ln2 A/ n.An/

n

nne npn (công thức Stirling)

< B0pne n.A lnA ln2 1/:

VớiAđủ lớn,

A lnA ln2 > 0;

và đó, chuỗi X

nD1

En eAn

hội tụ Theo Bổ đề Borel-Cantelli, với hầu hết sốxtrong

đoạnŒ0; 1/,xchỉ nằm hữu hạn tậpEn eAn

Hay nói cách khác, với hầu hết số khoảngŒ0; 1/, với mọinđủ lớn,

a1a2:::an< eAn:

Khi đó,

qn Danqn 1Cqn < 2anqn < ::: < 2nanan 1:::a1< 2neAn DeC n

vớiC DACln2

Chứng minh Định lý Khintchine. Giả sử chuỗi X

nD1

n/phân kỳ, đặt:

f x/DeC x eC x

;

vớiC số có từ Bổ đề 3.9 Tích phân:

Z b

a

f x/dxD

C

Z C b

C a

u/du

với0 < a < btiến đến vô khib ! 1, hàmf x/khơng tăng theo giả thuyết, ta có chuỗi

1 X

nD1

f n/phân kỳ

Áp dụng Định lý 3.5, với hầu hết sốxDŒa0Ia1; :::, tồn vô số sốnsao cho:

anC1

1 f n/:

Từ suy ra:

ˇ ˇ ˇ ˇ

x pn qn ˇ ˇ ˇ ˇ

1 qnqnC1

anC1qn2

f n/

(31)

Theo Bổ đề 3.9, với hầu hết sốx DŒa0Ia1; :::và với mọinđủ lớn,

qn < eC n () n > lnqn

C :

Vì vậy, ta có được, hầu hết mọixDŒa0Ia1; :::, tồn vô số sốnsao cho:

ˇ ˇ ˇ ˇ

x pn qn ˇ ˇ ˇ ˇ

f n/ q2

n

f

lnqn

C

q2 n

D qn/

qn

;

tức làx 2WA /

Ti liu tham kho

[1] A Y Khintchine,Einige Săatze ¨uber Kettenbr¨uche, mit Anwendungen auf die Theorie der Diophantischen Approximationen, Math Ann.92(1924), pp 115–125

[2] A Y Khintchine,Zur metrischen Theorie der Diophantischen Approximationen, Math. Zeitschrift24(1926), pp 706–713

[3] A Y Khintchine,Continued Fractions(1935)

[4] R M Solovay,A model of set-theory in which every set of reals is Lebesgue measurable, Ann of Math.92(1970), pp 1–56

[5] Lý Ngọc Tuệ,Xấp xỉ Diophantine trênRvà Liên phân số, Epsilon4, (2015).

[6] Lý Ngọc Tuệ,Xấp xỉ Diophantine trênRn- Quy tắc Dirichlet Hình học số, Epsilon

(32)(33)

Đàm Thanh Sơn

Được đồng ý giáo sư Đàm Thanh Sơn, số Epsilon trân trọng gửi đến độc giả dịch giáo sư từ bàiGeometría y entrelazamiento cntico Juan Maldacena,Investigación y Ciencia, số11=2015 Bản gốc độc giả xem trang nhà giáo sư ởđây

1 Giới thiệu

Vào đầu kỷXX có hai cách mạng vật lý: Cơ học lượng tử thuyết tương đối rộng Cơ học lượng tử cho ta biết định luật chi phối giới vi mô, thuyết tương đối rộng, Einstein xây dựng năm1915, lý thuyết không gian thời gian Theo thuyết tương đối rộng, khơng-thời gian có độ cong tĩnh, mà động

Tới tiên đoán hai lý thuyết thực nghiệm xác nhận Tuy nhiên hai lý thuyết thường áp dụng vào tượng khác Ta thường dùng học lượng tử để mô tả vật nhỏ (như nguyên tử hay photon), dùng thuyết tương đối rộng để nghiên cứu thay đổi không thời gian gần vật nặng (ví dụ ngơi hay thiên hà) Để nghiên cứu hệ vật lý vừa nặng vừa nhỏ, vũ trụ sau vụ nổ lớn, ta cần cách miêu tả lượng tử cho không thời gian Điều này, trăm năm sau ngày Einstein xây dựng thuyết cịn thách thức lớn cho vật lý

Hai năm trước, khích lệ tranh luận tính chất lỗ đen, nhà vật lý Leonard Susskind đại học Stanford tác giả đề xuất mối liên hệ hai tượng nghịch lý học lượng tử thuyết tương đối rộng: Hiện tượng rối lượng tử (quantum entanglement) lỗ giun (wormholes) Rối lượng tử dạng tương quan lượng tử tồn hệ vật lý cách xa Lỗ giun đường tắt xuất số nghiệm phương trình Einstein nối vùng xa không gian

Dưới thấy hai tượng có liên quan với Sự tương đương tạm thời có chứng minh chặt chẽ vài trường hơp cụ thể, có lẽ trường hợp tổng quát Ý tưởng chúng tơi mối liên hệ hình học rối lượng tử nguyên tắc mà tất lý thuyết lượng tử không-thời gian, hay hấp dẫn lượng tử, phải tuân theo Nguyên tắc có hệ quan trọng Thậm chí, cách đó, khơng-thời gian xuất từ rối lượng tử thành phần vi mô giới

(34)

2 Lỗ đen lỗ giun

Một tiên đoán đáng kinh ngạc lý thuyết Einstein lỗ đen Lỗ đen nhình thành ta gom khối lượng vật chất lớn vào vùng nhỏ không gian Vật chất không cần đặc biệt, ví dụ từ khơng khí ta tạo lỗ đen Chỉ có điều cần nhiều khơng khí: Ta cần phải làm đầy hình cầu có kích thước kích thước hệ mặt trời Nếu ta làm khối khí suy sụp sức nặng nén lại trở thành lỗ đen

Tất lỗ đen bao bọc mặt cầu giả tưởng gọi chân trời kiện Ta gọi mặt cầu giả tưởng nhà du hành vũ trụ rơi tự vào lỗ đen khơng thấy chỗ Tuy nhiên, vượt qua mặt cầu này, người khơng quay trở lại Người vào vùng mà khơng gian suy sụp vào “kỳ dị”, chỗ mà hình học co lại hoàn toàn Tới gần điểm kỳ dị nhà du hành vũ trụ chết bẹp lực hấp dẫn

Bên vùng chứa vật chất, lỗ đen mơ tả nghiệm phương trình Einstein mà nhà vật lý Karl Schwarzschild tìm năm1916:Mục tiêu ban đầu Schwarzchild tìm trường hấp dẫn chất điểm Trên thực tế, nghiệm ông ta khơng có vật chất: Nó mơ tả trường hấp dẫn tuý với đối xứng cầu, không không Tuy trơng đơn giản, tính chất khơng-thời gian khó diễn giải Chỉ đến năm 196o người ta hiểu tương đối cấu trúc toàn cục nghiệm

Năm1935;ở hai báo nói đến trên, Einstein Nathan Rosen, cộng tác viên Einstein Viện Nghiên cứu Cao cấp Princeton, tìm khía cạnh thú vị nghiệm Schwarzschild Họ tìm nghiệm chứa hai không gian độc lập nối với nhà “ống” Tại thời điểm định, ta hìng dung hình học nghiệm sau: Ở xa vùng trung tâm, khơng gian phẳng (khơng có độ cong đáng kể), vào gần trung tâm, hình học bị méo nối vào không gian thứ hai, không gian tiệm cận phẳng

Sự kết nối hình học mà Einstein Rosen tìm gọi “cầu Einstein-Rosen”.ER/;hay lỗ giun Einstein Rosen phân tích cấu trúc hình học siêu diện thời điểm cố định (nói cách khác, khơng gian cong ba chiều) nhiều năm trước ta hiểu cấu trúc toàn cục nghiệm Schwarzschild Mục tiêu Einstein Rosen tìm miêu tả hình học cho hạt khơng có kỳ dị Ngày biết diễn giải họ sai lầm

(35)

nhau bên lại nối với Ta tưởng tượng có lỗ đen nơi ta sống lỗ đen ngân hà khác Một người quan sát mà ta gọi Romeo đứng cách chân trời kiện lỗ đen thứ mét, lúc Juliet đứng cách đứng cách chân trời kiện lỗ đen thứ hai cũng1mét Nếu ruột hai lỗ đen nối với cầu

ER;khoảng cách Romeo Juliet qua lỗ giun là2mét, hai vùng không gian xung quanh hai lỗ đen xa đến mức

Những kiểu hình học có vấn đề Ta nhớ lại nguyên lý thuyết tương đối hẹp ta gửi tín hiệu nhanh tốc độ ánh sáng Thế lỗ giun cho phép vi phạm ngun lý ta gửi tín nhiệu qua Tuy nhiên, năm

1963;Robert W Fuller đại học Columbia John A Wheeler đại học Princeton chứng minh dùng cầuERđể gửi loại tín hiệu Để thấy điều ta phải xem xét tính chất động hình học thời gian đóng vai trị quan trọng Lỗ giun mơ tả hình học không gian thời điểm cố định Nhưng hình học tiến hố theo thời gian Fuller Wheeler chứng minh cầuERgiãn – độ dài cầu trở thành vô – trước người quan sát kịp vượt Điều làm nhân vật phim khoa học viễn tưởng thắt vọng, họ hay dùng lỗ giun để du hành vũ trụ với tốc độ lớn tốc độ ánh sáng

Trong trường hợp hai lỗ đen nối với bên lỗ giun, chân trời hai lỗ đen chạm vào khoảnh khắc, sau rời nhanh kịp vượt cầu sang bên Như Romeo muốn gửi thông điệp nhanh tốc độ ánh sáng cho Juliet, làm Romeo phóng tên lửa mang thơng điệp vào lỗ đen bên tên lửa rơi vào bên lỗ đen Thuy nhiên, bên trong, hai chân trời chạy khỏi với tốc độ nhanh, không gian suy sụp trước thơng điệp tới chân trời Juliet

(36)

Ta phải nhấn mạnh lỗ giun khác lỗ giun thường gặp phim khoa học viễn tưởng Những lỗ giun phim (những lỗ giun ta qua được) địi hỏi loại vật chất có lượng âm, có lẽ khơng tương thích với định luật vật lý ta biết Vì nhiều nhà vật lý tin loại lỗ giun phim khoa học viễn tưởng tồn tự nhiên Các lỗ đen ta xét cịn có khía cạnh khác đáng nhắc đến Các lỗ đen tạo vật chất suy sụp tương ứng với phần hình học Schwarzchild, có mặt vật chất làm nghiệm thay đổi Các lỗ đen loại nghiên cứu rõ; trường hợp khơng có lỗ giun hết Loại lỗ đen tạo qua trình vật lý thiên văn, ví dụ ngơi suy sụp, loại khơng có lỗ giun nối với vùng khác không gian hay nối chúng với nhau, khác với nghiệm đầy đủ Schwarzschild Tuy muốn hiểu rõ diễn giải vật lý không-thời gian Schwarzschild Dù sao, nghiệm đơn giản phương trình Einstein

3 Tương quan lượng tử

Đáng ngạc nhiên giải thích cho nghiệm Schwarzschild lại liên quan đến báo thứ hai Einstein ta nhắc tới Cơng trình ngày tiếng có ảnh hưởng Bài viết năm, với đồng tác giả Rosen Boris Podolsky, nhà nghiên cứu Viện Nghiên cứu Cao cấp Các tác giả (ngày biết đến tên viết tắtEPR) học lượng tử cho phép loại tương quan (correlation) lạ hệ vật lý xa nhau, mối tương quan mà sau gọi “rối lượng tử”

Sự tương quan vật xa xảy vật lý cổ điển Giả sử bạn khỏi nhà mà mang theo găng tay bạn quên nhà Trước nhìn vào túi, bạn khơng biết mang Tuy nhiên, bạn nhìn vào túi thấy mang găng phải, bạn biết nhà găng trái

Tuy nhiên, rối lượng tử liên quan đến tương quan giũa đại lượng lượng tử, đại lượng phải tuân thủ nguyên lý bất định Heisenberg Nguyên lý nói có cặp biến số mà ta khơng thể biết hồn tồn xác lúc Thí dụ tiếng vị trí vận tốc hạt: Nếu ta đo xác vị trí vận tốc trở thành khơng xác định ngược lại Trong báo mình,EPRhỏi xảy ta có hai hệ xa hệ ta định đo cặp biến chịu nguyên lý bất định

Trong ví dụ màEPRphân tích, ta xét hai hạt có khối lượng chuyển động chiều Ta gọi hai hạt làRvàJ ta chuẩn bị hai hạt cho trọng tâm chúng có toạ độ xác định, tức làXcm DxRCxJ D0:Ngoài ta làm cho vận tốc tương đối hai hạtvrel DvR vl;có giá trị xác, ví dụvrel Dv0:Trước tiếp tục, ta phải làm rõ điều Ta vừa đặt giá trị xác cho vị trí vận tốc Điều có vi phạm ngun lý bất định Heisenberg không? Ta nhớ lại ngun lý bất định nói đến vị trí hệ vật tốc liên quan đến vị trí Tuy nhiên, có hai hệ, khơng cấm ta biết vị trí hệ thứ vận tốc hệ thứ hai Trong trường hợp xét, khơng xác định vị trí vận tốc trọng tâm, mà vị trí trọng tâm vận tốc tương đối hai hạt Do hai đại lượng độc lập, khơng có vấn đề ta xét trạng thái ban đầu nhưEPRmuốn

(37)

này chuẩn bị, Juliet nhận giá trịxJ Romeo tìm hạt vị trí

xR D xJ:Mặt khác, họ đo vận tốc Juliet nhận kết quảxJ Romeo chắn tìm giá trịvR D v0CvJ:Tất nhiên Romeo Juliet muốn chọn đo đại lượng Nhưng Juliet đo vị trí Romeo đo vận tốc, kết họ hoàn toàn ngẫu nhiên khơng có tương quan hết

Cái lạ Juliet định đo vị trí hạt hạt Romeo có vị trí hồn tồn xác định ta biết kết đo Juliet Cũng hai người đo vận tốc Ta nghĩ Juliet đo vị trí, hạt Romeo “biết” phải chiếm vị trí xác định Mới nhìn điều truyền thơng tin tức thì: Lặp lặp lại thí nghiệm nhiều lầm, Juliet gửi cho Romeo thơng điệp gồm số0và1bằng cách chọn đo vị trí hay tốc độ hạt Tuy vậy, Romeo đọc thông điệp kết đo Juliet Do ta khơng thể dùng tương quan rối lượng tử để gửi tín hiệu nhanh ánh sáng

Rối lượng tử tính chất bí mật hệ lượng tử, nhiên qua nhiều năm tháng kiểm chứng thực nghiệm Trong hai mưoi năm trở lại đây, tương quan lượng tử đưa đến ứng dụng thực tế tiến lớn ngành mật mã thông tin lượng tử

4 ER = EPR

Ta quay lại lỗ đen Năm1974Stephen Hawking chứng minh hiệu ứng lượng tử làm cho lỗ đen phát xạ giống vật nóng Điều chứng tỏ lỗ đen có nhiệt độ Nhiệt độ cao vật nhỏ Thật lỗ đen trắng Cụ thể, lỗ đen với kích thước vi khuẩn, với xạ có bước sóng giống bước sóng ánh sáng nhìn thấy, có màu trắng xạ Hawking Lỗ đen không phát nhiều ánh sáng, đến gần ta thấy điểm sáng chói Nhưng lỗ đen kích thước có khối lượng khổng lồ, ta khơng thể sử dụng nguồn lượng

Đối với lỗ đen tạo cách tự nhiên từ suy sụp sao, xạ Hawking yếu đến mức thực tế quan sát Những vật thể to lạnh để cảm nhận hiệu ứng Tuy nhiên, việc lỗ đen có nhiệt độ có hệ quan trọng

Chúng ta biết từ kỷXIX nhiệt độ có chuyển động phần tử vi mơ hệ Ví dụ, chất khí, nhiệt độ nảy sinh chuyển động phân tử khí Vì ta chờ đợi lỗ đen có chứa thành phần vi mơ có khả tạo vơ số cấu hình, “trạng thái vi mô” Chúng ta tin rằng, nhìn từ bên ngồi, lỗ đen phải hành xử hệ lượng tử bình thường, chịu tất định luật học nhiệt động học

Xem xét từ bên trong, khơng có cấm ta xét trạng thái rối lượng tử lỗ đen Ta tưởng tượng hai lỗ đen cách xa nhau, lỗ đen có số lớn trạng thái vi mơ Ta nghĩ cấu hình trạng thái vi mơ lỗ đen thứ có tương quan với trạng thái vi mô tương ứng lỗ den thứ hai Cụ thể ta quan sát lỗ đen thứ trạng thái vi mô định, lỗ đen thứ hai trạng thái

(38)

trên – tức trạng tháiEPR– cho ta không thời gian có cầuERnối hai lỗ đen với Nói cách khác rối lượng tử gây liên kết hình học hai lỗ đen

Chúng tơi gọi sư tương đương giữaERvàEPR;hàyERDEPR;vì liên hệ hai báo Einstein cộng viết năm1935:Từ quan điểm củaEPR;các quan sát gần chân trời hai lỗ đen có tương quan với rối lượng tử Từ quan điểm củaER;các đo đạc có tương quan với khoảng cách hai hệ qua lỗ giun nhỏ Để xác lập tương đương, điều quan trọng ta gửi thông tin qua lỗ giun, ta gửi thơng tin dùng rối lượng tử

Ta nghĩ tới tương lai xa hai gia đình thù địch muốn giữ Romeo Juliet xa Họ gửi Romeo đến Tinh vân Tiên nữ giữ Juliet dải Ngân hà Tuy nhiên họ cho phép hai người trao đổi thông điệp cặp hệ lượng tử rối với Việc đòi hỏi rấtt nhiều thời gian, tương lai mà tuổi thọ cao nhiều Với kiên nhẫn, Romeo Juliet làm hai lỗ đen rối với Những lỗ đen nhìn từ ngồi hồn tồn bình thường, hai gia đình khơng nghi ngờ Tuy nhiên, sau làm hai lỗ đen, Romeo Juliet nhảy bên gặp lần cuối trước chết điểm kỳ dị

5 Một nguyên lý phổ quát?

Những ý tưởng dẫn ta tới nhiều nhà nghiên cứu phát triển qua nhiều năm, nghiên cứu Werner Israel đại học Alberta Cơng trình tơi Susskind khích lệ nghịch lý Ahmed Almheiri, Donald Marolf, Joseph Polchinski James Sully, tất lúc đại học California Santa Barbara, phát biểu năm2012:Ngược lại với người nghĩ trước đó, nhà nghiên cứu đưa ý kiến tượng rối lượng tử buộc ta phải thay chân trời kiện lỗ đen (một mặt cầu êm, theo lý thuyết Einstein), rào chắn lượng cao vượt qua Từ quan điểm mối liên hệERDEPRta giải nghịch lý

Sự tương đươngERDEPRgợi ý có rối lượng tử ta có mối liên kết hình học Điều phải kể trường hợp đơn giản ta có hai hạt rối với Tuy nhiên trường hợp mối liên kết hình học phải có cấu trúc nhỏ lượng tử, không giống khái niệm hình học bình thường Dù cịn chưa biết mơ tả hình học vi mơ nào, ý tưởng chúng tơi mối liên hệERDEPRcho ta nguyên lý mà tất lý thuyết hấp dẫn lượng tử phải tuân theo Lý thuyết lượng tử hấp dẫn nghiên cứu nhiều lý thuyết dây Trong lý thuyết dây, mối liên hệERDEPR

có thể chứng minh cách chặt chẽ số trường hợp mà rối lượng tử có dạng định, nhiên khơng có đồng thuận mội liên hệ thoả mãn tất trường hợp

(39)

I I Blekman, A D Myshkis, Ya G Panovko

Bài viết trích từ "Tốn học Ứng dụng", nhà xuất Khoa học Kĩ thuật Hà Nội, 1985 Đây dịch Việt ngữ Trần Tất Thắng từ gốc tiếng Nga ba tác giả I I Blekman, A D Myshkis, Ya G Panovko (độc giả đối chiếu thêm với tựa tiếng Anh sách “Mechanics and Applied Mathematics: Logics and Special Features of the Applications of Mathematics")

Phương hướng ứng dụng lý thuyết phát triển toán

học

Hai nguồn toán học bản

Phương hướng ứng dụng lý thuyết:Vị trí tốn học ứng dụng trở nên rõ ràng theo dõi sơ đường phát triển thân toán học Rõ ràng động lực phát triển toán học có hai nguồn tồn cách khách quan Một nguồn bên việc cần thiết phải dùng phương tiện toán học để giải tốn nằm ngồi phạm vi tốn học, toán khoa học khác, kỹ thuật, kinh tế, nguồn mặt lịch sử Nguồn thứ hai nguồn bên việc cần thiết phải hệ thống hóa kiện tốn học khám phá được, giải thích mối liên hệ chúng với nhau, hợp chúng lại quan niệm khái quát lý luận, phát triển lý luận theo quy luật bên nó, nguồn thời điểm dẫn tới chỗ tách toán học thành khoa học.1

Tất nhiên đơi khó phân giới nguồn Ví dụ nguồn thúc đẩy nảy sinh việc áp dụng phương pháp lĩnh vực toán học lĩnh vực khác2đôi lại giống nguồn thúc đẩy thâu nhận áp dụng phương tiện tốn học ngồi phạm vi tốn học Mặt khác, việc hệ thống hóa có nhu cầu thực tiễn trực tiếp

Vì mạo hiểm xác định chi tiết ranh giới hai nguồn Tuy nhiên đặc điểm nguồn ảnh hưởng chúng đại phận trường hợp dễ dàng nhận thấy Hai phương hướng phát triển toán học ứng với hai nguồn gọi phương hướng ứng dụng phương hướng lý thuyết (thuần túy)

Xin nhấn mạnh muốn nói ảnh hưởng chiếm ưu việc xây dựng phát triển phương pháp toán học, khái niệm khẳng định Còn

1

Trong lời giới thiệu cho sách phổ biến tiếng R.Courant G.Robbins [28] nói: “Rõ ràng vận động lên lĩnh vực toán học xuất nhu cầu mà mức độ nhiều hay có mang tính chất thực tiễn Nhưng xuất động phải có khn khổ nội vượt ngồi phạm vi tính hữu ích trực tiếp Chính biến đổi hoàn toàn khoa học ứng dụng thành khoa học lý thuyết thấy lịch sử cổ đại ngày mức độ hơn: Người ta thừa nhận đống góp kỹ sư nhà vật lý vào toán học đại.”

2

(40)

bản chất toán học xây dựng vấn đề thuộc phương hướng - lý thuyết hay ứng dụng - thường vô nghĩa Nên quy phương pháp Bubnov - Galeckin khái niệm hàm delta Dirac, công thức Taylor thuộc phương hướng nào? Chỉ trả lời câu hỏi muốn nói lịch sử phát sinh khái niệm hồn cảnh cụ thể bắt gặp chúng Đúng số chương đại số đồng điều chẳng hạn hồn tồn thuộc phương hướng lý thuyết vấn đề phương pháp chọn xác suất xác định khả thực tiễn kiện, khái niệm tính hội tụ thực tiễn hay vơ hạn thực tiễn thời hồn tồn thuộc phương hướng ứng dụng (ở ngẫu nhiên mà có từ “hiện nay, thời”)

Giai đoạn đầu phát triển toán học

Ở giai đoạn phát triển sớm toán học hai phương hướng thấy đặc biệt rõ nét Bởi lúc phương hướng tác động lẫn tương đối yếu nên chí cịn nói hai ngành toán học tách biệt - toán học ứng dụng toán học lý thuyết (thuần túy)

Ví dụ tốn học cổ Ai Cập cơng nhiên tốn học ứng dụng, liên hệ trực tiếp với việc đo đạc ruộng đất, tính tốn thể tích bình, tính đếm thực tiễn, tính thời gian (nói riêng để dự đoán nhật thực nguyệt thực), Toán học Mehico cổ đại số dân tộc khác có tính chất Tốn học túy có lẽ phát sinh lần Cổ Hy Lạp liên hệ với khoa học ngụy biện tách hẳng khỏi tốn học ứng dụng.3Chính khoa học Cổ Hy Lạp đề phương pháp suy diễn việc xây dựng lý thuyết Theo phương pháp khẳng định lĩnh vực hay khác phương pháp khác logic hình thức mà suy từ số khẳng định không chứng minh gọi tiên đề (chẳng hạn xem [30]) Kể từ phương pháp trình bày gọi nét tiêu biểu quan trọng tốn học (nếu khơng phải nét nhất) Tính chặt chẽ phương pháp suy diễn gây số ấn tượng mạnh mẽ cho hệ sau có ý đồ (nói thêm khơng thành) gắn hình thái suy diễn chặt chẽ cho lĩnh vực tri thức khác Ý đồ chí có triết học (“Đạo đức học” B.Spinoza)

Xin lưu ý tình tiết đáng ý từ khoa học cổ Hy Lạp tiếp cận khái niệm vơ hạn, tình tiết sau xóa bỏ lại tái sinh mức cao cơng trình logic tốn học kỷ XX Khoa học cổ Hy Lạp khơng thừa nhận tính vơ hạn thực khơng thể tìm thấy phát biểu tốn học thời điều mà gọi tập hợp vơ hạn q trình vơ hạn Một ví dụ tiêu biểu mệnh đề mà phát biểu là: “Tập hợp số nguyên tố vơ hạn” Euclide phát biểu là: “Nếu cho tập hợp nào (hiểu ngầm hữu hạn) số ngun tố cịn số nguyên tố nữa” Ở có

3

(41)

thể thấy có tương tự trực tiếp với khái niệm tính tiếp tục không hạn chế khái niệm mà phương hướng đại logic toán thừa nhận thay khái niệm tính vơ hạn thực

Mọi người biết không thừa nhận tính vơ hạn thực kéo theo khó khắn định mặt logic (về điểm ta nhớ lại ngụy biện Zenon) mà người Hy Lạp nói chung thấy rõ họ nhận xét khơng gian thời gian phân chia vơ hạn mặt thực khơng thể phân chia vô hạn Dưới cịn thấy thừa nhận vơ hạn thực cịn kéo theo khó khăn khơng lớn mặt logic mà cịn khó khăn thực tiễn

Những biểu cao tính chặt chẽ toán học cổ Hy Lạp lý thuyết tỷ lệ phương pháp vét cạn Evdocs tương tự lý thuyết tương tự lý thuyết đại số thực phương pháp chuyển qua giới hạn khác chỗ người Hy Lạp khơng thấy nói đến tập hợp vơ hạn q trình vơ hạn.4

Tuy nhiên với kiệt tác tính chặt chẽ, logic tốn học Hy Lạp cịn có lỗ hỗng mà theo quan điểm rõ ràng Ví dụ định nghĩa ban đầu khái niệm điểm, đường thẳng, thực chất định nghĩa (“điểm khơng có phận, ” ) sau không nhắc tới Các tiền đề bao hàm mối tương quan đại lượng tất mối quan hệ dùng đến Hồn tồn khơng có định nghĩa tiên đề liên quan tới khái niệm thứ tự điểm đường thẳng đường tròn, tức khái niệm dường thuộc số từ (giống “cho trước”, “người ta cho” , ) ngầm hiểu xây dựng lý thuyết Ngoài điều đáng ý người Hy Lạp tính tốn chiều dài, diện tích, thể tích đường thẳng, hình, vật thể khác phức tạp vấn đề thân tồn độ đo lại khơng đặt ra,

Trong người Hy Lạp (Archimede nói riêng) dùng phép chứng minh dựa tương tự cách máy móc, song phép chứng minh bị coi không chặt chẽ, có tính chất chuẩn bị, khẳng định thu nhận thiết sau phải chứng minh cách chặt chẽ (chúng cịn quay lại khái niệm tính chặt chẽ tốn học)

Có lẽ tách biệt rõ nét toán học túy toán học ứng dụng nét tiêu biểu cho nước đạo Hồi thời Trung cổ Ở lý thuyết thực tiễn giải phương trình đại số giải tích tổ hợp ngày ăn nhập sâu vào toán học túy, nói riêng, phát minh tốn học lớn hồi hệ số nhị thức, cơng thức giải phương trình bậc3và bậc4thì hồn tồn thuộc tốn học túy

4

(42)

Phục hưng khoa học

Với giai đoạn đầu thời kỳ Phục hưng khoa học, tình hình thay đổi bản, từ cơng trình G.Galilei, I.Kepler nhà khoa học khác Đối với họ, toán học phương tiện tư toán học trở thành vũ khí khoa học tự nhiên Áp lực mạnh mẽ khoa học tự nhiên có tác dụng tốt phát triển toán học Trong kỷ XVI - XVIII hai phương hướng ứng dụng lý thuyết liên tục tác động lẫn Một khung cảnh điển hình đời phát triển khái niệm toán học phục thuộc vào tốn khoa học tự nhiên hay hình học (lúc ứng dụng vào hình học phân biệt với ứng dụng vào học hay quang học), sau khái niệm xây dựng chẳng có sống độc lập tiếp tục phát triển theo quy luật nội toán học Một số kết phát triển “thuần túy” lại áp dụng vào khoa học tự nhiên điều lại làm xuất khái niệm tốn tốn học Ví dụ rõ ràng người biết phát triển việc hình thành phép tính vi phân tích phân

Trong thời “hồng kim” phát triển tốn học cách hài hịa tách biệt nữa, đối lập toán học túy toán học ứng dụng trở nên nghĩa Có điều nhà khoa học lớn thời kỳ N.Newton, L.Euler, J.Lagrange người khác khơng nhà tốn học mà cịn nhà vật lý, nhà học Trong công trình nhà khoa học phát triển phương hướng lý thuyết lẫn phương hướng ứng dụng toán học

Thời kỳ thống trị phương hướng lý thuyết tập hợp

Thời kỳ độ sang giai đoạn tiếp sau kéo dài nhiều thập niên quy ước cho thời gian bắt đầu thời kỳ kỷ XIX Nó liên hệ với hàng loạt cơng trình xuất sắc lý thuyết tập hợp (G.Cantor) lý thuyết hàm (K.Weierstrass), xây dựng cấu trúc đại số trừ tượng phân tích tiên đề hình học Các cơng trình tiếng tiến sâu sắc thời kỳ biến phận đáng kể toán học thành khoa học thống có yêu cầu thống định nghĩa, khẳng định phép chứng minh có tiêu chuẩn thống tính chặt chẽ

Nguyên nhân chủ yếu giai đoạn độ cuối thời kỳ trước tốn học tích lũy nhiều kiện, đời hàng loạt lý thuyết không thống với khơng có sở lập luận vững điều phát triển lý thuyết cách tin tưởng Sự phát triển tiếp sau mơn giải tích tự phát đời phương pháp dựa việc áp dụng vô hạn thực - chuỗi vô hạn, đại lượng vơ bé - địi hỏi phải xác định rõ ràng khái niệm hàm số việc chuyển qua giới hạn, điều tất nhiên kéo theo đời lý thuyết số thực lý thuyết tập hợp số, Quan điểm lý thuyết tập hợp cho phép phát biểu rõ nét khái niệm nhóm, mơt khái niệm quan trọng đại số học dẫn tới kết cấu logic hình học thỏa mãn người đương thời.5

5

(43)

Như bước chuyển tiếp toán học sang quan điểm lý thuyết tập hợp dựa sở, nói cách khoan dung lý thuyết tập hợp ngây thơ6là cần thiết tiến Trong việc xếp thứ tự sở khoa học, khả to lớn khám phá qua việc đó, nói riêng khả mạnh mẽ tương tác môn khoa học khác nhau7đã thực nâng cao vai trò phương hướng lý thuyết toán học mà giai đoạn (kéo dài tới chiến tranh giới thứ hai) chiếm ưu định phong cách tồn tốn học nói chung

Cịn phương hướng ứng dụng tiếp tục phát triển trước hết mối liên hệ với phát triển vật lý học học thiên thể song có lẽ khơng có bước ngoặt Thời kỳ mở đường ứng dụng mới, chẳng hạn đại số vector giải tích, đại số tenxơ giải tích sau cịn có phép tính tốn tử, lý thuyết hàm suy rộng, thân tính chất ứng dụng thời gian nguyên tắc cũ Bộ máy toán học cổ điển kết hợp với quan niệm sâu xa mặt vật lý dẫn tới chỗ thực nhiều khám phá tiếng, người ta viết sách phổ biến “đầu nhọn ngịi bút” ví dụ loại người biết đến rộng rãi tiên đốn sóng điện từ C.Maxwell, việc khám phá hành tin Neptun Pluton, tiên đoán P.Dirac Positron, Trên sở đời lĩnh vực quan trọng khoa học ngày vật lý lý thuyết

Những thành tựu phương hướng lý thuyết, việc xây dựng mức chặt chẽ thống cho tồn tốn học dẫn tới phương hướng giải toán toán học đời ứng dụng mức chặt chẽ phương hướng lý thuyết Biểu rõ khuynh hướng thấy D.Holbert A.M.Liapunov mà chúng tơi nói chi tiết sau Trong số trường hợp, khuynh hướng tỏ thực ngồi điều dẫn tới tính đối ngẫu giải tốn ứng dụng nói chung việc đặt tốn biểu thị phép giải tiến hành mức chặt chẽ vật lý (những cố gắng việc tiên đề hóa chương riêng lẻ vật lý sở lý thuyết tập hợp dường không thành công tránh khỏi mực chặt chẽ vật lý) phép giải toán học lại thực mức chặt chẽ tốn học (sự phân biệt đặc trưng cho thời kỳ thời kỳ “hồng kim” mức chặt chẽ gần trùng nhau) Trong trường hợp phức tạp nhà vật lý giải tốn tốn học ứng dụng phép giải thường có luận vật lý, song nhà tốn học coi phép giải khơng đầy đủ tìm cách thay phép giải hồn tồn đạt mức chặt chẽ “Weierstrass” Thế có phân đơi “nghề nghiệp” yêu cầu mức chặt chẽ phép giải toán ứng dụng nhà tốn học nhà ứng dụng

Góp phần vào việc phân đơi cịn có mặt phép tính tốn mà người biết không thực đầy đủ mức chặt chẽ “Weierstrass” Song rút khỏi ứng dụng truyền thống Euler nhạc trưởng khác thời kỳ “hoàng kim” nhà toán học phương hướng lý thuyết tập hợp ngừng việc tính tốn hoạt động đặt nhà thiên văn học, nhà pháo binh, cho nhóm khơng lớn

6

Nó đối lập với quan điểm đại dựa hoàn toàn vào phương pháp logic toán

7

(44)

các chuyên gia tính tốn người coi đứng chỗ nhà khoa học kỹ sư Những thành tựu mà đại phận nhà toán học đạt lĩnh vực không coi trọng trường hợp chúng bị coi hồn tồn khơng so sánh với thành tựu đáng ngạc nhiên phương hướng

Hoàn toàn gần đây, giảng nhà đại số học tiếng có nhấn mạnh cách hệ thống đến khác chủ yếu khoa học “tính toán”8

Xin lưu ý sau, toán học tính tốn trở thành mốt lại diễn phân tầng tiếp sau: Theo cách diễn tả hóm hỉnh R.S.Guter [1, tr.13] “những người làm việc lĩnh vực tốn học tính tốn chia thành người chứng minh tính hội tụ q trình tính tốn tồn nghiệm người áp dụng q trình tính tốn tìm lời giải”. Chính người sau trực tiếp phục vụ hữu ích cho khoa học ứng dụng

Còn mức chặt chẽ đạt dựa sở lý thuyết tập hợp ngây thơ giải tích tốn dựa lý thuyết giới hạn mô tả gọi ngôn ngữ -9 dường thực tiễn thu hút tất nhà toán học Đúng có người “khơng hài lịng” (ví dụ L.Brauer, G.Weyl, ) hoạt động thực tiễn họ sử dụng mức chặt chẽ Ở chỗ chỗ khác nơi chân trời có thấy mâu thuẫn lý thuyết tập hợp đại phận nhà tốn học thấy chuyện buồn cười khơng đáng khó chịu đụng đến ngáo ộp không cần đến tập hợp tất tập hợp, tập hợp tất tập hợp không chứa nó, Cuộc thảo luận tiền đề chọn (tiên đề Zermelo) kết thúc với thỏa thuận để tránh nó, có thể, trường hợp ngược lại phải rõ ràng việc áp dụng nó, tất nhiên nghĩ cẩn thận thỏa thuận có ngây thơ dường dẫn có làm thay đổi Trong năm gần cơng trình tốn học túy, tiên đề chọn có sử dụng q rộng rãi, chí mệnh đề chứng minh tiên đề ứng dụng cách có hệ thống, điều khiến người ta có ấn tượng rõ ràng có tùy tiện tiên đề

Những thành tựu phương hướng lý thuyết tập hợp, ngơn ngữ phương pháp trở thành quen thuộc vài hệ nhà toán học tạo quan niệm mức chặt chẽ đạt dường tuyệt đối Nhiều nhà toán học (và người khơng phải tốn học) có thói quen tin tưởng vào tuyệt đối đưa vào toán học mệnh đề chứng minh “chặt chẽ tuyệt đối”, toán đặt cách “chặt chẽ tuyệt đối”,

8

Cuốn sách hữu ích C.Lanczos [33] bắt đầu lời sau: “Trong nhiều năm tác giả nghiên cứu lĩnh vực giải tích tốn lĩnh vực trước hết quan tâm kỹ sư nhà vật lý học. Việc lĩnh vực toán học làm việc suốt kỷ XIX khơng ý cơng trình cổ điển của giải tích có lẽ kết quan niệm sai lầm mặt lịch sử Cho tới thời Gauss Legendre, phương pháp làm việc giải tích thu hút ý nhà toán học tiếng Phát minh lý thuyết giới hạn làm thay đổi tình hình Kể từ thời gian việc đưa q trình xấp xỉ vơ hạn để xác lập tính đúng đắn kết giải tích xác định độc lập với việc q trình áp dụng có thực tốn hay không, coi thỏa đáng Kết phân cách toán học túy toán học ứng dụng và như ta có nhà giải tích túy phát triển tư tưởng họ giới giới kết cấu túy lý thuyết nhà giải tích số người chuyển q trình giải tích thành phép tính tốn máy.

9

(45)

Quan điểm đại

Trong lời giới thiệu có nói nét “đặc thù toán học” giai đoạn nay, giai đoạn có liên hệ với mở rộng cách mạnh mẽ ngoại vi khối lượng ứng dụng toán học mặt khác với xuất phát triển kỹ thuật tính tốn điện tử làm tăng nhiều lần tính hiệu lực ứng dụng Rõ ràng bên tốn học năm gần có thành tựu xuất sắc, sau nói tới số thành tựu Song xét phát triển khoa học chẳng hạn 30 năm lại theo lập trường văn minh nói chung thành tựu “bên ngồi” tốn học thấy lớn nhiều Nói cách khác, phát triển phương hướng ứng dụng toán học (xét tất biểu nó) trở nên trội Rõ ràng thập niên tới đây, ưu bảo đảm chí cịn đẩy mạnh

Điều cho phép nói sau giai đoạn phát triển toán học mà quy ước gọi giai đoạn tiền hy Lạp, giai đoạn Hy Lạp, phục hưng giai đoạn lý thuyết tập hợp, ta bước sang giai đoạn chất việc “tốn học hố tổng qt” Có thể quy ước đánh dấu giai đoạn đầu thời kỳ từ năm 40 kỷ XX (thời gian xây dựng máy tính điện tử đầu tiên) Bước chuyển chất tốn học kéo theo biến đổi chất nhiều lĩnh vực khoa học tự nhiên, kỹ thuật, khoa học đời sống xã hội

Nếu dùng thuật ngữ lịch sử phát triển xã hội gọi giai đoạn giai đoạn “lưỡng quyền” (trong thời kỳ độ sang giai đoạn dân chủ)

Một mặt toán học phương pháp lý thuyết tập hợp tiếp tục phát triển Sự kết hợp, phát triển hình thành quan niệm đại số học, tôpô đại cương tơpơ đại số, giải tích cổ điển giải tích hàm, lý thuyết hàm hình học cho phép xây dựng chương mới, chín muồi toán học, cho phép tiến hành khái quát suy rộng tìm phương pháp cho toán cũ giải nhiều tốn đó, cho phép nêu lên lớp toán đáng ý Một kiện bật việc xây dựng giáo trình “Cơ sở tốn học” gồm nhiều tập N Bourbaki, mục đích giáo trình xây dựng sở tổng quát, nguồn tư liệu ngôn ngữ thống để tiếp tục phát triển tốn học theo lý thuyết tập hợp.10N Bourbaki chưa đề cập đến nhiều chương toán học (nhưng thật vĩ đại!) chương khơng tránh “Bourbaki hố” Hiện khuynh hướng toán học tuý vang lên điệu lất át điệu khác

Tuy nhiên thập niên gần người ta thấy mặt yếu khuynh hướng này.11Điều biểu thị theo hai tuyến – logic tốn ứng dụng Tuy nhiên , thấy, chúng lại có liên hệ với Trước hết, tính chặt chẽ “tuyệt đối” lý thuyết tập hợp ngây thơ lý thuyết lấy làm sở dường khơng hồn tồn tuyệt đối điều thấy từ luận chung khơng thể khơng cần đến nhà logic học Không phải người nghĩ lý thuyết tập hợp ngây thơ khơng có hệ thống hợp lý tiên

10

Chi tiết xem [34]

11

(46)

đề làm sở dựa vào “dễ thấy” đường ngăn cấm tuỳ tiện, ví dụ phép coi tập hợp tất số tự nhiên có lực lượng cấm khơng coi tập hợp tất số thứ tự (siêu hạn) hay tập hợp tất số có lực lượng Việc phân tích nhà logic hệ thống tiên đề lý thuyết tập hợp, hệ thống đưa thứ tự đó, chứng minh số lập trường quan trọng “tự giải quyết” (ví dụ tiên đề chọn) mà thực chấp nhận khơng điều dẫn tới tồn bình đẳng “mơn tốn học lý thuyết tổng hợp” khác nhau, không tương đương với tựa mơn hình học khác Sự độc lập giả thiết continum với tiên đề khác lý thuyết tập hợp phát tròn cơng trình K Gidel P Cohen (1960) cond có hiệu lực : có nghĩa xây dựng lý thuyết chấp nhận giả thiết tiên đề bổ sung, chấp nhận khẳng định trái ngược tiên đề, từ suy vấn đề giả thiết continum “thực ra” có hay không lại vô nghĩa ! (xem vấn đề [39]) Nói riêng, phát kiến tiếng chứng minh khái niệm tồn mà lý thuyết tập hợp ngây thơ bám vào thực khơng phải tự giải

Song theo quan điểm phát triển điều cịn quan trọng mặt yếu ớt tốn học lý thuyết tập hợp ứng dụng Chúng ta không than phiền tất khẳng định thuộc ngẫu nhiên tốn ứng dụng chuyển hồn tồn sang ngơn ngữ tốn học Điều tồi nhiều trường hợp, điều khẳng định nghiệm có tính chất tuý tồn tại, tức chứng minh định lý tồn nghiệm định lý khơng nói đến việc nghiệm tìm thấy cách xác hay xấp xỉ

Mặc dầu kết cấu nghiệm suy từ phép chứng minh định lý điều thường gây thất bại, cho nên, nói vui chút người ứng dụng giống vào hoàn cảnh người đàn ông đăng ký kết hôn với người phụ nữ mà chưa trông thấy nàng đâu Theo cách nói H Weyl “Tốn học là kho tàng khổng lồ vị tiền giấy” [40, tr 106] Trong nhiều trường hợp, kết cấu coi kết thực q trình vơ hạn hay khác mà khơng phân tích phép xấp xỉ hữu hạn Nếu cơng thức xấp xỉ dùng để giải có kèm theo đánh giá sai số thì, trừ trường hợp giản đơn ví dụ lựa chọn đặc biệt để giảng dạy, đánh giá thường không áp dụng hữu hiệu ví dụ cụ thể thực (ấy chưa nói đại phận đánh giá lại có tính chất gần bao gồm nhân tử số chưa biết) Những tính tốn số, máy tính điện tử có phần đóng góp thực đánh giá “tuyệt đối chặt chẽ” ảnh hưởng việc làm tròn số phép tính cồng kềnh tỏ lại khó khăn đánh giá thực tế chưa làm

Những tiêu chuẩn cứng ngắc mức độ chặt chẽ toán học lý thuyết đại dẫn tới chỗ làm trì trệ không đáng mong muốn việc nghiên cứu khái niệm tốn học mà đầu khơng thoả mãn tiêu chuẩn đó, ví dụ hàm denta, entropi,

Tất điều kiện nhận xét dẫn tới tình hình đại phận nghiên cứu ứng dụng lập luận tốn học khơng mức suy diễn t mà tốn học t địi hỏi, chúng khơng thể khơng cần phải mức chúng tơi nói chi tiết vấn đề Có thể nói kết cấu suy diễn không theo kịp nhịp điệu sống nay12 trình tiếp cận đến chân lý chúng thường có hiệu suất

12

(47)

quá thấp nhà ứng dụng tự phát tìm biện pháp hữu hiệu nhiều cho lập luận toán học13mà chúng tơi nói chi tiết Những biện pháp lập luận mức chặt chẽ “vật lý”, “ứng dụng” nhà ứng dụng thuộc phương hướng khác giống Những biện pháp đặc trưng cho phong cách lập luận tốn học ứng dụng, phổ biến nhà ứng dụng phong cách suy diễn phổ biến nhà tốn học t Đó điều đề cập tới phần nói “lưỡng quyền”.14

Cần phải đặc biệt lưu ý phương hướng ứng dụng ảnh hưởng đáng kể đến tồn tốn học đại nói chung kể phương hướng lý thuyết Nói riêng điều giải thích đặc biệt ý đến vấn đề thuật toán hoá vấn đề tối ưu hoá đặc trưng cho toán học đại có vết tích biểu lĩnh vực đa dạng

Để kết luận xin đưa đặc trưng bật phát triển toán học thời kỳ phục hưng lời nói đầu sách R Courout G Robin [28] “Sau thời kỳ tích luỹ lực lượng cách chậm chạp – vào kỷ 17 với đời hình học giải tích phép tính vi tích phân – mở thời kỳ cách mạng vũ bão phát triển toán học vật lý Trong kỷ XVII XVIII, tư tưởng người Hy Lạp kết tinh theo tiên đề và sự suy diễn cách có hệ thống mờ dần ảnh hưởng hình học cổ đại vẫn tiếp tục có bơng hoa nở rộ Tư tưởng tuyệt vời logic xuất phát từ định nghĩa rõ nét tiên đề “hiển nhiên” khơng mâu thuẫn ngừng nhập cảng vào những người khai phá tri thức tốn học Ham mê say sưa chân với thơng đốn trực quan, chuyển dịch kết luận bất khả xâm phạm với lời khẳng định nửa huyền bí vơ nghĩa, tin tưởng mù qng vào lực lượng siêu nhân thủ tục hình thức, họ khám phá giới toán học vơ phong phú Nhưng một, trạng thái nhập định tư tưởng bị ham mê thành tựu làm chóng mặt nhường chỗ cho tinh thần nhẫn nại phê phán Trong kỷ XIX, ý thức cần thiết phải củng cố khoa học, mối liên hệ với yêu cầu giáo dục cao đẳng phổ biến rộng sau cách mạng Pháp dẫn đến việc xét lại sở mơn tốn học mới, nói riêng, ý hướng phép tính vi tích phân tích phân nhằm làm sáng tỏ khái niệm giới hạn ngầm hiểu giải tích Như kỷ XIX thời đại thành tựu mà thời đại chứng kiến hồn lại tính xác độ chặt chẽ phép chứng minh lý tưởng

học mà ngưỡng mộ độ xác khơng cho phép đẩy mạnh lên” L.J.Mandeishtam nói [42, tr 51] Phụ hoạ thêm lời phát biểu Kapys.P.L [40, tr 30] : “Tư logic sắc bén, thuộc tính của các nhà toán học, tiền đề hoá sở gây nên phiền tối làm te liệt óc tưởng tượng

13

Và chí có ngơn ngữ riêng, thuật ngữ “Sự hội tụ thực tiễn”, “chất lượng phương pháp tính”, “chất lượng mơ hình tốn học”,

14

V.V.Novozhilov [44] viết : Hiện ngày thấy rõ khuynh hướng phân cơng lao động: Có nhà bác học hồn thiện tốn học theo hướng logic phát triển nội nó, có nhà khoa học khác lại nghiên cứu phương pháp toán học áp dụng cách mau chóng Những đại biểu hai phương hướng rõ ràng khác nhóm vấn đề “của họ” mà cịn tư chất tư

(48)

cổ điển Về mặt hình ảnh Hy Lạp có trội hẳn lên Lại lần lắc lại đưa phía tính tuyệt vời logic tính trừu tượng Có lẽ chưa bước khỏi thời kỳ phép hy vọng gián đoạn đáng tiếc toán học tuý ứng dụng sinh động thay kỷ nguyên hợp chặt chẽ Phần dự trữ có lực lượng bên với giản lược khác đạt sở quan niệm rõ ràng cho phép ngày nghiên cứu lý thuyết tốn học mà khơng tách rời khỏi ứng dụng Xác lập lần mối liên hệ hữu tri thức tuý tri thức ứng dụng, cân lành mạnh tính tổng quát trừu tượng tính cụ thể sinh động – điều chúng tơi thấy nhiệm vụ tốn học tương lai khơng xa”.15

Trong tốn học gồm có gì?

Tốn học ứng dụng gì? nói chung có tồn hay không? Những vấn đề đến lúc cần phải thảo luận cách triệt để Đáng ý thuật ngữ “toán học ứng dụng” trở thành thuật ngữ mốt (nhất người khơng phải chun gia)

Có lẽ quan điểm phổ biến khái niệm “toán học ứng dung” hàng ngũ nhà toán học quan điểm cho nói chung khơng có tốn học ứng dụng Ngồi ra, nhà tốn học khác lại cho từ nội dung hồn tồn khác tuỳ theo nhà tốn học có gia nhập vào thân mơn tốn học hay khơng.16

Có người cho kết cấu tuý suy diễn gọi toán học Tất nằm ngồi kết cấu đó, khơng có quan hệ với tốn học mơn tốn học khơng gọi tốn học.17Hiện quan điểm phát biểu ầm ĩ song cách “khơng thức” phổ biến, bên cạnh việc khác, quan điểm tỏ “thuận tiện” cho nhiều người dạy tốn với người khơng phải nhà tốn học

Thực tế quan điểm thu hẹp cách vô lý đáng kể ranh giới Khoa học Toán học vĩ đại trước hết mang lại bất lợi cho mơn Tốn học (và tất nhiên cho nghiệp đào tạo nhà bác học trẻ) Về vấn đề M Las S Ulam viết : “Những cố gắng – tiếc rằng lại phổ biến – nhằm tách rời toán học “thuần tuý” khỏi toàn phần hoạt động khoa học cịn lại nhằm chắt lại để nấu thành loại nước cốt đặc biệt làm nghèo nàn cả tốn học lẫn mơn khoa học khác” [13, tr 234].

F Klein phát biểu tư tưởng : “Những quan niệm logic tuý phải tạo ra, người ta nói, xương rắn thể toán học, đảm bảo cho độ ổn định

15

A.N.Kolmorokov có nói ước mơ mà ơng theo đuổi từ lâu toán phân cách phương pháp “chặt chẽ” nhà toán học tuý biện pháp “không chặt chẽ” lập luận toán học nhà toán học ứng dụng, nhà vật lý kỹ thuật (dẫn theo [51])

16

Khi liệt kê quan điểm phần nói riêng dùng ý kiến nói miệng M.A.Krasnoselski

17

(49)

và đáng tin cậy Nhưng thân sống toán học, quy nạp quan trọng tính tích cực lại chủ yếu liên hệ với ứng dụng nó, tức với tương quan giữa những khách thể trừu tượng với toàn lĩnh vực khác Loại trừ ứng dụng khỏi tốn học chẳng khác tìm thực thể sống từ hài cốt, không bắp thịt, không thần kinh, không mạch máu” [29, tr.46].

Cuối xin dẫn lời A Poincaré: “Vật lý học không nhũng cho (các nhà toán học – tác giả) lý để giải vấn đề, cịn giúp tìm thấy phương tiện để giải vấn đề Điều xảy theo hai đường Một cho ta linh cảm của phép giải, hai gợi ý cho ta tiến trình lập luận” [54, tr.108].

Thực chất biểu thị quan điểm thứ hai mà cho dễ dàng chấp nhận hơn, quan điểm cho gia nhập vào lĩnh vực hoạt động tốn học cịn có phương pháp giải thực tiễn toán bắt nguồn từ bên ngồi tốn học

Song cịn phấn khởi với quan điểm thứ ba, rộng nhất, cho tốn học khơng bao hàm lĩnh vực suy diễn mà cịn bao hàm tồn thực chất toán học – khách thể toán học, phương pháp tư tưởng gặp toán học lý thuyết ứng dụng : tức kết cấu mơ hình tốn học, thực nghiệm toán học, lập luận quy nạp hay lập luận hợp lý khác có tính chất tốn học,

Trong sách hay G Polya [56, tr.309] có nói : “Giới hạn tốn học miền những lập luận chứng minh thuộc khoa học đạt mức phát triển khái niệm thuộc khoa học biểu thị dạng logic tốn trừu tượng” Chúng tơi muốn thêm vào khái niệm chứng minh khơng nên gắn cho nội dung giáo điều, hẹp hịi

Tất nhiên mơn sinh quan niệm này, quan niệm mà cho tiến chín muồi tốn học (và điều nhà tốn học) phải khước từ “tính thống lý thuyết tập hợp” toán học mà coi “hạt nhân” tốn học mà thơi

Các quan điểm tốn học ứng dụng

Trước hết lấy làm buồn rầu mà lưu ý theo ý kiến số nhà tốn học nghiên cứu ứng dụng nói chung điều hổ thẹn

Về vấn đề F Klein viết : “Rất tiếc cịn gặp giáo viên đại học khơng che dấu lời lẽ ngạo mạn người làm ứng dụng Cần phải đấu tranh quyết liệt với tính kiêu căng thể quan điểm Mọi thành tựu xác đáng, cho dù thuộc lĩnh vực lý thuyết hay ứng dụng, phải đánh giá cao vì nó tạo cho người khả nghiên cứu vật mà cảm thấy gần gũi Khi đó mỗi người biểu đa dạng số lớn tài có : thiên tài vĩ đại như Aschimede, Newton, Gauss người luôn chiếm lĩnh lý thuyết lẫn thực tiễn như nhau” [57, tr.314].

(50)

của mình, người phục vụ cho họ Sự “phân đẳng cấp” trường hợp tốt nhất cũng triệu chứng óc hẹp hịi” [12, tr.27] N.Bailey nói bất lợi đua địi sách hữu ích ơng [58, tr.138] nói ứng dụng tốn học vào sinh vật y học

V.V Novozbilov viết : “Tiếc nhà lý thuyết thường coi “nhà ứng dụng” một nhà tốn học loại hai, nhà khoa học khơng có khả làm việc cách thật chặt chẽ và lấy riêng làm tổn thất chung Dễ dàng phát “nhà ứng dụng” sai sót trong lập luận xét tính chặt chẽ, nhà lý thuyết thường tỏ thờ với thành tựu họ -đó việc họ biết cách giải tốn thực với số xác đủ dùng cho mục đích thực tiễn mà thân nhà lý thuyết giải phương pháp chặt chẽ”. [44]

Qua câu trích dẫn ta thấy bật khía cạnh tâm lý vấn đề Nhưng độc lập với điều tưởng cần phải nhấn mạnh người ta ngày thừa nhận tồn khách quan toán học ứng dụng Song đằng sau thừa nhận cịn nhiều quan điểm khác

Ví dụ số người cho toán học ứng dụng phận toán học “thường dùng ngày” với nghĩa xấu từ đó, tồn dạng số biện pháp, đơn thuốc quy tắc thiếu chặt chẽ khơng hồn thiện mặt logic (có thể trình độ tốn học cịn thấp chuyên gia lĩnh vực này) Những thiếu sót tốn học ứng dụng khắc phục kết “toán học khuyết” nâng lên mức tốn học tiêu chuẩn.18

Chúng cho quan điểm ngây thơ phổ biến này, khơng phải biểu đua địi dựa sở khơng nhận thức tình hình đắn vấn đề Thực theo quan điểm làm giải thích việc nhà vật lý, kỹ sư nghiên cứu lý thuyết chuyên gia khác rõ ràng có khơng người thơng minh áp dụng tốn học cố tình né tránh ngôn ngữ suy diễn chặt chẽ? Và học viện họ học ngôn ngữ cách có hệ thống song họ muốn (liệu có hại chăng?) học lại chuyển sang ngơn ngữ tốn học ứng dụng xây dựng lại toàn phong cách tư suy diễn t địi hỏi thiết phải có Chúng tơi nghĩ xây dựng lại hồn tồn tự nhiên điều giải thích cho là: Sự xây dựng lại cần thiết Dưới cố gắng chứng minh vắng mặt yêu cầu thiết phải có hồn thiện logic hình thức ứng dụng tốn học điều khơng thể tránh được, dấu hiệu thua mà nguồn sức mạnh đặc biệt toán học ứng dụng

Một quan niệm khác đồng tốn học ứng dụng với tốn học tính tốn tốn học máy tính Đây quan điểm hẹp hịi có xu hướng phiến diện

Bây xin đề cập đến quan điểm phát biểu báo [2] Xuất phát điểm cho phép giải toán học toán ứng dụng có

18

(51)

những nét đặc thù Ở đây, trước hết mặt nguyên tắc khơng thể tiến hành phép chứng minh mức cơng trình nghiên cứu tốn học t mơ hình tốn học khách thể thực mơ tả nét theo nghĩa hay khác khách thể khơng có ý định khơng thể có ý định mơ tả cách đầy đủ Mặt khác việc giải toán thực tiễn trước hết có u cầu mà nghiên cứu toán học tuý coi yêu cầu thứ cấp: Bởi tốn ứng dụng phải giải khơng mà phải kịp thời, tiết kiệm sức lực, nghiệm phải chấp nhận phương tiện tính tốn có phải thuận tiện sử dụng thực tiễn phải có độ xác thích hợp với tốn, 19

Chúng tơi quy định gọi việc thực tốt tất yêu cầu mà đơi cịn mâu thuẫn tính tối ưu nghiệm (đối với ứng dụng) dù phải lưu ý trước giai đoạn phát triển khoa học khó mà chức mục đích Xuất phát từ điều đề nghị định nghĩa là: Toán học ứng dụng khoa học phương pháp giải tối ưu, mà thực tiễn chấp nhận được, toán học nảy sinh từ bên ngồi tốn học Như vậy, tốn học ứng dụng toán học bị gián tiếp thực tiễn, phận khoa học hợp thành tựa sinh hoá hay nhiệt kỹ thuật Sự phát triển môn xác định mở rộng nhóm ứng dụng thay đổi nội dung cụ thể khái niệm tính tối ưu phép giải tốn, nói riêng, nội dung hồn tồn thay đổi ảnh hưởng phương tiện tính tốn Tất nhiên tìm thấy nghiệm tối ưu điều khơng có nghĩa ta loại bỏ nghiệm đáp ứng gần yêu cầu tính tối ưu Phần lớn nghiệm thực mà dùng nghiệm mà thời gian đó, mức độ thoả mãn yêu cầu

Về vấn đề ta nhớ đến câu cách ngơn tiếng : “Tốn học t làm có thể cần cịn tốn ứng dụng làm cần có thể”.20Câu cách ngơn truyền xu hướng nói chung đúng, từ “cần” dùng theo nghĩa khác Chỉ để ý đến ý nghĩa thứ hai, cố gắng chứng minh toán học ứng dụng làm cần cần làm

Cũng đáng ý đến quan điểm L.V Ovsjannikov phát biểu lời: “Toán học ứng dụng khoa học mơ hình tốn học, chi tiết hơn, nói rằng, khoa học kết cấu, nghiên cứu, diễn tả tối ưu hố mơ hình tốn học Định nghĩa nhằm vào đối tượng của khoa học theo chúng tơi khơng mâu thuẫn với định nghĩa định nghĩa nặng về tính chất chức Như vậy, muốn so sánh tương tự - nói chung xa – giữa tốn học ngơn ngữ tốn học t tốn học ứng dụng làm người ta lần lượt nhớ đến văn phạm ngữ nghĩa”.

Bàn vấn đề tốn học ứng dụng có tạo thành khoa học độc lập khơng việc làm có chút kinh nghiệm tính nhiều nghĩa cách nói “khoa học độc lập” đắn

19

Với tinh thần này, I.Babuska, E.Vitasek M.Phager viết sách hay [59, tr.9] : “ ngày nay, toán coi giải có phương pháp hiệu lực cho kết cần thiết với độ chính xác dủ dùng khoảng thời gian định” Trong sách N.S.Bakhvalov [60, tr.14] viết với quan niệm sâu sắc tình ứng dụng thực có nói rằng: “Tìm thấy nghiệm thoả mãn bài toán mà kịp thời tốt có nghiệm đầy đủ tốn vào lúc ý nghĩa

20

(52)

khơng nên nói khoa học mà khía cạnh tốn học đời ứng dụng nó, có thể, nên nói kết phép “chiếu” toán học cách độc đáo lên văn minh, điều quan trọng với phép chiếu tốn học có nét chất phép chiếu ấy, nét định nghĩa cho tốn học ứng dụng

Do chúng tơi dùng từ tốn học ứng dụng coi thuật ngữ làm việc xác định quan điểm cuối nêu dành vấn đề tính độc lập tồn tốn học ứng dụng với tính cách khoa học cho nhà triết học 21 Để phân biệt với điều đó, nói tốn học t, chúng tơi trước hết quan niệm tốn học thống từ Weierstrass đến Bourbaki dựa sở lý thuyết tập hợp ngây thơ.22

Chúng chủ yếu ý đến vấn đề cụ thể : nét đặc trưng nảy sinh ứng dụng toán học, đặc điểm phương pháp lập luận tốn học ứng dụng nói riêng lập luận thừa nhận chứng minh tốn học ứng dụng đó, Việc thảo luận vấn đề hữu ích, chí sơi sơi cơng trình hồn tồn có tính chất cụ thể

Để kết luận, đưa lời nói sáng sủa R Courant nói khác phương pháp tiếp cận vấn đề toán học tuý toán học ứng dụng dùng làm phần giới thiệu độc đáo cho phần trình bày tiếp sau

“Cùng vấn đề tốn học giải khác nhau, người theo quan niệm toán học chặt chẽ (và khuynh hướng đơi cịn thấy người nghiên tư khoa học) địi hỏi hồn thiện khơng nhân nhượng Anh ta khơng cho phép có lỗ hỗng logic tư cách giải toán đặt ra, kết đạt theo ý phải đỉnh cao mắt xích liên tục lập luận toàn thiện Và đối phương quan điểm mà gặp khó khăn dường khơng khắc phục mau chóng tìm cách phát biểu lại tốn thạm chí đặt khác loại với tốn cũ, khắc phục khó khăn (“cái cần” – tác giả) Cịn có đường vịng khác nữa, xác định lại xem coi “nghiệm tốn” Trong thực tế, cách làm bước sơ chấp nhận để tới nghiệm chân tốn ban đầu

Trong cơng trình nghiên cứu có tính chất ứng dụng thứ khác Trước hết khơng thể dễ dàng làm thay đổi lảng tránh toán đặt Ở đòi hỏi khác đưa câu trả lời đắn đáng tin cậy theo quan điểm chung người ta Trong trường hợp cần thiết, nhà tốn học có nhân nhượng: Anh ta phải sẵn sàng đưa dự đốn vào

21

Về điểm cịn có khó khăn phụ khái niệm “nghiên cứu ứng dụng”, “chương ứng dụng”, tương đối, điều đơi dẫn đến hiểu lầm khác Ví dụ dẫn đến chỗ có người không gọi nhà ứng dụng (và tương ứng nhà lý thuyết) Có nhiều cơng trình nghiên cứu sách gọi ứng dụng (nếu chúng xem xét theo lập trường trừu tượng hơn) gọi toán học tuý (giả sử theo lập trường kỹ sư) Tất nhiên tính tương đối khái niệm “ứng dụng” có vật lý, học môn khoa học khác Về khái niệm triển vọng phát triển toán học ứng dụng xem thêm [61-72]

22

(53)

xích lập luận cho phép sai số định giá trị số Những tốn chủ yếu theo phương hướng thực tiễn, ví dụ tốn dịng có sóng va chạm, địi hỏi cơng trình nghiên cứu tốn học để xác định xem tốn đặt có hay khơng Trong cơng trình nghiên cứu ứng dụng địi hỏi phép chứng minh định lý toán học tuý tồn tin tưởng có nghiệm đảm bảo cho độ tin cậy mơ hình tốn học sử dụng, (thực điều có phức tạp đơi chút sau chúng tơi nói lại vấn đề – tác giả) Và cuối cùng, chế ngự toán học ứng dụng phép xấp xỉ thiếu chúng khơng thể chuyển q trình vật lý thực thành mơ hình tốn học

(54)(55)

SỰ KHỞI ĐẦU CHO KỶ NGUYÊN KỸ THUẬT SỐ

Desmond MacHale

(Dịch giả: Hoàng Cao Phong, Hà Nội)

Đại học Cork kỷ niệm200năm ngày sinh nhà toán học logic học George Boole Từ tháng10năm2015;Đại học Cork lên kế hoạch cho loạt kiện năm dành cho George Boole1 Sức ảnh hưởng công trình Boole ngày trở nên mãnh liệt vài thập niên trở lại đây, từ khoa học máy tính cơng nghệ kỹ thuật số

Cuốn tiểu sử Boole viết Desmond MacHale vừa xuất Nhà xuất Đại học Cork Đây chỉnh sửa phiên năm1985xuất Nhà xuất Boole Lời tựa phiên trước viết nhà toán – vật lý học tiếng người Ai-len - John L Synge Lần này, sách có lời tựa viết Ian Stewart Đại học Warwick Ấn biên tập Ivor Grattan-Guinness2

Đây tiểu sử tồn diện, đóng góp vơ to lớn cho hiểu biết nhà toán học logic học có tầm ảnh hưởng lớn kỉ XIX Nó khắc họa cách đầy đủ chân dung Boole, không người khoa học mà nhà cải cách xã hội, nhà tư tưởng tôn giáo người đàn ông gia đình Ơng triết gia tinh túy Cuốn sách kết nghiên cứu tìm hiểu kĩ lưỡng, cơng phu, kể lại câu chuyện sống nghiệp Boole cách có hệ thống với hút bền bỉ

1 Thuở thiếu thời

George gia đình có4con, ơng niên nhút nhát đặc biệt thông minh Từ thời niên thiếu, ơng sử dụng thành thạo tiếng Latin tiếng Hy Lạp, ngồi ơng cịn tự học tiếng Pháp, tiếng Đức tiếng Ý Điều giúp ơng sau dễ

1

http://www.georgeboole.com

(56)

dàng tiếp cận với phát triển tốn học tồn châu lục, cho phép ông tiến xa hầu hết đồng nghiệp Ơng học qua lớp tiểu học, nghèo nên tiếp tục bậc trung học; vậy, ơng phải tự học kiến thức toán cao cấp cách nghiên cứu tác phẩm nhà toán học hàng đầu thời gian

Boole bắt đầu nghiên cứu tốn học cách thực nghiệm túc ơng khoảng16tuổi Sau thành thạo giải tích, ơng nghiên cứu sang tác phẩm Newton, Lagrange, Laplace, Jacobi Poisson Như người tự học, Boole tạo phương pháp độc lập tiếp cận vấn đề nghiên cứu Cảm hứng ban đầu ông với tốn học đến từ ứng dụng nhằm giải vấn đề khoa học khác, dần dần, ông bị quyến rũ vẻ đẹp thú vị tốn học túy Ơng cơng bố số cơng trình phương trình vi phân, tích phân, logic, xác suất, hình học đại số tuyến tính

Năm 1841, sách “Lý thuyết tổng quát phép biến đổi tuyến tính” Boole đặt móng cho phát triển ngành tốn học - lý thuyết bất biến Cayley Sylvester người phát triển lý thuyết bất biến lên tầm cao mới, hai thừa nhận Boole khơi nguồn cảm hứng cho nỗ lực họ

Tài liệu “Về phương pháp tổng qt giải tích” ơng xuất vào năm1844

trong tuyển tập cơng trình triết học Hội Hồng gia Cũng nhờ vậy, ơng giành Huy chương vàng Hội, giải thưởng lĩnh vực toán học Bài báo giới thiệu phương pháp tổng quát hóa cách giải lớp lớn phương trình vi phân sai phân với hệ số thay đổi

2 Đại học Cork Queen

Tháng10năm1846;Boole ứng tuyển cho vị trí giáo sư tốn học ba trường đại học Queen: Ở Belfast, Galway Cork Đơn xin xét tuyển Boole làm hội động sửng sốt ông tuyên bố: “Tôi thành viên trường đại học chưa học tại trường đại học nào” Tuy vậy, đơn ông nhận ủng hộ mạnh mẽ từ nhà toán học hàng đầu Sau thời gian trì hỗn, ơng giao phó vị trí Đại học Cork làm việc vào tháng10năm1849với mức lương khởi điểm250bảng Anh năm

Các đại học Queen sở đa giáo phái, gây nên tranh cãi từ đầu bị hệ thống Công giáo mô tả trường đại học vô thần Trong thời gian, Boole cố gắng tránh tham gia trực tiếp vào xung đột tôn giáo, nhiên ông không bị ảnh hưởng Mặc dù chất hịa nhã, ơng dính vào loạt tranh luận gay gắt trường đại học Có lẽ nhiều thơng tin chi tiết cung cấp MacHale thực cần thiết, từ thư dài đến tờ báo trích dẫn

Trong năm đầu Cork, Boole cô đơn dường không hạnh phúc Tuy nhiên, thứ hồn tồn thay đổi vào năm1855khi ơng kết hôn với Mary Everest23tuổi lúc tuổi

(57)

3 Các định luật tư duy

Năm1833;khi mới18tuổi, Boole có ý tưởng thống qua tất quan hệ logic biểu diễn dạng cơng thức Chính ý tưởng sau trở thành cống hiến to lớn ông ngành khoa học: Giải thích cho trình suy nghĩ người ngơn ngữ tốn học xác Những nỗ lực để biến logic thành môn khoa học khởi nguồn từ Aristotle, Leibniz đặt bước đường thể mối quan hệ logic dạng ơng chưa tìm ký hiệu phù hợp Năm1847;Boole viết sách “Phân tích Logic học”, miêu tả với tựa đề “Phép tính cho lập luận hợp lý” Cuốn sách đánh dấu cho khởi đầu logic hình thức

Tác phẩm lớn Boole, “Các định luật tư tưởng”, viết suốt thời gian ông Đại học Queen Một nhận xét sâu sắc ơng tốn học khơng giới hạn số hay số lượng, mà mang ý nghĩa lớn cách biểu diễn vạn vật dạng thức phù hợp với số quy tắc định Mục tiêu ơng chuyển hóa mệnh đề logic thành dạng cơng thức từ đó, kết luận logic trở thành kết phép toán với giả định ban đầu Việc xem xét lớp thay tập trung vào số mở đường cho lý thuyết tập hợp, tâm điểm tảng toán học

Boole người dùng phép toán đại số để biểu diễn thay cho mệnh đề logic, từ tạo động lực mạnh mẽ cho lĩnh vực logic hình thức Trước đây, khơng có đánh giá cao chất tốn học xuất ngơn ngữ hàng ngày Ngành đại số học phát triển Boole trở thành công cụ lý tưởng việc xử lý thông tin hoạt động máy tính dựa theo ngun tắc Đại số Boolean bao gồm nhiều chủ đề, có lý thuyết tập hợp, số nhị phân, không gian xác suất, cấu trúc mạch điện tử công nghệ máy tính Nhiều ý tưởng Boole coi hiển nhiên tìm thấy lý thuyết tập hợp xác suất sơ cấp Nó mang lại ứng dụng rộng rãi lĩnh vực chẩn đoán y tế, bảo hiểm chứng pháp lý

4 Boole Hamilton

Boole thử làm thơ, nhiên giống với người bạn - William Rowan Hamilton, tác phẩm thơ ơng khơng có đặc sắc Cả chương dành riêng để nói thiếu liên lạc đáng ngạc nhiên hai người đàn ông Boole sinh sau Hamilton10năm qua đời năm Họ có nhiều sở thích chung tốn học lẫn niêm luật thơ, có nhiều hội để cộng tác hay tương tác Tuy nhiên, hạn chế tiếp xúc hai người cho thấy họ có số khó khăn đáng kể bất đồng quan điểm, khơng có chứng rõ ràng điều Vào năm1985;MacHale viết bí mật mối quan hệ họ chưa bật mí Chương có tiêu đề “Một số câu hỏi chưa trả lời” Tuy nhiên, điều đáng thất vọng sau hơn30năm, khơng có thêm thơng tin điều này.3

3

(58)

5 Gia đình

Chương cuối thú vị thơng tin ngồi lề George Mary Boole có5người gái, tất đặc biệt theo cách khác Alicia có khám phá quan trọng hình học4chiều Cơ đưa vào thuật ngữ “polytope” tương đương4chiều khối đa diện GI Taylor, nhà thủy khí động lực học tiên phong kỉ20;là gái Boole-Margaret Một người gái khác, Ethyl Lilian (Voynich) sống đời phiêu lưu tác giả tiểu thuyết “Ruồi Trâu” vô phổ biến Nga Câu chuyện đời cô hấp dẫn

Kết thúc chương cuối- “những năm tháng cuối cùng”, MacHale viết Boole: “Tên ông sống tựa máy tính kỹ thuật số ln phụ thuộc vào đại số Boolean để vận hành, và miễn sinh viên tốn học cịn nghiên cứu lý thuyết vành, phương trình vi phân, lý thuyết xác suất, phương trình sai phân, lý thuyết bất biến, lý thuyết toán tử, lý thuyết tập hợp và tất nhiên, SÁCH101logic” Ông kết bút ghi Boole vô vui mừng khi biết tất thơng tin liên lạc đại, cho dù liệu, văn hay hình ảnh, ln bao gồm chuỗi dài biểu tượng Boolean0và1:

6 Lời kết

Cuốn sách xuất cách kỳ cơng dường khơng có lỗi Một vấn đề nhỏ làm người đọc khó hiểu việc nhắc nhắc lại Lebesgue Nhà phân tích Henri Lebesgue chưa sinh năm1875;rất lâu sau chết Boole Tài liệu tham khảo lẽ thuộc nhà số học Victor-Am’ed’ee.1791 1875/nhưng lại liệt kê Henri phần bảng mục lục Một vấn đề nghiêm trọng việc bỏ qua phụ lục danh sách27;phần “Các tài liệu tham khảo bổ sung”, xuất phiên lại bị bỏ từ những phiên Một số trích dẫn dài kèm theo số đoạn lạc đề tối nghĩa Ví dụ việc thêm vào danh sách dài dặng dặc nhà tư tưởng tôn giáo thời đại ảnh hưởng lớn đến Boole Việc làm có nên hay khơng tất nhiên vấn đề quan điểm cảm thấy khiến cho cậu chuyện Boole bị gián đoạn

Người đọc tập san cảm thấy mức độ chi tiết tốn học khơng đầy đủ Thật sự, với độc giả tốn học có ý tưởng với cơng trình Boole muốn đào sâu mảng kiến thức không thật thấy sách lý tưởng cho mục đích đó, họ phải đọc lại ấn phẩm gốc Boole Tuy nhiên, có danh sách đầy đủ giá trị vấn đề nêu sách

Peter Lynch Peter Lynch Giáo sư Khoa học Khí tượng UCD Chun mơn ông bao gồm khí động học, dự báo thời tiết số, học Hamilton lịch sử khí tượng học Ơng viết số ấn phẩm tốn học phần “Đây Toán học” “Thời báo Ai-len” Xem blog ơng tạihttp://thatsmaths.com

Trường Khoa học Tốn học, Đại học College Dublin

(59)

Bình Nguyễn

(Trường Đại học Khoa học Tự nhiên, ĐHQG - TP.HCM)

TĨM TẮT

Tốn học khơng khơ khan nhiều người nghĩ mà gắn liền mật thiết với sống xung quanh Đã từ lâu nay, có nhiều ứng dụng thiết thực xây dựng dựa sở toán học bao gồm phép biến đổi thuật toán biến đổi Fourier, biến đổi Wavelet, thuật toán SVD (singular value decomposition), thuật toán LDA (Linear Discrimination Analysis), thuật toán phân rã trị riêng (eigen decomposition) Trong số đó, nói đến ứng dụng liên quan đến nhận dạng hình ảnh (image recognition) điện thoại thơng minh (smart phones), nơi người ta quan sát hình ảnh ngơi nhà thơng qua camera, tự động nhận tin nhắn có người lạ đột nhập vào nhà, biết thú cưng nhà có lút làm khơng Trong viết này, tác giả giới thiệu độc giả xây dựng hệ thống nhận dạng chó mèo đơn giản thơng qua thuật tốn Tốn học

1 Giới thiệu

Bài viết tập trung vào việc xây dưng hệ thống thơng minh phân biệt chó mèo Nếu nhìn mắt thường, khơng khó để thực xác việc Vấn đề đặt trường hợp có ảnh chó mèo cho trước, tìm phương pháp giúp cho máy tính nhận biết hình ảnh chó mèo Hệ thống thơng minh đạt độ xác gần giống người

Để nhận biết ảnh chó mèo từ ảnh, thông thường dựa vào đường nét bên ảnh Ảnh chó chứa số nét đặc trưng góc cạnh (edge) riêng biệt giúp phân biệt với ảnh mèo Trong phân tích thời gian - tần số, biến đổi Wavelet

(60)

Hình 7.2: Một số ảnh chó mèo tập huấn luyện

biết phương pháp hữu ích để biểu diễn thơng tin tín hiệu theo nhiều tỉ lệ khác Khơng thế, biến đổi wavelet cịn hữu ích việc dị tìm cạnh (edge detection) ảnh Chính thế, tốn này, chúng tơi sử dụng biến đổi wavelet để biểu diễn thơng tin ảnh chó mèo

Như đề cập trước đó, viết giới thiệu thuật toán đơn giản giúp máy tính phân biệt chó mèo Thuật toán bao gồm bước sau đây:

Bước 0: cho trướcM ảnh chó vàM ảnh mèo tập huấn luyện (training set) Bước 1: Phân rã (decompose) ảnh chó mèo thơng qua hàm wavelet sở

(Wavelet basis functions) Mục đích bước thực thao tác dị tìm cạnh (edge detection) ảnh

Bước 2: Từ ảnh mở rộng wavelet (wavelet expanded images) tập huấn luyện, dùng thuật tốn SVD để tìm kiếm cách thành phần (the principal components) liên quan tương ứng đến chó mèo

Bước 3: Tìm kiếm ngưỡng định để phân biệt chó mèo thuật toán LDA (liner discrimination analysis)

Bước 4: Kiểm tra tính hiệu thuật tốn N ảnh chó N ảnh mèo Thơng thường, người ta chọn tỉ lệM WN D4W1

(61)

Hình 7.3: Chó Bull

2 Xây dựng thuật tốn

Để minh hoạ cho thuật toán, xét ảnh chó Bull Hình 7.3 Trong tập huấn luyện, có tất cả98ảnh chó và98ảnh mèo tất ảnh để điều chỉnh kích cỡ kích thước64x64 Q trình tiền xử lý mô tả hàmpreprocess.mdưới

function [newImage] = preprocess(imagePath) oldImage = imread(imagePath);

newImage = imresize(oldImage,[64,64]); end

Sau đó, tất ảnh huấn luyện phân rã thành sở wavelet cách sử dụng biến đổi wavelet rời rạc Trong Matlab, đọc giả sử dụng hàmdwt2sau để thực phép biến đổi wavelet rời rạc hai chiều cho ảnh này:

[cA, cH, cV, cD] = dwt2(X,wname)

Ở đây,cAlà ma trận hệ số thành phần xấp xỉ (the approximation coefficient matrix) còncH,

cV, vàcD ma trận hệ số thành phần chi tiết (the detail coefficient matrix) tính thơng q việc phân tích ảnh đầu vàoX Tham sốwnameđại diện cho tên hàm wavelet sử dụng Trong ví dụ này, sử dụng lọc Haar để thực trình phân rã wavelet

Trong hình 7.4 hình 7.5, thấy phân rã Wavelet hình chó Sau phân rã ảnh chó mèo biểu diễn wavelet, tìm phương pháp tiếp cận để phân biệt ảnh chó mèo Để thực điều này, cách tiếp cận phân rã tập ảnh chó mèo thuật tốn SVD, nghĩa chó mèo biểu diễn thành phần (principal components) Sau đó, dùng phương pháp LDA để huấn luyện hệ thống phân biệt chó mèo

(62)

Hình 7.4: Biến đổi Wavelet ảnh chó Bull (ảnh 7.3) tạo thành bốn ma trận:cA,cH,cV

vàcD (từ trái sang phải, từ xuống dưới)

(63)

dog_folder_path=’./training/dog’; cat_folder_path=’./training/cat’;

dog_wave = dc_wavelet(dog_folder_path); cat_wave = dc_wavelet(cat_folder_path); feature=20;

[result,w,U,S,V,th] = dc_trainer(dog_wave,cat_wave,feature);

Trong thuật tốn trên, ảnh chó mèo đưa vào hệ thống sau đưa vào thuật tốn Wavelet mơ tả sau:

function [dcData] = dc_wavelet(dc_folder_path) allFiles=dir(dc_folder_path);

allNames =fallFiles.nameg; nw=32*32;

index=1;

for i = 1:length(allNames)

filename=fullfile(allNamesfig);

filename=fullfile(dc_folder_path,filename); display(filename)

I=imread(filename); J=imresize(I,[64,64]); J=rgb2gray(J);

[ ,cH,cV, ] = dwt2(J,’Haar’); nbcol = size(colormap(gray),1); cod_cH1 = wcodemat(cH,nbcol); cod_cV1 = wcodemat(cV,nbcol); cod_edge=cod_cH1+cod_cV1;

dcData(:,index)=reshape(cod_edge,nw,1); end end

Thuật toán huấn luyện mô tả hàmdc_trainer.m.

function [result,w,U,†,V,threshold]=dc_trainer(dog0,cat0,feature) nd=length(dog0(1,:));

nc=length(cat0(1,:));

[U,†,V] = svd([dog0,cat0],0); animals =†*V’;

U = U(:,1:feature);

dogs = animals(1:feature,1:nd); cats = animals(1:feature,nd+1:nd+nc); md = mean(dogs,2);

mc = mean(cats,2); Sw=0;

for i=1:nd

(64)

for i=1:nc

Sw = Sw + (cats(:,i)-mc)*(cats(:,i)-mc)’; end

Sb = (md-mc)*(md-mc)’; [V2,D] = eig(Sb,Sw);

[lambda,ind] = max(abs(diag(D))); w = V2(:,ind); w = w/norm(w,2); vdog = w’*dogs; vcat = w’*cats; result = [vdog,vcat];

if mean(vdog)>mean(vcat) w = -w;

vdog = -vdog; vcat = -vcat; end

sortdog = sort(vdog); sortcat = sort(vcat); t1 = length(sortdog); t2 = 1;

while sortdog(t1)>sortcat(t2) t1 = t1-1;

t2 = t2+1; end

threshold = (sortdog(t1)+sortcat(t2))/2; end

Có thể thấy, bước trình huấn luyện, sử dụng phân rã SVD liệu tạo từ biến đổi Wavelet cho ảnh huấn luyện Với tập huấn luyện gồm 98 ảnh mèo 98 ảnh chó, sau thực phép biến đổi Wavelet, thu véc tơ có tổng độ dài3232D1024 Kết hợp liệu huấn luyện (98C98D196ảnh), tạo ma trận có kích thước1024196 Đến bước này, sử dụng thuật toán SVD dạng rút gọn ma trận để trích thành phần chínhU,†vàV

Tuy nhiên, không thiết phải sử dụng tất thành phần (principal components) thu từ thuật tốn SVD để làm đặc trưng cho việc nhận dạng Ở đây, cần dùng số lượng vừa đủ thành phần chính, mà xác định thơng qua biếnfeature Giả sử số lượng thành phần chọn là20 Khi đó, từ ma trậnU

†Vthu được, trích ra20cột ma trậnU và20dịng ma trận†V Trong hình 7.6, độc giả thấy hình ảnh minh họa tương ứng bốn thành phần ảnh chó mèo trích từ tập huấn luyện

Một câu hỏi đặt lại sử dụng ma trận†V? Như biết,†là ma trận đường chéo mà thành phần đường chéo singular value ma trận1024196ban đầu Chính thế, việc chọn ma trận†Vsẽ tạo nên trọng số cho mode ảnh (mèo chó) cho trước chiếu lên†V

(65)

Hình 7.6: Minh hoạ cho bốn thành phần (principal components) tập ảnh chó mèo Những thành phần sử dụng làm sở để phân loại ảnh chó mèo

trước ảnh vật Thuật toán LDA 2-lớp sử dụng toán Một mục tiêu thuật toán LDA tìm phép chiếu hiệu cho liệu chiều xuống không gian cho trước phân chia tốt

Như hình 7.7, với liệu hai lớp cho trước, phép chiếu LDA tạo phân bố lý tưởng cho liệu giá trung bình liệu hai lớp hệ trục mới1và2hoàn toàn tách rời Nếu xét khía cạnh tốn học, phép chiếu thoả mãn:

w Darg max w

wTSBw

wTS Ww

; (2.1)

trong đó, ma trận phân tán lớpSB lớpSW tính sau:

SB D 1/.2 1/T (2.2)

SW D X

iD1 X

x

.x i/.x i/T (2.3)

Dễ thấy, nghiệm tốn 2.4 véc tơ riêng tương ứng với trị riêng lớn nhấtcủa toán trị riêng:

SBw DSww: (2.4) Để giải toán trị riêng trên, dùng hàmeig Matlab để tìm giá trị

(66)

Hình 7.7: Phép chiếu LDA tạo phân bố lý tưởng liệu thu hệ trục hồn tồn phân chia thành hai lớp khác

Để kiểm tra chất lượng đạt hệ thống vừa xây dựng, tiến hành kiểm tra với16

ảnh mèo và16ảnh chó (hồn tồn độc lập với tập ảnh huấn luyện ban đầu) %Test on the testing dataset:

testing_set=’./testing’ % wavelet transformation

testing_wavelet = dc_wavelet(testing_set); % SVD projection

TestMat = U’*testing_wavelet; % LDA projection

pval = w’*TestMat; % 16 cats and 16 dogs

hiddenlabels=[1,1,1,1,1,1,1,1,1,1,1,1,1,1,1,1 ,0,0,0,0,0,0,0,0,0,0,0,0,0,0,0,0];

TestNum = length(pval); %cat = 1, dog =

ResVec = (pval>threshold) disp(’Number of mistakes’);

errNum = sum(abs(ResVec - hiddenlabels)) disp(’Rate of success’);

sucRate = 1-errNum/TestNum

Với liệu xét, tỉ lệ nhận dạng đạt khoảng72% Chúng xin nhường lại phần cải tiến cho độc giả Epsilon số báo Đoạn codes tham khảo tìm thấy tại:Pet-recognition

Acknowledgement

(67)

Đặng Nguyễn Đức Tiến (Đại học Trento, Italia)

LỜI GIỚI THIỆU

Tiếp nối số báo trước, chun mục Tốn học Giải trí kỳ giới thiệu với độc giả toán logic kinh điển: Bài toán cân tiền (counterfeit coin problem)

Dạng thức chung toán cân tiền có đồng tiền giả đặt lẫn lộn với đồng tiền thật Các đồng tiền giả có bề ngồi giống hệt tiền thật khác khối lượng so với tiền thật Cần sử dụng cân đĩa cân số với số lần cân xác định để tìm (một số) đồng tiền giả Ở nhắc lại hai khái niệm hai loại cân sử dụng Cân đĩa giúp so sánh khối lượng vật đặt bên đĩa cân kết trả nặng hơn, nhẹ hơn, cân thăng Cân số cho phép xác định xác khối lượng vật đặt cân Trong tốn trình bày số này, giả sử cân sử dụng ln có độ xác tuyệt đối

Tương tự tốn đội nón, tốn cân tiền theo đời từ sớm, đến kỷ 20, lời giải thức ghi nhận cơng bố Và tương tự toán anh em mình, tốn cân tiền có nhiều biến thể khác nhau, mà sưu tập 53 phiên toán

Trong chun mục Tốn học Giải trí kỳ này, chúng tơi chọn lọc giới thiệu với bạn đọc5bài tốn cân tiền sử dụng cân đĩa Trong số tiếp theo, chúng tơi giới thiệu tốn sử dụng cân số

(68)

1 Bài toán cân tiền với đồng xu

Bài toán cân tiền có phát biểu sau:

Có đồng xu giống hệt nhau, có đồng giả nặng đồng lại Sử dụng cân đĩa không sử dụng thêm cân, lần cân, đồng giả

Bài toán đơn giản xuất nhiều tạp chí sách tốn học vui, giải trí Bạn đọc hẳn nhanh chóng tìm phương án cho toán với cách cân sau:

Lần 1:Đặt đĩa cân đồng Ta có trường hợp: cân thăng cân lệch Nếu cân thăng bằng, ta suy đồng giả thuộc đồng chưa cân Nếu cân lệch, ta suy đồng giả thuộc nhóm đồng nặng Như vậy, ta ln xác định nhóm đồng có chứa đồng giả

Lần 2:Chọn đồng đồng vừa xác định đặt đĩa cân đồng Ta có trường hợp đây: Nếu cân thăng bằng, suy đồng chưa cân đồng giả Nếu cân lệch, đồng nặng đồng giả

Như vậy, với lần cân, ta xác định đồng giả (biết trước nặng hơn) số đồng

Ta nâng cấp tốn lên27đồng với3lần cân, lần đầu cân bên9đồng áp dụng phương pháp tương tự biết đồng giả đồng Tổng quát hóa tốn vớinlần cân, ta xác định đồng giả (biết trước nặng hơn) số3nđồng!

Bài toán đơn giản, thay đổi điều kiện, ta có tốn hóc búa vơ thú vị, mà tiêu biểu toán

2 Bài toán 12 đồng xu

Theo chúng tôi, toán kinh điển tiếng số toán cân tiền Phiên mà chúng tơi tìm xuất phát từ Dyson Lyness vào năm 1946 [1]

(69)

Có12đồng xu giống hệt nhau, có đồng giả có khối lượng khác với11đồng cịn lại, khơng biết nặng hay nhẹ Sử dụng cân đĩa không sử dụng thêm cân, bằng3lần cân, đồng giả

Thoạt nhìn, tốn giống với toán9đồng xu Tuy nhiên, với yếu tố khác biệt đồng giả nhẹ hay nặng so với đồng thật Và với yếu tố này, tốn trở nên khó khăn nhiều

Trước bước sang phần lời giải, hi vọng độc giả dành thời gian để thử sức với toán để thấy thú vị Một khảo sát chi tiết toán độc giả xem [2] Richard Bellman, nhà toán học tiên phong Quy hoạch động

Và bây giờ, giới thiệu3cách giải khác cho toán mở rộng cho toán

2.1 Cách giải logic

Chúng tơi sử dụng tốn diễn đàn Toán học, chuyên mục đố vui với bạn bè 15 năm qua cách giải mà hay nhận Vì chất cách giải chuỗi suy luận logic nên tạm gọi phương pháp cách giải logic

Cách làm sau: Chia 12 đồng thành nhóm đặt tên cho đồng tương ứng với nhóm làaaaa,bbbbvàcccc

Lần 1:Cânaaaavàbbbb

Trường hợp 1.Nếu cân thăng bằng, ta suy đồngaaaavàbbbbđều thật

Lần 2:đặt đồng bất ỳ đồng đồng đồng cịn lại, ví dụaaavàccc Ở có trường hợp:

Trường hợp 1.1.Nếu cân thăng bằng, suy đồngccòn lại giả Ta cân lần biết nặng nhẹ

Trường hợp 1.2.Nếu cân lệch, ta biết đồng giả đồngccc, ta biết nặng hay nhẹ Lần cuối đơn giản cân đồng số đồngccc với Nếu cân bằng,ccòn lại giả (đã biết nặng nhẹ), không cân bằng, xác định

Trường hợp 2.Nếu cân lệch, ta suy đồngccccđều thật Khơng tính tổng qt, giả sử

aaaa < bbbb

Lần 2:cânaccc vàbaaa(mấu chốt tách nhómavàbra cho thành đồng đồng) Có trường hợp sau:

(70)

Trường hợp 2.2.Nếuaccc < baaathì đồnganày giả nhẹ, đồngbgiả nặng Lần cuối câncvàahoặcbta xác định

Trường hợp 2.3. Nếuaccc > baaathì đồng giả đồngaaa, giả nhẹ Với lần cuối ta xác định

Và vậy, toán giải trọn vẹn với3lần cân

2.2 Cách giải gán nhãn

Cách giải theo ghi nhận đề xuất Brian D Bundy vào năm 1996 [4]

Trước vào phân tích cách giải này, chúng tơi nêu đáp án ví dụ sau: gán số từ đến 12 cho 12 đồng thực lần cân:

Lần 1:Cân (1, 2, 7, 10) (3, 4, 6, 9)

Lần 2:Cân (1, 3, 8, 11) (2, 5, 6, 7)

Lần 3:Cân (2, 3, 9, 12) (1, 4, 5, 8)

Khi với kết lần cân, cách làm xác định đồng giả giả nặng hay nhẹ!

Ví dụ ta có kết lần nhau, lần 2: bên phải nặng hơn, lần 3: bên trái nặng hơn, ta biết đồng giả đồng đồng nhẹ

Làm có điều này? Ta thấy lần cân có khả năng, cân (ký hiệu0), bên trái nặng (ký hiệuT) bên phải nặng (ký hiệuP) Và ứng với12đồng ta có24khả đồng giả giả nặng hay giả nhẹ Do vậy, cách làm tìm cách chia cân cho kết lần cân ứng với đáp số nhất, ví dụ kết quảT TP ta có tương ứng đáp số “đồng giả nặng”, kết quảTP T có đáp số “đồng giả nặng” Cách làm đồng giả, giả nặng hay nhẹ mà cách xếp cân cho phù hợp

Bây ta phân tích cách làm Xét nhận xét sau:

Nhận xét 1:Ta có27khả khác lần cân (33D27) và24kết phân biệt Ta loại bỏ trường hợp đặt đồng bên cân lần cân (do đặt khơng cân đủ) Khi đó3trường hợp 000,T T T vàPPP bị loại bỏ ta lại 24khả ứng với24đáp số!

Nhận xét 2:Ta thấy kết quả0P T ứng với đáp án “đồng giả nhẹ” kết đối xứng

0TP, phải cho “đồng giả nặng” do0TP cho thấy đồng (giả) xuất ở2lần cân sau Vì ta phân24khả thành2nhóm, nhóm ứng với12đồng xu

(71)

T T T T P P P P 0 0 0 T T T T P P P P T P P T P T P T 10 11 12

Ví dụ sau (để tìm cách gán thỏa u cầu khơng khó cần phải có chiến thuật, chúng tơi xin phép lược bớt cách gán sau gợi ý):

Để chọn đồng xu lần cân, ta đặt theo dòng, làT, ta đặt đồng xu bên trái vàP, ta đặt bên phải Khi phương án với thứ tự, giả nặng ngược lại ta xét đối xứng có giả nhẹ Với phương án ví dụ trên, ta có:

Lần 1:(1, 2, 3, 4) (5, 6, 7, 8)

Lần 2:(4, 5, 6, 7) (8, 9, 10, 11)

Lần 3:(1, 6, 9, 12) (2, 4, 7, 10)

Ta thấy cách khác xếp đồng xu khác với cách nêu Thử xét vài ví dụ sau:

- Nếu kết làT TP, tra vào bảng lập ta thấy ứng với “4”, “4 giả nặng”

- Nếu kết làP 0T, tra vào bảng ta thấy khơng có, giả nhẹ, lấy đối xứng ta có

T 0P, ứng với đồng 2, “2 giả nhẹ”

Và với cách làm này, ta tổng quát lên vớinlớn hơn3 Các mở rộng khác tốn làm dựa sở Tuy nhiên, nhược điểm vớinlớn, để tránh sai sót gán nhãn thường ta cần đến trợ giúp máy tính

2.3 Cách giải Jack Wert

Đây cách giải yêu thích người viết này, theo ghi nhận trangCut The Knotthì lời giải đưa Jack Wert [5], dùng tên tác giả để gọi cho phương pháp

Về mặt ý tưởng, cách làm giống cách đầu tiên, tác giả phát biểu hình thức dễ hiểu tổng quát hóa lên với trường hợpn > 3khá dễ dàng

Cách làm Jack Wert cho tốn sau:

Chia12đồng thành3nhóm, nhóm4đồng nhóm chia thành "đống"3đồng "đống"1đồng lẻ (ví dụ gói3đồng vào bao giấy giả sử khối lượng bao 0)

(72)

Lần 2:Hoán đổi "đống" lớn: "đống" cân bên trái qua cân bên phải, "đống" cân bên phải đặt "đống" chưa cân đặt cân bên trái Ghi nhận tình trạng cân

Nếu tình trạng cân thay đổi: ta xác định đống có chứa đồng giả, giả nặng hay giả nhẹ Với lần cân lại, ta biết đồng giả Nếu tình trạng cân không thay đổi: ta biết đồng giả vào đồng lẻ với lần cân, ta xác định đồng giả biết giả nặng hay giả nhẹ Một cách lý giải tuyệt đẹp mà không cần phải gán nhãn!

Bây thử xem làm cách để mở rộng cách giải chonlớn hơn, ví dụ1092

đồng với7lần cân: ta chia thành3nhóm, nhóm gồm 364đồng Sau đó, nhóm, ta “gói” đồng thành đống243,81,27,9,3,1đồng (6 “đống”)

Lần 1:Đặt toàn đống nhóm lên cân Ghi nhận tình trạng cân

Lần 2:Hốn đổi đống lớn (chứa 243 đồng) từ cân bên trái sang cân bên phải, từ cân bên phải từ bên ngồi vào cân bên trái Ghi nhận tình trạng cân

Nếu tình trạng cân thay đổi, ta biết đống chứa đồng giả nặng hay nhẹ tương ứng Và ta tìm đồng giả trong243đồng sau5lần cân (35D243, biết giả nặng hay giả nhẹ)

Nếu cân khơng thay đổi, hốn chuyển đến đống thứ2(đống chứa81đồng) ghi nhận tình trạng cân Nếu cân thay đổi, ta biết đồng giả đống với tính nặng/nhẹ tương ứng, tìm đồng sau4lần cân Nếu cân lại khơng thay đổi tình trạng, ta lại tiếp tục hoán chuyển đống (27đồng) đống cuối cùng, ta sử dụng tối đa lần cân!

(73)

3 Nặng hay nhẹ hơn?

Bài tốn cân tiền thứ chúng tơi trích nguồn quen thuộc từblog Tanya Khovanova, biến thể thú vị (nhưng dễ hơn) tốn

CóN > 2đồng xu giống hệt nhau, có đồng giả có khối lượng khác với đồng cịn lại, khơng biết nặng hay nhẹ Sử dụng cân đĩa không sử dụng thêm cân, xác định số lần cân để xác định đồng giả nặng hay nhẹ so với đồng khác mà không yêu cầu phải xác định cụ thể đồng giả

Sau lời giải toán đăng tác giả (Tanya Khovanova): Chỉ cần lần cân ta xác định đồng giả nặng hay nhẹ so với đồng lại

Trường hợpN D3, đơn giản ta so sánh cặp hai đồng xu với

Xét trường hợpN D Ta cân lần đầu bên đồng Cân cân Không tính tổng quát, giả sử cân bên trái nhẹ Lần so sánh đồng cân bên trái, cân bằng, ta biết đồng giả nhóm cịn lại biết giả nặng Nếu cân lệch, ta biết đồng nhẹ giả nhẹ

Như vậy, ta tổng quát lên trường hợpN D3kvàN D4k với cách làm tương tự

VớiN bất kỳ, ta táchN làm3nhóm với số lượnga,a, vàbsao choab 2a Lần đầu ta cân nhóma, cân thăng bằng, ta có2ađồng thật, cân lần vớibđồng bên Nếu cân lệch, ta biếtbđồng lại thật ta cân tiếpađồng từb với trong2

nhómađã cân

Cách làm này, cịn thiếu cho trường hợpN D5, vớiN D5ta khơng thể tách thành3nhóm u cầu cách giải VớiN D5, lần đầu ta cân bên2đồng Nếu cân thăng bằng, ta cân lần cuối với đồng trong4đồng đồng lại Nếu cân lệch, ta làm lần cân thứ cho trường hợpN D4 Và toán giải

4 Bài toán 10 đồng xu

Ở tốn này, chúng tơi giới thiệu tốn khó, giải từ cách gần 20 năm (1997) nhà toán học Việt Nam quen thuộc với Epsilon số trước: Giáo sư Vũ Hà Văn (bạn đọc xem chi tiết [3])

Chúng phát biểu toán dạng đơn giản với10đồng xu sau:

(74)

Phát biểu tổng quát toán sau:

Cho tập gồmmđồng xu với tối đa2loại khối lượng khác Hãy xác định liệu tất các đồng có khối lượng lần cân nhất.

Bạn đọc tham khảo lời giải chi tiết [3], mục sử dụng lời giải sơ cấp cụ thể trường hợpN D10này sau:

Trước tiên, đánh số đồng xu từ đến 10 Dễ thấy, lần cân bất kỳ, cân chênh lệch ta giải tốn Do vậy, giả sử cân ln cân lần cân

Xét nhóm: nhóm gồm đồng1, nhóm gồm đồng2và3 Gọiavàblần lượt số đồng giả nhóm Như ta cóaD0hoặcaD1vàb D0, hoặcbD1, hoặcbD2

Lần 1:cân đồng (1, 2, 3) đồng (4, 5, 6) Do cân cân nên ta biết số đồng giả đồng (4, 5, 6) a + b

Lần 2:cân đồng lại (7, 8, 9, 10) đồng (1, 4, 5, 6) Do cân cân nên ta biết số đồng giả đồng (7, 8, 9, 10) là2aCb

Lần 3:cân (1, 7, 8, 9, 10) (2, 3, 4, 5, 6) Do cân cân nên ta có3aCbDaC2b, suy

2a Db Điều nghĩa hoặcaDb D 0(nghĩa tất đồng thật) hoặca D1và

b D 2(nghĩa tất đồng giả) Trong tình huống, ta tất đồng có khối lượng với hay khơng!

5 Bài tốn đồng xu

(75)

Có đồng xu, đồng có ghi số từ đến cho biết khối lượng tương ứng chúng 1gr, 2gr, 9gr Biết có đồng số bị lỗi có khối lượng nhẹ khối lượng quy định Với cân đĩa không sử dụng thêm cân, liệu với lần cân, đồng bị lỗi này?

Lời giải toán đơn giản xin dành cho độc giả Chúng kết thúc chuyên mục hẹn gặp lại số tới, với chuyên đề toán cân tiền với cân số

Tài liệu tham khảo

[1] F J Dyson and R C Lyness,Math Gazette, vol 30, 1946.

[2] R Bellman and B Glass, “On various versions of the defective coin problem,”Information and Control, vol 4, pp 118–131, 1961.

[3] D N Kozlov, V H Vu, “Coins and Cones,"Journal of Combinatorial Theory, Series A, vol 78, no 1, pp 1–14, 1997

[4] B D Bundy, “The Oddball Problem,”Mathematical Spectrum, vol 29, no 1, pp 14–15, 1996/7

(76)(77)

KỲ THI VMO 2014

Nguyễn Tiến Dũng - Hà Nội

Tóm tắt

Bài viết đưa góc nhìn tác giả hình học số kỳ VMO 2014 khai thác xung quanh cấu hình tốn

Bài hình học số kỳ VMO 2014 có nội dung đề cập [1] sau:

Bài toán Cho tam giác nhọnABC nội tiếp đường trịn.O/vớiAB < AC GọiI là trung điểm cungBC khơng chứaA TrênAC lấy điểmK khácC sao choIK DI C Đường thng

BK ct.O/tiD.D ÔB/v ct ng thngAI tiE ng thẳngDI cắt đường thẳng

AC tạiF.

1 Chứng minh rằngEF D BC

2 .

2 TrênDI lấy điểmM sao choCM song song vớiAD Đường thẳngKM cắt đường thẳng

BC tại N Đường tròn ngoại tiếp tam giỏcBKN ct.O/ti P P ÔB/ Chng minh rng ng thẳngPK đi qua trung điểm đoạn thẳngAD.

Có thể thấy hai ý toán không liên quan tới nhau, ý ý tốn Vì vậy, đề phát biểu gọn lại sau

Bài toán Cho tam giác nhọnABC nội tiếp đường tròn.O/vớiAB < AC GọiI là trung điểm cungBC không chứaA TrênAC lấy điểmK khácC sao choIK DI C Đường thẳng

BK cắt O/tại D.D ÔB/ TrờnDI ly imM sao cho CM song song với AD Đường thẳngKM cắt đường thẳngBC tạiN Đường tròn ngoi tip tam giỏcBKN ct.O/tiP P ÔB/ Chng minh rng đường thẳngPK đi qua trung điểm đoạn thẳngAD.

Lời giải thứ nhất. DoI trung điểm cungBC không chứaAcủa đường tròn.O/nênIB D

I C D IK Ta thấy ∠AKI D 180ı ∠CKI D 180ı ∠I CK D ∠ABI nên AIB D

AIK.g:c:g/ Vì K đối xứng với B qua AI Ta có ∠DCK D ∠ABK D ∠AKB D

∠DKC nên dễ thấyDI trung trực đoạnCK Chú ý đến tính đối xứng qua trụcDI

CM k AD ta có∠MKC D ∠M CK D ∠DAC D∠KBN nênAC tiếp tuyến đường trịn.BKN / GọiE giao củaDI vàAC, thì∠EKP D ∠KBP D∠EIP nên tứ giác

EP IK nội tiếp, suy raIPK DIEK D90.PK ct.O/tiF F ÔP / Chỳ ý rằngK trực tâm tam giácADI nên ta dễ dàng chứng minh đượcAFDKlà hình bình hành Do

(78)

F

P

N

M

E

D

K

I

O

A

B

C

Lời giải thứ hai. Trong lời giải thứ nhất, ta chứng minh đượcAC tiếp tuyến đường trịn.BKN / Từ đó, ý đến tính đối xứng quaAI, ta cóAB tiếp tuyến đường trịn.BKN / GiLl giao caAP v ng trũn.BKN /.LÔP / VỡKLP D ∠KBP D

∠DAP nên KL k AD.PK cắt AD tạiS Bằng biến đổi góc đơn giản, ta thu cặp tam giác đồng dạngSAK vàKPL,SDK vàBPL Chú ý tứ giácBPKLđiều hịa ta có

SA SK D

KP KL D

BP BL D

SD

SK nênSADSD Suy đpcm

L

P

N

M

D

K

I

A

O

(79)

Nhận xét.

1 Trong lời giải thứ nhất, ta có ∠KPN D ∠KBN D ∠DAC D ∠KIE D ∠KPE nên

P; N; Ethẳng hàng

2 Việc phát biểu lại làm cho đề tốn hay có ý nghĩa Trên đây, tác giả trình bày hai lời giải túy hình học cho toán Lời giải thứ tác giả Lời giải thứ hai dựa ý tưởng sử dụng kiến thức tứ giác điều hòa chùm điều hịa thành viên diễn đàn Tốn học Mathscope.org có nicknamevinhhai(trong [3]), tác giả chỉnh lí lại để có lời giải túy hình học Chú ý rằngK trực tâm tam giácIAD, ta phát biểu lại tốn sau:

Bài toán Cho tam giácABC nội tiếp đường tròn.O/ Các đường cao hạ từBC theo thứ tự cắt.O/tại DE KẻEF k BC.F 2AB/.HF cắtDE tạiK Đường tròn.HKD/cắt

.O/tiP P ÔD/ Chng minh rngPH chia ụiBC.

P

K

H

F

E

D

O

A

B

C

Từ nhận xét tốn tốn 3, ta đề xuất toán sau:

Bài toán Cho tam giácABC nội tiếp đường trịn.O/, có trực tâmH Các đường caoBD

CE theo thứ tự cắt.O/tạiF G KẻFM k GN kBC.M AC; N 2AB/.HM HN

(80)

P

K

L

M

H

N

E

D

G

F

O

A

B

C

Lời giải. Dễ thấy tứ giácADHE nội tiếp đường trịn đường kínhAH GọiP giao đường trũn ng kớnhAH vi.O/.P ÔA/ Chỳ ý rngAB l trung trực củaGH vàGN k BC

ta có ∠HLF D ∠LGH C∠LHG D ∠FBC C∠NGH D ∠HBC C∠H CB D ∠BAC

D 90ı ∠ABF D ∠APH ∠APF D ∠HPF nên tứ giácHLPF nội tiếp Từ đó, ta có

∠LPH D ∠LFH D ∠EAH D ∠EPH, thếP; E; Lthẳng hàng Chứng minh tương tự

P; D; K thẳng hàng Do đóDK; ELcắt điểmP thuộc.O/

Bài tốn Cho tam giácABC có đường caoBD; CE cắt tạiH GọiF; Gtheo thứ tự điểm đối xứng vớiH quaAC; AB.I là trung điểmF G.K; Ltheo thứ tự giao điểm khác

I của đường tròn ngoại tiếp tam giácHID; HIE vớiF G.DK; ELcắt tạiP Chứng minh rằngPH chia đôiBC.

I

P

K

L

H

E

D

G

F

A

B

C

(81)

Bài tốn Cho tam giácABC nội tiếp đường trịn.O/, trực tâmH, đường caoBK; CL.

AH; CH theo thứ tự cắt.O/tạiDF Lấy điểmM sao cho hai tam giácMFH HBC

đồng dạng (M; A nằm phía đối vớiKL/.MLcắt O/tại N KẻDP k KL.P 2.O//. Chứng minh rằngNP chia đôiKL.

N

P

R

L

M

D

H

F

E

K

Q

O

A

B

C

Lời giải. Dễ thấyHM ?AF; FM kKL.BH cắt.O/tạiE thìKLlà đường trung bình tam giácHEF nênM thuộcEF Từ đó, theo tốn dễ dàng suy raN thuộc đường trịn đường kínhAH.NH cắtBC tạiQthìQlà trung điểm củaBC,QK vàQLlà tiếp tuyến đường trịn đường kínhAH Theo tính chất quen thuộc thìAO ?KLnênAO ?DP, suy

AD DAP GọiRlà trung điểmKL Chú ý rằngNH đường đối trung tam giácNKL

ta có∠ANRD ∠ANK C∠KNR D ∠ALKC∠LNH D ∠ACB C∠BAD D ∠ACD D

∠ANP nênN; R; P thẳng hàng VậyNP chia đôiKL

Từ cấu hình tốn tốn [4], ta thu toán hay sau:

Bài toán Cho tam giácABC nội tiếp đường tròn.O/ Các đường cao hạ từB; C của tam giácABC theo thứ tự cắt đường tròn.O/tạiD E.DEtheo thứ tự cắtAC AB tạiM

N KẻDP kEQkBC.P 2AC; Q2 AB/ Chứng minh rằng:

1 Đường thẳng nối trung điểmMQNP đi qua tâm đường tròn Euler tam giácABC. 2 Trực tâm tam giácABC; AMQ; ANP thẳng hàng.

Lời giải. Để giải toán ta cần sử dụng đến bổ đề ERIQ (Equal Ratios In Quadrilateral) phát biểu sau:

Bổ đề ERIQ. Cho hai ba điểm thẳng hàng A1; A2; A3 B1; B2; B3 cho

A1A2

A1A3 D

B1B2

B1B3

D k Nếu C1; C2; C3 theo thứ tự chia A1B1; A2B2; A3B3 theo tỷ số

C1; C2; C3thẳng hàng

C1C2

(82)

Bổ đề ERIQ kết trở nên quen thuộc với nhiều bạn đọc, tìm thấy số cách chứng minh tài liệu khác (chẳng hạn [2]), xin khơng trình bày Trở lại toán,

1 Chứng minh đường thẳng nối trung điểmMQvàNP qua tâm đường tròn Euler tam giácABC

X

X'

X

2

X

1

N

M

K

P

H

Q

L

D

E

O

A

B

C

Các đường cao BK; CL tam giác ABC cắt H Dễ thấy KL k DE nên ta có

∠CMD D∠AMN D∠ABC D∠BQE, từ đóCDM BEQ Chứng minh tương tự ta cóCDP BEN Do BQ

BN D CM

CP Dkvà LQ LN D

KM

KP GọiX; X

0; X

1; X2theo thứ tự trung điểmBC; KL; MQ; NP Theo bổ đề ERIQ, ta cóX0; X1; X2thẳng hàng;X; X1; X2 thẳng hàng XX1

XX2

Dk Từ với ý rằngXX0đi qua tâm đường trịn Euler tam giác

ABC ta có đpcm

2 Chứng minh trực tâm tam giácABC; AMQ; ANP thẳng hàng

Y

Y1

Y2

Z

Z1

Z2

O1

O2

M

G

G

1

G2

X

X1

X

2

M

A

A

O

B

C

N

P

Q

B

C

N

P

(83)

GọiG; G1; G2theo thứ tự trọng tâm tam giácABC; AMQ; ANP Ta có

AG AX D

AG1

AX1 D

AG2

AX2 D

3 nên theo định lý ThalesG; G1; G2 thẳng hàng GG1

GG2 Dk

GọiY; Y1; Y2theo thứ tự hình chiếu O; O1; O2 AC; Z; Z1; Z2 theo thứ tự hình chiếu O; O1; O2 AB Ta có Y Y1 D AY1 AY D

1

2.AM AC / D

2CM Y Y2 DAY2 AY D

1

2.AP AC / D

2CP nên Y Y1

Y Y2

D CM

CP Dk Chứng minh tương tự ta

có ZZ1

ZZ2

D k Chú ý rằngOY kO1Y1 kO2Y2 vàOZ k O1Z1 kO2Z2, theo định lý Thales,

ta có O; O1; O2 thẳng hàng

OO1

OO2

D k Gọi H1; H2 theo thứ tự trực tâm tam giác

AMQ; ANP HG

HO D

H1G1

H1O1

D H2G2

H2O2 D

3 Chú ý GG1

GG2

D OO1

OO2

D k, theo bổ đề ERIQ thìH; H1; H2thẳng hàng (đpcm)

Nhận xét.Bài toán giải dựa ý tưởng Nguyễn Văn Linh (trong [2]) Kostas Vittas (trong [4]) Ta chứng minh trực tâm bốn tam giácABC; AMQ; ANP; AKL

thẳng hàng Một cách tổng quát, ta có kết sau: Các điểm chia đoạn thẳng nối trọng tâm tâm đường tròn ngoại tiếp tam giácABC; AMQ; ANP; AKLtheo tỷ số thẳng hàng

Từ tốn 7, ta đề xuất tốn sau:

Bài toán Cho tam giácABC nội tiếp đường tròn.O/ Các đường cao hạ từB; C của tam giácABC theo thứ tự cắt đường tròn O/tạiD E.DE theo thứ tự cắtAC AB tạiM

N KẻDP kEQkBC.P 2AC; Q2AB/ Chứng minh đường thẳng nối trung điểm các đoạnMQ; NP và đường thẳng nối tâm đường tròn Euler tam giácAMQ; ANP

cắt điểm thuộc đường thẳng Euler tam giácABC.

Giản lược yếu tố toán 7, tác giả tìm tốn hình học thú vị sau:

(84)

N

Q

M

P

L

K

T

G

F

H

E

D

O

A

B

C

Lời giải. GọiBK; CL đường cao tam giácABC Đường tròn.AKL/cắt O/tại

P khácA Tương tự toán 8, ta chứng minh hai tam giác BEG vàCDF đồng dạng cóEL; DK đường cao tương ứng nên LB

LG D KC

KF Chú ý hai tam giác PBLvà P CK đồng dạng, ta có hai tam giácPLG vàPKF đồng dạng suy hai tam giácPLK

P GF đồng dạng Từ tứ giácAP GF nội tiếp Tiếp tuyến tạiF; G đường tròn.AF G/cắt tạiT Tiếp tuyến tạiK; Lcủa đường tròn.AKL/cắt trung điểmM củaBC Dễ thấyP; H; M thẳng hàng.PLcắtDE tạiN Ta có∠NPH D∠LAH D∠ECB D∠NDH

nên tứ giácNHDP nội tiếp Từ cặp tam giác đồng dạngPLG vàPKF,LGH vàKFD

dễ thấy hai tam giác P GH PFD đồng dạng Chú ý tứ giác NHDP nội tiếp ta có

∠PHG D∠PDF D∠PHN nênN; G; H thẳng hàng.P T cắt đường tròn.AF G/tạiQkhác

P Do hai tam giácPLK vàP GF đồng dạng nên ta có cặp tam giác đồng dạngPLM

P GT,PLH vàP GQ Từ đó∠GAQD∠GPQD∠LPH D∠LAH nênQthuộcAH Chú ý rằngKLkDE, ta có∠AF N D∠AKLD180ı ∠APN nên tứ giácAPNF nội tiếp Ta thấy

∠F NQD∠FAQD∠DBC D∠FEC vậyNQ kCE Vì∠PNG D∠P GT D∠PLM

nênNH kLM Ta có PN

PL D PH PM D

PQ

P T suy raNQkLT Từ đóLT kNQk CE nênT

thuộcCE (đpcm) Nhận xét.

1 Đây tốn hay cách phát biểu giản đơn tính không cân xứng yếu tố giả thiết tốn Lời giải hồn thiện với cộng tác thầy Trần Quang Hùng – giáo viên Trường THPT Chuyên KHTN, ĐHKHTN – ĐHQG Hà Nội

2 Trong lời giải trên, ta chứng minh PH

PM D PQ

P T nênHQ kM T Chú ý rằngQthuộc AH nên ta cóM T ?BC, suy raT thuộc trung trực củaBC

(85)

Bài tốn 10 Cho tam giácABC nội tiếp đường trịn.O/ Các đường cao hạ từB; C của tam giác ABC theo thứ tự cắt O/ tạiD E.DE theo thứ tự cắt AC AB tại M N Kẻ

DP kEQ kBC.P AC; Q2AB/ Tiếp tuyến tạiM; Qcủa đường tròn ngoại tiếp tam giác

AMQcắt tạiS Tiếp tuyến tạiN; P của đường tròn ngoại tiếp tam giácANP cắt nhau tạiT Chứng minh rằngS T vng góc vớiBC.

Bài tốn 11 Cho tam giácABC nội tiếp đường trịn.O/có trực tâmH.BH; CH theo thứ tự cắt.O/tạiD E Kẻ DM k EN k BC.M 2AC; N AB/ Trung trực củaBC theo thứ tự cắtBH; CH tại P Q Chứng minh đường tròn ngoại tiếp tam giác HMP

HNQcắt điểm thuộc.O/.

G

P

M

S

L

K

Q

N

H

E

D

A

O

B

C

Lời giải. GọiBK; CLlà đường cao tam giácABC Đường trịn đường kínhAH cắt

.O/tạiGkhácA GọiS trung điểm củaBC thìG; H; S thẳng hàng Vì∠LGS D∠LAH D

∠LCS nên tứ giácC GLS nội tiếp VìS C DSLnênGS phân giác∠LGC Từ cặp tam giác đồng dạngGLH vàGS C,LHN vàS CQta có hai tam giácGLN vàGSQđồng dạng Suy hai tam giácGNQvàGLS đồng dạng Do ta có∠NGQD∠LGS D∠LAH D∠NHL

nên tứ giácHNGQnội tiếp hay đường tròn.HNQ/đi quaGthuộc.O/ Chứng minh tương tự ta có đường trịn.HMP /cũng quaG Từ ta có đpcm

(86)

M

P

L

K

T

G

F

H

E

D

O

A

B

C

Lời giải thứ hai cho toán 9. Gọi BK; CL đường cao tam giác ABC Đường tròn.AKL/cắt.O/tạiP khácA, ta chứng minh tứ giácAP GF nội tiếp GọiM trung điểm củaBC thìP; H; M thẳng hàng Trung trực củaBC cắtCH tạiT Tương tự chứng minh tốn 11, ta có hai tam giácP GT vàPLM đồng dạng Chú ý hai tam giácP GF

vàPLKđồng dạng, ta thấy hai tam giácMKLvàTF G đồng dạng Chú ý rằngMK; MLlà tiếp tuyến đường tròn.AKL/, suy raTF; T G tiếp tuyến đường tròn.AF G/ Ta có đpcm

Ẩn giấu cấu hình trực tâm thơng qua tâm đường trịn nội tiếp tam giác, đề xuất nhiều tốn hay khó Chẳng hạn như:

(87)

D

M

Z

Y

X

A

3

A

2

A

1

F

E

I

O

A

B

C

Gợi ý. GọiAI; BI; CI theo thứ tự cắt đường tròn.O/ngoại tiếp tam giácABC tạiX; Y; Z Thế thìI trực tâm tam giácX Y Z Từ đưa cấu hình tốn 4, ta có kết quảEA1 cắtFA2tạiA3 giao điểm của.O/và đường tròn đường kínhIX Theo tính chất quen thuộc,

A3là tiếp điểm đường trịn mixtilinear nội tiếp ứng với gócAcủa tam giácABC Tương tự với điểmB3vàC3, từ đóAA3; BB3; C C3đồng quy điểm thuộcOI

Bài toán 13 Cho tam giácABC nội tiếp đường tròn.O/, ngoại tiếp đường tròn.I / Đường tròn.Oa/tiếp xúc vớiAB; AC và tiếp xúc với đường tròn.O/tạiS.D; Etheo thứ tự là các giao điểm khácB; C của đường trònSIB SI C vớiBC Đường thẳng quaB vng góc vớiAI cắtIE tạiM, đường thẳng quaC vng góc vớiAI cắtID tạiN LấyP; Qthuộc

BC sao cho IP k AB; IQ k AC Chứng minh đường thẳng nối trung điểm đoạn

MQ; NP chia đơiOI.

Bài tốn 14 Cho tam giácABC nội tiếp đường tròn.O/, ngoại tiếp đường tròn.I /.AI cắt

.O/tạiDkhácA.E; F theo thứ tự trung điểm củaIB I C.DE; DF theo thứ tự cắtBC

tạiM N Đường thẳng quaI vng góc vớiAI theo thứ tự cắtAB; AC tạiK; L GọiP; Q

theo thứ tự giao điểm khácB; C của đường tròn.BIK/; CIL/với BC.IP cắt DF

tạiR,IQ cắtDEtạiS Chứng minh đường thẳng nối trung điểm đoạnMR; NS đường thẳng nối tâm đường tròn Euler tam giácDMR; DNS cắt điểm thuộcOI.

Bài toán 15 Cho tam giácABC nội tiếp đường trịn.O/, cóI là giao điểm đường phân giác.AI cắt O/tại D khácA.E; F; M theo thứ tự trung điểm củaIB; I C BC Gọi

H; K theo thứ tự giao điểm caBC viDEv ng trũn.IMF /.K ÔM /.IKctDF ti

(88)

Như vậy, tác giả trình bày xong góc nhìn tốn hình học số kỳ VMO 2014 Từ cấu hình đơn giản, ta đưa khai thác thú vị thơng qua góc nhìn khác

Cuối cùng, tác giả xin gửi lời cảm ơn tới thầy Trần Quang Hùng – giáo viên Trường THPT Chuyên KHTN, ĐHKHTN – ĐHQG Hà Nội, người đọc toàn viết đưa ý kiến quý báu giúp viết hoàn thiện

Tài liệu tham khảo

[1] Trần Nam Dũng, Lê Phúc Lữ Phan Đức Minh Đề thi chọn học sinh giọi cấp quốc gia THPT mơn tốn năm học 2013 – 2014, Lời giải chi tiết bình luận Diễn đàn Toán học Mathscope.org

http://forum.mathscope.org/showthread.php?t=46403

[2] huynhcongbang [VMO 2014] Bài – Hình học phẳng Mathscope.org http://forum.mathscope.org/showthread.php?t=46327

[3] Nguyễn Văn Linh Định lý ERIQ ứng dụng Euclidean Geometry Blog https://nguyenvanlinh.wordpress.com/2015/03/02/eriq-theorem/

(89)

MỘT BÀI TOÁN TRONG SÁCH "CÁC PHƯƠNG PHÁP

GIẢI TOÁN QUA CÁC KỲ THI OLYMPIC"

Nguyễn Ngọc Giang –TPHCM

Tóm tắt

Bài viết đưa cách giải khác khám phá phát triển tốn Qua đó, thể mối quan hệ biện chứng hình học Euclide, Afin xạ ảnh

1 Giới thiệu

Chương trình "Gặp gỡ tốn học 2015" kết thúc tốt đẹp Ngồi hoạt động dạy học, giao lưu việc cho đời sách "Các phương pháp giải toán qua kỳ thi Olympic" phần nói lên thành cơng chương trình Với nội dung phong phú, sách đề cập đến phương pháp giải phát triển tư dành cho em học sinh giỏi Trong đề toán cho em học sinh giỏi đặc biệt trọng đến toán sau:

Bài toán ([1], tr 205). Cho tam giácABC:Các điểmM; N thuộcBC:Các điểmP; Qtheo thứ tự thuộcAC; AB:Đặt O D MP \NQ; K D BO \NP; LD CO \MQ:Chứng minh rằng:AO; BL; CKđồng quy.

Đây toán hay gợi lên nhiều suy nghĩ Vì tốn đề cập đến khái niệm, tính chất "tam giác", "đồng quy", "giao điểm" nên tốn tốn hình học xạ ảnh Vì tốn hình học xạ ảnh nên hiển nhiên tốn hình học afin tốn hình học Euclide

Trong báo, chúng tơi đề cập đến mối quan hệ toán sơ cấp cao cấp qua tốn Mối quan hệ thể dạng tư tư sáng tạo Trong thuyết Giáo dục Bloom tư sáng tạo tư cao khó Đây dạng tư cần thiết khơng cho Giáo dục mà cịn cho tồn lĩnh vực khoa học đời sống Sáng tạo sáng tạo kim nam cho hành động người kỉ XXI

2 Các cách giải toán

2.1 Phương pháp chiếu xuyên tâm

ĐặtI DBL\CK; U DBO\MQ; V DCO\NP:Ta có

B.ALOC /D.QLUM /

D.M ULQ/(theo tính chất tỷ số kép hàng điểm) D.PK V N /(xét phép chiếu xuyên tâmO)

(90)

Áp dụng định lí:Cho hai chùms; a; b; c s; a0; b0; c0 Khi đó.sabc/D.sa0b0c0/khi khi

a\a0Ib\b0Ic\c0thẳng hàng.

Vớis DBC; aDBA; a0DCA; bDBL; b0DCK; c DBO; c0DCO ta cóA; I; Othẳng hàng VậyAO; BL; CKđồng quy [1]

Ngồi cách giải ta cịn giải toán phương pháp tọa độ Phương pháp tọa độ có ưu điểm phương pháp có tính chất thuật tốn Dù việc tính tốn nhiều tốn nhiều gặp khó khăn ngày ta có phần mềm trợ giúp tính toán nên cách giải phương pháp tọa độ cách giải ưa dùng

2.2 Phương pháp tọa độ khối tâm nhất

Tọa độ khối tâm (Homogeneous barycentric coordinates) phương pháp tọa độ Mobius tìm Nội dung phương pháp sau Cho tam giácABC;điểmP có tọa độ ba số.u W v W w/hiểu theo nghĩa hệ khối tâmutạiA; v tạiB;vàw tạiC

sao cho điểm cân tam giác tạiP Các khối tâm tỉ lệ với diện tích tam giácPBC; P CA

vàPAB:Khi điểmP nằm tam giác, ta sử dụng diện tích có dấu tam giác định hướng Các tọa độ khối tâm củaP với tam giác tham chiếuABC ba số

.x Wy Wz/thỏa mãn

x Wy Wz DPBC WP CAWPAB:

Ví dụ: Trọng tâmGcủa tam giác có tọa độ khối tâm làGD.1W1W1/:

2 Tâm đường trịn nội tiếpI có tọa độ khối tâm làI D.aWb Wc/:

Sau lời giải toán phương pháp tọa độ khối tâm

Ta kí hiệu phương trình đường thẳngX Y có thành phần tọa độx; y; zlàX Y DŒxWy Wz:

Phương trình đường thẳngMP có tọa độ ˇ ˇ ˇ ˇ 1 m ˇ ˇ ˇ ˇW ˇ ˇ ˇ ˇ p m ˇ ˇ ˇ ˇW ˇ ˇ ˇ ˇ p 0 ˇ ˇ ˇ ˇ

DŒ1Wmp; p:

Tương tự, phương trình đường thẳngNP có tọa độ

Œ1WnpW p:

Phương trình đường thẳngNQcó tọa độ ˇ ˇ ˇ ˇ q n ˇ ˇ ˇ ˇW ˇ ˇ ˇ ˇ n ˇ ˇ ˇ ˇW ˇ ˇ ˇ ˇ q ˇ ˇ ˇ ˇ

(91)

Tương tự, phương trình đường thẳngMQcó tọa độ

Œq mW m W1:

O DMP \NQnên tọa độ điểmO thỏa mãn hệ phương trình

nqx1 nx2Cx3 D0

x1Cmpx2 px3 D0 Giải hệ ta cóO D.p.n m/WnpqC1Wn.1Cmpq//:

Phương trình đường thẳngAO có tọa độ

ˇ ˇ ˇ ˇ

npq C1 n.1Cmpq/

0 ˇ ˇ ˇ ˇW ˇ ˇ ˇ ˇ

n.1Cmpq/ p.n m/

0 ˇ ˇ ˇ ˇW ˇ ˇ ˇ ˇ

p.n m/ npqC1

1 ˇ ˇ ˇ ˇ

DŒ0Wn.1Cmpq/W npqC1/:

Phương trình đường thẳngBO có tọa độ

ˇ ˇ ˇ ˇ

npq C1 n.1Cmpq/

1 ˇ ˇ ˇ ˇW ˇ ˇ ˇ ˇ

n.1Cmpq/ p.n m/

0 ˇ ˇ ˇ ˇW ˇ ˇ ˇ ˇ

p.n m/ npqC1

0 ˇ ˇ ˇ ˇ

DŒ n.1Cmpq/W0Wp.n m/:

Phương trình đường thẳngCO có tọa độ

ˇ ˇ ˇ ˇ

npq C1 n.1Cmpq/

0 ˇ ˇ ˇ ˇW ˇ ˇ ˇ ˇ

n.1Cmpq/ p.n m/

1 ˇ ˇ ˇ ˇW ˇ ˇ ˇ ˇ

p.n m/ npqC1

0 ˇ ˇ ˇ ˇ

DŒnpqC1Wp.m n/W0:

K DNP \BO nên tọa độ điểmK thỏa mãn hệ phương trình

x1Cnpx2 px3 D0

n.1Cmpq/x1Cp.n m/x3 D0 Giải hệ ta có

K D np.n m/Wm.npqC1/Wn2.1Cmpq/

:

(92)

LDCO\MQnên tọa độ điểmLthỏa mãn hệ phương trình

.npqC1/x1Cp.m n/x2 D0

mqx1 mx2Cx3 D0 Giải hệ ta có

LD.p.n m/WnpqC1Wm.1Cmpq//:

Phương trình đường thẳngBLcó tọa độ

Œ m.1Cmpq/W0Wp.n m/

Xét định thức

D ˇ ˇ ˇ ˇ ˇ ˇ

m.npqC1/ np.m n/ m.1Cmpq/ p.n m/

0 n.1Cmpq/ npqC1/

ˇ ˇ ˇ ˇ ˇ ˇ

Dm.npqC1/: n.1Cmpq//:p.n m/Cm.1Cmpq/:np.m n/: .npqC1//

Dmnp.npqC1/.1Cmpq/.n m/ 1C1/

D0:

VậyBL; CK; AI đồng quy điểm

2.3 Phương pháp tọa độ afin

Các tính tốn cách giải trợ giúp phần mềm Maple XVIII

Xét mục tiêu afinfAI AB;! AC!g, ta có

AD.0I0/; B D.1I0/; C D.0I1/:

Phương trình đường thẳngBC

.BC /WxCy 1D0: M; N 2BCnênM D.mI1 m/; N D.nI1 n/:

(93)

P D.0Ip/: Q2AB nên tọa độ điểmQlà

QD.qI0/:

Phương trình đường thẳngMQlà

x xM

xM xQ D

y yM

yM yQ hay

.MQ/W x m

m q

y 1Cm m D0:

Tương tự, phương trình đường thẳngMP

x m m

y 1Cm m p D0:

Phương trình đường thẳngNQlà

x n n q

y 1Cn n D0:

Phương trình đường thẳngNP

x n n

y 1Cn n p D0:

VìO DMP \NQnên tọa độ điểmO thỏa mãn hệ phương trình

ˆ <

ˆ :

x m m

y 1Cm m p D0 x n

n q

y 1Cn n D0

Giải hệ ta có

O D

m.np nq pqCq/ mq npCpq mCn qI

mnp mnq npq mpCmqCnqCpq q mq npCpq mCn q

:

Phương trình đường thẳngCO

x.mq npCpq mCn q/ m.np nq pqCq/

y

1 mnp mnq npq mpmq npCpq mCmqCn qCnqCpq q D0

Phương trình đường thẳngBO

x

1Cmq npm.np nq pqCpq mCCn qq/ C

y.mq npCpq mCn q/

(94)

LDCO\MQ;nên tọa độ điểmLlàLD.xLIyL/với

xL D

.np nq pqCq/m2

m2p 2mnpCmnqCnpq m2Cmn nq:

yLD

m2p m2q mpq mpC2mqCpq q

n m2p 2mnpCmnqCnpq m2Cmn nq:

K DBO\NP;nên tọa độ điểmK làK D.xKIyK/với

xK D

.mnp mnq mpqCmq npCnqCpq q/n

mnp 2mnq mpqCn2qCmnC2mq n2 npCpq mCn q:

yK D

mn2p mn2q n2pq 2mnpC2mnqCn2qC2npqCmp mq 2nq pqCq

mnp 2mnq mpqCn2qCmnC2mq n2 npCpq mCn q : Phương trình đường thẳngCK

x mnp 2mnq mpqCn2qCmnC2mq n2 npCpq mCn q

.mnp mnq mpqCmq npCnqCpq q/n y

1C mn2p mnmnp 2mnq mpq2q n2pq 2mnpCnC2q2mnqCmnCCn2mq n2qC2npq2 npCmp mq 2nq pqCpq mCn q Cq/ D0:

Phương trình đường thẳngBLlà

x

1C m2p 2mnp.np nq pqCmnqCnpq mCq/m22Cmn nq

Cy m

2p 2mnp

CmnqCnpq m2Cmn nq

.m2p m2q mpq mpC2mqCpq q/n D0: Phương trình đường thẳngAO

AO D

xC mq npm.np nq pqCpq mCCn qq

.mq npCpq mCn q/ m.np nq pqCq/

y mnp mnq npq mpmq npCpq mCmqCn qCnqCpq q.mq npCpq mCn q/ mnp mnq npq mpCmqCnqCpq q D0: I DCK\BLnên tọa độ điểmI D.xIIyI/là

xI D

.mnp mnq mpqCmq npCnqCpq q/mn/ı

m2nqCm2pq mn2p mnpq

Cn2pq m2n m2p m2qCmn2C2mnp n2q npq Cm2 mnCnq

yI D n m2np m2nq mnpq m2pCm2q mnpC2mnqCmpqCnpq Cmp

2mq nq pqCq//ı

(95)

Xét hiệu D.xA xI/.yO yI/ yA yI/.xO xI/:

Ta có D

.mnp mnq mpqCmq npCnqCpq q/mn

mnp mnq npq mp

CmqCnqCpq q mq npCpq mCn q

n m2np m2nq mnpq m2pCm2q mnpC2mnqCmpqCnpqCmp 2mq nq pq

Cq//ı

m2nq Cm2pq mn2p mnpqCn2pq m2n m2p m2qCmn2C2mnp n2q npqCm2 mnCnq///ı

m2nqCm2pq mn2p mnpqCn2pq m2n m2p m2qCmn2C2mnp n2q npqCm2 mnCnq

C n m2np m2nq mnpq m2p Cm2q mnpC2mnqCmpqCnpq

Cmp 2mq nq pqCq/

m.np nq pq

Cq/ mq npCpq mCn q

C .mnp mnq mpqCmq npCnqCpq q /mnı

m2nqCm2pq mn2p mnpqCn2pq m2n m2p m2qCmn2C2mnp n2q npqCm2 mnCnqı

m2nqCm2pq mn2p mnpqCn2pq m2n m2p m2qCmn2C2mnp n2q npqCm2 mnCnq

: Đơn giản hóabằng lệnh simplify Maple XVIII, ta được D0:

VậyAO; BL; CK đồng quy

2.4 Phương pháp tọa độ xạ ảnh

Xét mục tiêu xạ ảnhfA; B; CIO/:Ta có:

AD.1; 0; 0/IB D.0; 1; 0/IC D.0; 0; 1/:

Phương trình đường thẳngAB làx3 D0: Phương trình đường thẳngAC làx2D0: Phương trình đường thẳngBC làx1 D0:

VìM thuộcBC nênŒM DŒBCŒC hay ta có hệ phương trình

4

m1

m2

m3

5D

4

0

3

5C

4

0

3

5:

VậyM D.0; 1; m/:

Tương tự, ta cóN D.0; 1; n/:

(96)

ˇ ˇ ˇ ˇ 1 m ˇ ˇ ˇ ˇ ; ˇ ˇ ˇ ˇ 1 m ˇ ˇ ˇ ˇ ; ˇ ˇ ˇ ˇ 1 ˇ ˇ ˇ ˇ

DŒm 1; m; 1:

Tương tự, phương trình đường thẳngNO làŒn 1; n; 1: P DMO\AC nên tọa độ điểmP thỏa mãn hệ phương trình

.m 1/x1 mx2Cx3D0

x2 D0 VậyP D 1; 0; m 1/:

QDNO \ABnên tọa độ điểmQthỏa mãn hệ phương trình

.n 1/x1 nx2Cx3 D0

x3 D0 VậyQD.n; n 1; 0/:

Phương trình đường thẳngMQcó tọa độ ˇ

ˇ ˇ ˇ

1 m n

ˇ ˇ ˇ ˇ ; ˇ ˇ ˇ ˇ m 0 n ˇ ˇ ˇ ˇ ; ˇ ˇ ˇ ˇ n n

ˇ ˇ ˇ ˇ

DŒm.1 n/; mn; n:

Phương trình đường thẳngNP có tọa độ ˇ

ˇ ˇ ˇ

0 m 1 n ˇ ˇ ˇ ˇ ; ˇ ˇ ˇ ˇ

m 1 n ˇ ˇ ˇ ˇ ; ˇ ˇ ˇ ˇ 0 ˇ ˇ ˇ ˇ

DŒ1 m; n; 1:

Phương trình đường thẳngBO làx1 x3D0 Phương trình đường thẳngCO làx1 x2D0:

K DBO\NP nên tọa độ điểmK thỏa mãn hệ phương trình

.1 m/x1Cnx2 x3 D0

x1 x3 D0 VậyK D.n; m; n/:

LDCO\MQnên tọa độ điểmLthỏa mãn hệ phương trình

m.1 n/x1Cmnx2 nx3 D0

x1 x2 D0 VậyLD.n; n; m/:

Đường thẳngBLcó tọa độ ˇ ˇ ˇ ˇ n m ˇ ˇ ˇ ˇ ; ˇ ˇ ˇ ˇ 0 m n ˇ ˇ ˇ ˇ ; ˇ ˇ ˇ ˇ n n ˇ ˇ ˇ ˇ

DŒm; 0; n:

Đường thẳngCK có tọa độ ˇ ˇ ˇ ˇ m n ˇ ˇ ˇ ˇ ; ˇ ˇ ˇ ˇ n n ˇ ˇ ˇ ˇ ; ˇ ˇ ˇ ˇ 0 n m ˇ ˇ ˇ ˇ

DŒ m; n; 0:

Đường thẳngAI có tọa độ ˇ ˇ ˇ ˇ 0 1 ˇ ˇ ˇ ˇ ; ˇ ˇ ˇ ˇ 1 ˇ ˇ ˇ ˇ ; ˇ ˇ ˇ ˇ 0 ˇ ˇ ˇ ˇ

(97)

Xét định thức

D ˇ ˇ ˇ ˇ ˇ ˇ

m n m n 0 1

ˇ ˇ ˇ ˇ ˇ ˇ

Dm:n m/: n/D0:

VậyAI; BL; CK đồng quy điểm

3 Tương tự hóa tốn

Từ tốn xạ ảnh không gian xạ ảnh, cách chọn siêu phẳng khác đóng vai trị siêu phẳng vơ tận ta có nhiều tốn afin khác mà kết ta suy từ kết biết không gian xạ ảnh

Bằng cách chọnBC đường thẳng vô tận, ta toán sau

Bài toán Cho hình bốn đỉnhAPOQ Đường thẳng quaQsong song vớiPO cắt đường thẳng quaO song song vớiAP tạiL:Đường thẳng quaP song song vớiQO cắt đường thẳng quaO

song song vớiAQtạiK:Dựng hình bình hànhOLIK Chứng minh rằngA; I; Othẳng hàng.

Bằng cách chọn đường thẳngAC làm đường thẳng vô tận, ta toán sau

Bài toán Cho hai tiaBx; By TrênBxlấy hai điểmM; N và trênBylấy điểmQ GọiO điểm trênQN:Đường thẳng quaN song song vớiMO cắtBOtạiK:Đường thẳng quaO song song vớiBxcắtQM tạiL:Đường thẳng quaK song song vớiBx cắtBLtạiI:Chứng minh rằngOI kBy:

4 Mở rộng toán

Từ tốn 1, chúng tơi tổng qt hóa thành tốn cho đường trịn Ta toán sau

Bài toán Cho điểmM; B; Q; P; C; N nằm đường tròn GọiI là giao điểm củaMP

NQ: L; K lần lượt giao điểm củaCI vớiMQBI vớiNP:GọiT là giao điểm củaBL

(98)

Sau số cách giải toán

Chứng minh. Cách [2]

GọiS DBM \CN:

Áp dụng định lí Pascal cho lục giácMBQP CN ta cóA; I; S thẳng hàng GọiX1; X2; Y1; Y2 giao điểm củaBC vớiMQ; NP; MP; NQ:

Để chứng minhA; I; T thẳng hàng ta chứng minhB.CAT S /DC.BAT S /tức là.X1QLM /D

.X2PKN / 1/

Ta có, áp dụng định lí Ménélaus cho tam giácQX1Y2vớiL; I; C thẳng hàng

.X1QLM / D

LX1

LQ W M X1

MQ D I Y2

IQ: CX1

C Y2

(99)

Khi (1) tương đương với

I Y2

IQ: CX1

C Y2

:MQ M X1 D

I Y1

IP : BX2 BY1 :NP NX2 , MQ NP : IP IQ : CX1

M X1 W

BX2

NX2

D I Y1

BY1 W

I Y2

C Y2

:

Chú ý CX1

M X1

D CQ MBI BX2 NX2 D BP CN: 1/tương đương với

sinQIM\

sin\IMQ:

sinI NP[

sinP I N[

! : CQ MB: CN BP

D I Y1

BY1 W

I Y2

C Y2

: 2/

Áp dụng định lí sin cho tam giácIBY1vàIBM ta có

I Y1

BY1

D sinIBC[ sin\BIM D

sinIBC[ MB:sinBMPIB\

:

Tương tự cho I Y2

C Y2

:

Vậy ta có

I Y1

BY1 W I Y2

C Y2

D sinIBC[

MB:sinBMPIB\ W

sinI CB[ NC:sinICCNQ\

: 3/

Từ (2), (3), ta cần chứng minh

CQ MB:

NC BP D

I Y1

BY1

:C Y2 I Y2

D IB

MB: NC

I C :

sinIBC[

sinBMP\:

sinCNQ\

sinI CB[

, IB sinI CB[:

sinIBC[ I C

!

: sinCNQ\

sinBMP\

!

D CQ

BP

, CQ sinCNQ\ D

BP

sinBMP\ ,

CQ

sinQBC\ D BP

sinBCP[ ,

BC

sinBQC\ D

BC

sin\BP C:

Điều hiển nhiên Vậy, ta có điều phải chứng minh Cách [3]

GọiD DBM \CN; U DBI \MQvàV DCI \NP:

Áp dụng định lí Pascal cho điểmB; Q; N; C; P; M nằm đường trịn.O/ta có AD

BQ\CP; I DQN \PM vàD DNC \MB nênA; I vàD thẳng hàng

Tiếp theo ta chứng minhT nằm đường thẳngAID phương pháp tỉ số kép Ta có

B.ALID/ D.QLUM /

D.M ULQ/(theo tính chất tỷ số kép hàng điểm) D.PK V N /(xét phép chiếu xuyên tâmI)

DC.AKID/:

(100)

Thực phép chiếu xuyên tâm từ điểmO0nằm mặt phẳng tờ giấy xuống mặt phẳng không song song với mặt phẳng tờ giấy đường trịn khơng cịn đường trịn mà trở thành thiết diện cônic (elip, hypebol, parabol) Từ toán ta thu toán sau

Bài toán Cho điểmM; B; Q; P; C; N nằm thiết diện cônic GọiI là giao điểm của

MP NQ: L; K lần lượt giao điểm củaCI vớiMQ BI vớiNP:GọiT là giao điểm củaBLCK:Chứng minh rằngA; T; I thẳng hàng.

Bài toán tổng quát toán toán sau

Bài toán Cho điểmM; B; Q; P; C; N nằm cônic.S / GọiI là giao điểm củaMP

NQ: L; Klần lượt giao điểm củaCI vớiMQBI vớiNP:GọiT là giao điểm củaBL

CK:Chứng minh rằngA; T; I thẳng hàng.

Để chứng minh toán 6, ta sử dụng cách chứng minh toán

Chứng minh. GọiD DBM \CN; U DBI \MQvàV DCI \NP:

Áp dụng định lí Pascal cho điểmB; Q; N; C; P; M nằm cơnic.S /ta cóADBQ\

CP; I DQN \PM vàD DNC \MB nênA; I vàDthẳng hàng

Tiếp theo ta chứng minhT nằm đường thẳngAID phương pháp tỉ số kép Ta có

B.ALID/ D.QLUM /

D.M ULQ/(theo tính chất tỷ số kép hàng điểm) D.PK V N /(xét phép chiếu xuyên tâmI)

DC.AKID/:

Từ suy raBL AK cắt điểmT AID hay làA; T; I thẳng hàng Vậy

(101)

5 Kết luận

Chúng ta vừa có số khám phá thú vị từ toán Các cách giải khác nhau, toán mở rộng mang đến cho nhiều điều bổ ích Bài viết cần trao đổi thêm? Mong chia sẻ bạn

Tài liệu tham khảo

[1] Huỳnh Chí Hào (2015), "Hàng điểm điều hịa", Các phương pháp giải tốn qua kỳ thi Olympic (Trần Nam Dũng (chủ biên), Võ Quốc Bá Cẩn, Trần Quang Hùng, Lê Phúc Lữ, Nguyễn Tất Thu), Chương trình trọng điểm quốc gia phát triển tốn học giai đoạn 2010-2020

[2] Võ Duy Khánh, Diễn đàn T1K23-Huyền Thoại CHT

(102)(103)

NGÀY THỨ HAI

Trần Quang Hùng – THPT chuyên KHTN, Hà Nội

Tóm tắt

Bài viết xoay quanh tốn hình học thi IMO năm 2009 ngày thứ hai với mở rộng khai thác đồng thời ứng dụng mở rộng khai thác với cơng cụ hình học túy

1 Mở đầu

Trong kỳ thi IMO năm 2009 ngày thứ ngày thứ hai có tốn hình học Trong tốn hình học ngày thứ toán lớn khai thác nhiều với mở rộng ứng dụng hình học ngày thứ hai ý hơn, thực tốn vơ thú vị sâu sắc Bài tốn sau

Bài tốn Cho tam giác ABC cân tại A Phân giác góc ∠CAB ∠ABC cắt các cạnh BC CA lần lượt tại D E Gọi K là tâm nội tiếp tam giác ADC Giả sử rằng

∠BEK D45ı Tìm tất giá trị có góc∠CAB.

Bài toán đề nghị ba tác giả làJan Vonk,Peter Vandendriesschetừ nước Bỉ Hojoo Leetừ nước Hàn Quốc Bài tốn xếp vào vị trí thứ coi toán dễ ngày Đây coi tốn tính góc đẹp mắt Có nhiều lời giải đề nghị [1] Tôi xin giới thiệu lời giải coi đơn giản lấy theo ý tưởng đề nghị nick nameBlackMaxtrong [1]

B C

A

E

D K I

F

Lời giải. GọiI giao củaADvàBE tâm nội tiếp tam giácABC.F hình chiếu

I trênAC NếuF trùngE thìBE vừa đường cao vừa phân giác nến tam giácABC Trong trường hợp dễ thấy∠BEK D45ıthỏa mãn đề

NếuF không trùngE Dễ thấyF vàD đối xứng quaI C nên∠IFK D∠IDK D45ıD

(104)

KI D KE Vẫn từ tứ giácIFEK nội tiếp nênFK phân giác∠IFE Từ đóK tâm bàng tiếp tam giácIAF mà∠IKE D 90ı, ta suy ra∠IAE D 45ıhay tam giácABC vuông cân Trong trường hợp dễ thấy∠BEK D45ıthỏa mãn đề

Nhận xét.Theo ý tưởng lời giải ngắn gọn túy hình học đẹp Trong [1] cịn nhiều lời giải tính tốn khác Điểm thú vị tốn góc∠BAC nhận hai giá trị ứng với trường hợp tam giác tam giác vuông cân hai tam giác đặc biệt Chúng ta thấy trường hợp tam giác "tầm thường" trường hợp tam giác vng cân Vì ta phát biểu thành toán thuận sau

Bài toán Cho tam giácABC vuông cân tạiAvới phân giácAD; BE.K là tâm nội tiếp tam giácADC Chứng minh rằng∠BEK D45ı.

Mặc dù trường hợp tốn hồn tồn khơng khó mơ hình tốn lại có nhiều đặc điểm thú vị để khai thác

2 Mở rộng ứng dụng

Trong cách giải tốn trường hợp khiF khơng trùngE ta ý đến chi tiết thú vị làDvà

F đối xứng quaI C Nhờ ta có mở rộng thú vị sau

Bài tốn Cho tam giácABC có phân giácAD; BE.K là tâm nội tiếp tam giácADC Giả sử∠BEK D∠ADK Chứng minh rằngCB DCAhoặcDADDC.

A

B C

I

D

F

K E

Lời giải. GọiF đối xứng củaDquaI C NếuF trùngE dễ thấyIE; IDđối xứng qua

I C nênCB DCA

Nếu F khơng trùng E bốn điểm I; E; F; K thuộc đường trịn Nếu F nằm ngồi đoạnEC ta có 180ı D ∠IEK C∠EKI C∠KIE D

2∠ADC C∠IFAC∠CIE D

∠B C

∠A C

∠A

2 C∠C C

∠B C

∠C D

3∠A

4 C∠BC 3∠C

2 Ta suy ra∠A D 2∠C hay DADDC NếuF nằm đoạnEC ta có biểu thức góc tương tự Từ ta có điều phải chứng minh

(105)

Bài toán Cho tam giácABC nội tiếp.O/có phân giácAD; BE.K là tâm nội tiếp tam giác

ADC Giả sử tam giácDAC cân tạiD Chứng minh tứ giácBDKEnội tiếp.

Khi phát biểu tốn thuận tốn 2, ta có để ý rằngEKkhi qua điểmLđối xứng củaI quaBC mặt khác dễ thấy∠BALD 45ıD∠BELnên tứ giácBAELnội tiếp Ngay cho ta câu hỏi liệu ta thay tam giác vng cân để dẫn tới toán tổng quát tam giác vng Câu hỏi giải trọn vẹn toán sau

Bài toán Cho tam giácABC vuông tạiA Đường caoAH vớiH thuộcBC.I là tâm nội tiếp tam giácABC.BI cắt CAtại E.K là tâm nội tiếp tam giác AH C.KE cắtIH tại L. Chứng minh tứ giácBAELnội tiếp.

B

A

C H

K E

I

L

Lời giải. Từ hệ thức lượng tam giác vuông, ta dễ chứng minhCI:CK D CE:CH

từ 4CEK 4CIH suy ∠CEK D ∠CIH tứ giác CEIL nội tiếp Từ ta có

IL:IE CL:CE D

IK

KC suy IL CL D

IK KC:

CE

IE Ta chứng minh IL CL D

IB

AC đẳng thức tương

đương

IK KC:

CE IE D

IB AC

, CE

CK D IB AC:

IE

IK (Do4CEK 4CIH)

, CI

CH D IB AC:

IE

AI2=CI (Do4AIK 4CIA) , BC

AC2 D

IB:IE AI2:AC ,BC:AI2 DIB:IE:AC

Đẳng thức cuối ta dễ kiểm tra nhờ hệ thức lượng tam giác vuông Vậy kết hợp

∠BIL D ∠ACL suy ra4BIL 4ACLsuy ∠LBI D ∠LAC nên tứ giácBAEL nội tiếp Ta có điều phải chứng minh

Nhận xét.Ta nhận thấy hệ qua đơn giản từBAELnội tiếp thì∠BLED90ı Ta lại có∠ELC D∠EI C D45ı Suy ra∠BLC D90ıC45ı D135ı Lại từ∠BLC D135ıta dễ chứng minh tâm ngoại tiếp tam giácBLC trung điểm cungBCd chứaAcủa đường tròn ngoại tiếp tam giácABC

(106)

Bài toán Cho tam giácABC vuông tạiAvới đường caoAH và phân giácBE.K là tâm nội tiếp tam giácAHB Giả sử∠BEK D45ı, chứng minh tam giácABC vuông cân. Lời giải. GọiI tâm nội tiếp tam giácABC vàEKcắtIH tạiL Theo trước tứ giác

BAEL nội tiếp nên∠BAL D ∠BEL D 45ı D ∠BAI Từ đóALđi qua I hayA; H; L; I

thẳng hàng, tam giácABC vuông cân

Ngay câu hỏi lại đặt tốn phát biểu cho tam giác vng, với tam giác ? Ta có mở rộng thú vị tốn phần nhận xét cho tam giác sau, toán tham khảo [2]

Bài toán Cho tam giácABC có phân giácBE Đường trịn quaA; B tiếp xúcAC cắtBC

tạiD khácB.K là tâm nội tiếp tam giácADC.EKcắt đường tròn ngoại tiếp tam giácABE

tạiL Chứng minh tâm đường tròn ngoại tiếp tam giácLBC nằm đường tròn ngoại tiếp tam giácABC.

A

B C

I

E

D

K

L

S

J

Lời giải thứ nhất. GọiI tâm nội tiếp tam giácABC Đường tròn ngoại tiếp tam giácAIE

cắt CI J khác I Ta thấy CI:CJ D CE:CA Lại có 4CKA 4CIB nên CK

CI D CA

CB D CD

AC Từ CK:CJ D CK

CI :.CI:CJ / D CE:CD nên 4CEK 4CJD suy

∠CEK D∠CJD Ta định nghĩa lại điểmLlà giao củaDJ vàKE tứ giácCEJLnội tiếp Suy ra∠DLK D∠KCE D∠KCDsuy tứ giácDKCLnội tiếp Cũng từ4CKA 4CIB

nên ∠AKJ D ∠CIE D ∠CAJ Từ đóJA2 D JK:J C D JD:JL suy ra4JAD 4JLA

suy ra∠ALD D∠JADD∠DAC ∠JAC D∠ABC ∠AKJ D∠ABC

2.∠DABC

∠ACB/D

2.∠ABC ∠ACB/ Suy ra∠ALE D∠ALDC∠DLE D

2.∠ABC ∠ACB/C

2∠ACB D

2∠ABC D∠ABE Từ tứ giácABLEnội tiếp Từ dễ chứng minh∠BLC D 180ı

2∠BAC nên tâm ngoại tiếpScủa tam giácBLC trung điểmBCd chứaAcủa đường

(107)

A

B C

D

K

E

L M

N

F

Lời giải thứ hai. Gọi AK cắt BC F Ta có∠FAC D

2∠DAC D

2∠ABC D ∠CBE

vậy suy tứ giácAEFB nội tiếp DoBE phân giác∠ABC nênE điểm cung

AF nênLE phân giác∠ALBsuy LA

LF D KA KF D

CA

CF GọiCLcắt đường tròn ngoại tiếp

tam giácABE tạiLkhácM Dễ thấy tam giác đồng dạng góc góc4CEM 4CLAvà 4CBM 4CLF Từ ME

M C D LA CA D

LF CF D

MB

M C suy raLC phân giác∠BLEhay

∠BLC D ∠CLE D

2∠BAE GọiN điểm cung BC chứa Acủa đường tròn

ngoại tiếp tam giácABC NB D NC và∠BNC D ∠BAC D 2∠BLC Từ đóN tâm ngoại tiếp tam giácBLC

A

B D C

K E I

L

N M

Lời giải thứ ba. GọiI tâm nội tiếp tam giácABC.LC cắt đường tròn ngoại tiếp tam giác

(108)

∠ACB D∠AEBnênM nằm đường tròn ngoại tiếp tam giácABE Ta ýC; K; I

thẳng hàng, áp đụng định lý Pascal đảo cho

E N M A B L

suy raA; I; N thẳng hàng Từ

∠ELC D∠CAN D

2∠BAC vậy∠BLC D180

ı

2∠BAC nên tâm ngoại tiếpS tam

giácBLC trung điểmBCd chứaAcủa đường tròn ngoại tiếp tam giácABC

Nhận xét.Lời giải thứ tác giả đề nghị Ý tưởng lời giải dựa vào ý tưởng lời giải toán tác giả Lời giải thứ hai bạnNguyễn Lê Phướcđề nghị Lời giải thứ ba bạnLê Thị Hải Linhđề nghị Các lời giải phụ thuộc nhiều vào vị trí điểm hình vẽ gốc, có nét đẹp đặc trưng hình học túy Để tránh việc phụ thuộc hình vẽ bạn đưa vào biến đổi góc định hướng nhiên điều không thực cần thiết Bài toán toán toán tổng quát có nhiều giá trị ứng dụng, bạn làm tốn sau để ứng dụng tốn

Bài tốn Cho tam giácABC vuông tạiAvới đường caoAH Phân giácBE; CF cắt nhau tạiI GọiK; Llà tâm nội tiếp tam giácAH C; AHB.J đối xứng vớiI quaBC Chứng minh rằngEK FLcắt đường trịn ngoại tiếp tam giácJBC.

Bài tốn Cho tam giácABC vuông tạiAvới đường caoAH Phân giácBE; CF cắt nhau tạiI GọiK; Llà tâm nội tiếp tam giácAH C; AHB Chứng minh rằngEK; FLIH

đồng quy.

Bài toán 10 Cho tam giác ABC với đường cao AH. M; N thuộc BC sao cho AM ?

AB; AN ?AC.I là tâm nội tiếp tam giácAMN.NI; MI cắtCA; AB lần lượt tạiE; F Gọi

K; Llà tâm nội tiếp tam giácAHM; AHN Chứng minh rằngEK; FLIH đồng quy.

Bài toán 11 Cho tam giácABC với phân giácBE; CF Đường tròn quaA; Btiếp xúcAC cắt

BC tạiM khácB Đường tròn quaA; C tiếp xúcAB cắtBC tạiN khácC GọiK; Llà tâm nội tiếp tam giácACM; ABN.EK; FLlần lượt cắt đường tròn ngoại tiếp tam giác

ABE; ACF tạiP; QkhácE; F Chứng minh rằngP; Q; B; C cùng thuộc đường tròn.

Bài tốn 12 Cho tam giácABC nội tiếp.O/có phân giác AD; BE.K là tâm nội tiếp tam giácADC GọiEK cắtADtạiL.LB cắt đường tròn nội tiếp tam giácABE tạiN khácB. Giả sử tam giácDAC cân tạiD, chứng minh tâm ngoại tiếp tam giácNBC nằm trên.O/. Cuối cùng, tác giả xin gửi lời cảm ơn tới bạnNguyễn Tiến Dũngsinh viên K50 đại học ngoại thương người đọc toàn viết đưa góp ý giúp tác giả hoàn thiện viết

Tài liệu tham khảo

[1] IMO 2009, problem

http://artofproblemsolving.com/community/c6h289054p1562847

[2] Mỗi tuần toán: Tuần tháng năm 2015

(109)

TRONG XÂY DỰNG CẤU HÌNH TỔ HỢP

Trần Minh Hiền

(THPT Chuyên Quang Trung, Bình Phước)

Thuật tốn tham lam làtìm tối ưu địa phương bước đivới hy vọngtìm tối ưu tồn cục Dĩ nhiên thuật tốn tham lam không đảm bảo tối ưu địa phương cho ta lời giải tối ưu toàn cục Nhưng nhiều tốn, việc xảy Ví dụ:"Cho trước tập hợpS đồng xu Khi cần lấy đồng xu trongS để tổng số tiềnM cho trước KhiS D f1; 5; 10; 25g, áp dụng thuật tốn tham lam để xây dựng phương án tối ưu sau:

Thêm đồng xuxlớn trongS choxM; Áp dụng lại thuật tốn choM x

Ví dụ, vớiM D 91, ta chọn 25 S, sau lại tiếp tục áp dụng thuật tốn cho

91 25D66 Khi ta lại tiếp tục chọn số 25.Tiếp tục áp dụng thuật toán cho66 25D41, ta chọn số 25 Lại áp dụng thuật toán cho41 25D16, ta chọn số 10 Lại tiếp tục áp dụng thuật tốn cho16 10D6, ta chọn số Và cuối áp dụng thuật toán cho6 5D1, ta chọn số Từ tập hợp tối ưu làf25; 25; 25; 10; 5; 1g.Tức với tậpS D f1; 5; 10; 25gthuật toán tham lam cho ta phương án tối ưu Tuy nhiên thuật tốn tối ưu khơng phải lúc cũng cho ta phương án tối ưu Ví dụ vớiS D f1; 3; 4g M D 6, thuật toán tham lam cho ta tập

f1; 1; 4g, nhiên phương án tối ưu làf3; 3g." Dưới số toán minh họa

Câu 1(BMO 1998). Một đại lý vé tàu hỏa phân phối vé tàu hỏa cho 200 đại lý Trong ngày đặc biệt, có tất 3800 người 200 đại lý đến mua vé, người mua vé

1 Chứng minh có đại lý có số người đến mua vé ngày hơm

2 Ý khơng cịn thay số

Lời giải. Gọi s1; s2; : : : ; s200 số người đến đại lý thứ nhất, đại lý thứ hai,: : :, đại lý thứ 200 mua vé tàu hỏa ngày Khơng tính tổng qt, ta giả sử

s1 s2: : :s200

ĐặtS Ds1Cs2C Cs200thìS D3800 Bây ta tìm hiểuS nhỏ bao nhiêu khi điều kiện tốn khơng thỏa,tức khơng tồn dãysi; siC1; siC2; : : : ; siC5nào mã

si DsiC1 D: : :DsiC5 Rõ ràngS nhỏ khita lấy nhiều số nhỏ, số nhỏ không lấy Do đóS nhỏ

s1 Ds2D: : :Ds5 D1;

s6 Ds7D: : :Ds10 D2;

:::

(110)

Nhưng tổngS đạt

1C1C C1

„ ƒ‚ …

5số

C2C2C C2

„ ƒ‚ …

5số

C C40C40C C40

„ ƒ‚ …

5số

D4100:

Do đóS > 3800, điều mâu thuẫn Điều chứng tỏ phải tồn số hạng liên tiếp dãy Hay có đại lý có số người đến mua vé ngày hơm

2 Chúng ta xây dựng ví dụ thỏa mãn thuật tốn Với mỗi1i 198thì

si D

i 1

6

C1:

Khi đós1Cs2C Cs198 D3366 Khi chọns119 Ds200 D220 Khi đóS D3800 rõ ràng khơng có phần tử liên tiếp nhau, tức khơng có cửa hàng có số người đến mua vé Bài toán giải hoàn toàn

Câu 2(Iran 1997). Cho hai số nguyên dươngm; k Chứng minh tồn số nguyên dươngak > ak > : : : > a1 0sao cho

m D ak

k

!

C ak

k

!

C C a1

1

!

Tổng ak

k

!

C ak

k

!

C C a1

1

!

được gọi khai triển Macaulay củamứng vớid

Ví dụ khai triển Macaulay 17 ứng với

5 ! C ! C 1 !

D10C6C1D17;

trong khai triển Macaulay 16 ứng với

5 ! C ! C !

D10C6C0D16:

Lời giải. Đầu tiên ta chứng minh Giả sử m biểu diễn hai dãy

ak; : : : ; a1 vàbk; : : : ; b1 Khi hai dãy a1; : : : ; ak fb1; : : : ; bkg (theo thứ tự) phải có hai phần tử khác Ta gọi ví trí mà chúng khác nhau, khơng mt tớnh tng quỏt lk, tcak Ô bk Gi sak > bk mà khơng làm tính tổng qt đề Vìb1< b2 < : : : < bk < bk dãy số nguyên nên

bk bk 1; bk bk 1bk 2; : : : ; b1< bk kC1:

mD bk

k

!

C bk

k

!

C C b1

1

(111)

nên

m bk

k

!

C bk 1

k

!

C C bk kC1

1

!

:

Theo tính chất nhị thức

bk

k

!

C bk 1

k

!

C C bk kC1

1

!

< bk C1 kC1

!

)m < bk C1 k

!

:

Vìbk < ak nênbk C1ak Do

bkC1

k ! ak k ! : Dẫn đến

m < bk C1 k

!

ak

k

!

< ak k

!

C ak

k

!

C C a1

1

! m

vô lý Vậy điều phản chứng sai

2 Để chứng minh tồn tại, ta sử dụng thuật tốn tham lam sau: ta tìm sốak lớn nhấtsao cho

ak

k

! m:

Sau lại áp dụng thuật tốn trên, thay chomvàk, chom ak k

!

vàk Cứ tiếp tục quy trình ta dựng dãy thỏa mãn Dãyfa1; a2; : : : ; akglà dãy tăng dần vì, theo cách xây dựng nên

m < ak C1 k

!

)m ak k

!

< ak k

!

do tính chất nhị thức akC1

k

!

D ak

k

!

C ak

k

!

Nhưng lại theo cách dựng

ak

k

!

< m ak k

!

) ak

k

!

< ak k

!

)ak < ak:

(112)

Câu 3(Russian 2005). Trong bảng vng kích thước2n(nlà số ngun dương khơng nhỏ hơn2) người ta điền vào ô vuông số thực dương cho tổng hai số cột ln Chứng minh ta chọn cột số để tổng số chọn dịng khơng vượt nC1

4

Lời giải. Do đề u cầutổng số dịng khơng vượt quá nC1

4 , tức yêu

cầu tổng số dòng nhỏ tốt Từ ta nghĩ đến việcchọn mỗi cột số nhỏ nhấtthì khả thành cơng cao Tuy nhiên tất số nhỏ (trên cột) lại nằm dịng sao? Khi thuật tốn tham lam thất bại, với minh họa

0.4 0.4 0.4 0.4 0.6 0.6 0.6 0.6

thì ta chọn cột số nhỏ nhất, ta chọn tồn số 0.4 dịng Nhưng tổng số dòng là0:4n, vượt khỏi nC1

4

Ta cần cải tiến thuật toán chút Tức đảm bảo cho hai dòng cùng thỏa điều kiện lúc Tức mong muốn cách phải chọn đượcđồng thờicác số nhỏ có dịng số nhỏ có ở dịng thứ hai Bằng cách thay đổi thứ tự cột, ta xếp số dịng thứ nhất theo thứ tự khơng giảm, tổng hai số cột 1, nên dẫn đến số dòng thứ hai theo thứ tự không tăng Gọia1; a2; : : : ; anlà số dòng thứ mà

a1a2 : : :an

a1 a2 a3 an

1 a1 a2 a3 an

Với thứ tự này,thì dịng thứ ta chọn số từ trái qua phải, dòng thứ hai ta chọn số từ phải qua trái Bây ta chọni làchỉ số lớn nhấtsao cho

a1Ca2C Cai

nC1

(tức số chọn dòng thỏa mãn) Bây ta kiểm tra

nC1

4 aiC1/C.1 aiC2/C C.1 an/

D.n i / aiC1CaiC2C Can/ hay

aiC1CaiC2C Can

3n i:

Vìa1a2 : : : aiC1nên

a1Ca2C CaiC1

(113)

và doaiC1 aiC2 : : :annên

aiC1CaiC2C Can

n i aiC1:

Kết hợp hai bất đẳng thức ta có

aiC1CaiC2C Can

n i

a1Ca2C CaiC1

iC1

hay

aiC1CaiC2C Can.n i /

a1Ca2C CaiC1

i C1 > n i /

nC1

i C1

(bất đẳng thức cuối có doi số lớn thỏaa1Ca2C Cai

nC1 )

Cuối ta cần chứng minh

n i iC1

nC1

3n

4 i ).n 1/

2

4i.n i 1/:

Điều có bất đẳng thức Cauchy

4i.n i 1/4 i Cn i 1/

4 D.n 1/

2

:

Đến tốn chứng minh hồn tồn

Câu 4(IMO 2003). ChoAlà tập chứa 101 phần tử tậpS D f1; 2; : : : ; 1000000g Chứng minh tồn phần tửt1; t2; : : : ; t100trongS cho tập

Aj D fxCtjjx2 Ag; j D1; 2; : : : ; 100 rời đôi

Lời giải. Ta mong muốn tìm thuật tốn xây dựngt1; t2; : : : ; t100 Giả sử tay ta cót1, thìt2phải chọn cho (theo giả thit)

xCt1 ÔyCt2;8x; y 2A)t2Ôt1Cx y;8x; y 2A:

Để đảm bảo dãy ftig phân biệt ta xây dựng dãyftig tăng dần Từ ta cần t2 ¤

t1C jx yj;8x; y Athì đảm bảo tính tăng dần Nhưng chọn đượct2thì phải chọnt3; : : : ; t100 Do chọnt2phải vừa đảm bảo "đủ xa",tức khác tất các giá trị trước đó, phân tích trên, vừa đảm bảo "đủ gần",để xây dựng tiếp chot3; : : : ; t100 Từ ý tưởng xây dựngt2 hình thành:

Trên trục số ta biểu diễn phần tử củaS theo thứ tự tăng dần

Chọn ngayt1 D S, ta tô màu số t1 tất số dạng:ft1C jx yj D

1C jx yjj8x; y Ag trục số (các giá trị hiệujx yj trùng với cặp số.x; y/khác A, nên bước ta đánh dấu không

1C 101

2

!

(114)

Sau bước này, ta chọnt2 làsố nhỏ trục số mà chưa bị tô màu Tiếp tục ta lại tô màu sốt2và số trục số dạng:ft2C jx yjjx; y 2Ag Ở bước vài số tô màu bước Khi trục số, sau bước này, có tối đa25051số trongS tô màu Với cách xây dựng thìt2 > t1

Cứ tiếp tục thuật tốn trên, sau xây dựng sốt1; : : : ; t99 tơ màu tối đa500049phần tử trongS trục số Vì500049 < 1000000nên ta tiếp tục xây dựng sốt100 theo thuật toán Vậy sốt1 < t2< : : : < t100được xây dựng

Bây ta kiểm tra lại, với cách xây dựng t1; t2; : : : ; t100 thuật toán trên, tập

Aj; Ak rời đôi với mọi1j < k 100 Thật vậy, giả sử ngược lại, tức tồn tạixCtj DyCtk vớix; y 2Anào Do cách xây dựng đảm bảotk > tj Do

x > y Điều dẫn đến

tk Dtj Cx y Dtj C jx yj;

vơ lý vìtk chọn bước thứkthì phải khác tất số tơ màu trục số: fti C jx yj W 8i D1; 2; : : : ; k 1g VậyAi\Aj D ; Bài tốn chứng minh hồn tồn

Câu 5. ChoAlà tập hợp số nguyên dương thỏa mãn: với phần tửx; ythuộcAthì

jx yj xy

30:

HỏiAchứa nhiều phần tử?

Lời giải. Ta nhận thấyx; y số lớn Bắt đầu với tậpA D f1g, áp dụng thuật toán tham lam, lần thử tìm phần tử nhỏ nằm trongA Quá trình ta thu đượcAD f1; 2; 3; 4; 5; 6; 8; 11; 18; 45gvà sau trình khơng thể thêm phần tử vàoA Từ đódự đốnAcó nhiều 10 phần tử ĐặtA D fa1; a2; : : : ; ang Ta chứng minh n 10 Khơng tính tổng qt, giả sử

1a1 < a2 < an Khi

aiC1

aiC1ai

30 )

1 aiC1

1

30;81i n 1:

Tổng tất đánh giá vớii D5; 6; : : : ; n 1ta

1 a5

1 an

n 30 )

1 a5

n 30 :

Vìai i;8i D5; : : : ; nnên từ đánh giá ta có

5a5<

30

n )n10:

(115)

Câu 6(IMO 2014). Một tập hợp đường thẳng mặt phẳng gọi "tốt" khơng có hai đường thẳng chúng song song ba đường thẳng chúng đồng quy Tập hợp đường thẳng "tốt" chia mặt phẳng thành miền, với số miền có diện tích hữu hạn gọi miền hữu hạn Chứng minh vớinđủ lớn, với tập hợpnđường thẳng "tốt" ta ln tơ màupnđường thẳng màu xanh để khơng có miền hữu hạn có tồn biên màu xanh

Lời giải. (Dựa theo lời giải GS Nguyễn Tiến Dũng) Ta thử làm theo kiểu thuật tốn, tức tìm thuật tốn tơ màu xanh đường Tại lại số bậc hai, chút xíu rõ Thuật tốn đơn giản sau:

Đầu tiên tô đường tùy ý màu xanh (hiển nhiên có thơi chưa thể vây tồn miền nào)

Tiếp theo chọn đường để tô: chọn thêm đường để tô xanh (mà không phạm luật bài) tơ, cịn đến khơng cịn đường dừng

Giả thuật tốn dừng lại sau tô đượckđường Bây phải chứng minh lànk2 Hay ta chứng minh ngược lại: nếun > k2 tơ tiếp

Nếun > k2thì số đường cịn lại chưa tơ lớn hơnk2 kDk.k 1/, tức lớn lần số điểm nút xanh (điểm nút xanh = điểm giao của2đường xanh) Gọi đường (trong số đường cịn lại đó) làđường cấmnếu mà tơ thêm đường màu xanh tạo miền bị chặn với biên toàn màu xanh Bây cần chứng minh số đường bị cấm không vượt qk.k 1/, dẫn đến có nhất1đường khơng bị cấm, tơ xanh Để chứng minh điều đó, ta tìm cách lập mối quan hệ đường bị cấm với nút xanh, cho tất đường cấm ứng với nhất1nút, ngược lại nút ứng với khơng q2đường cấm Nếu có quan hệ số đường cấm khơng vượt q2lần số nút) Quan hệ nào?

Một quan hệ hiển nhiên là: đường cấm tức tạo nhất1miền cấm(miền có1cạnh biên đường đó, cạnh cịn lại xanh) Lấy miền cấm đó, lấy2cái nút xanh là2đỉnh miền cấm mà kề sát đường cấm.2nút trùng miền cấm tam giác Để phân biệt, thay vị đặt quan hệ với nút , ta đặt quan hệ vớiđoạn cấm: gồm nút cạnh xanh miền cấm từ nút chạm vào đường cấm.Như vậy, đường cấm ứng với nhất2đoạn cấm

Ngược lại, nút cho nhiều nhất4đoạn cấm, từ nút có4nửa đường thẳng xanh, nửa đường thẳng có nhiều nhất1đoạn cấm xuất phát từ nút (khơng thể có2đoạn cấm đè lên từ1 nút xanh) Như vậy, quan hệ mà theo1chiều 2, theo chiều ngược lại thì4, số đường cấm2lần số nút xanh

Câu 7(IMO 1983). Chứng minh chọn2048số nguyên dương phân biệt, tất số nhỏ bằng100000;sao cho khơng có ba số hạng tập tạo thành ba phần tử liên tiếp cấp số cộng

Lời giải. Ở có trực giác là: số 100.000 dường nhưkhơng thích hợpvà q lớn Do làm suy nghĩgiải toán với số nhỏ mở rộng Chúng ta chắc chắn nên bắt đầu với việc xây dựng dãy nhỏ thuật tốntham lam.

(116)

khơng thể thêm vào tạo thành cấp số cộng1!2!3; Chúng ta thêm 4, vào dãy

lại khơng thể thêm vào dãy được, tạo thành cấp số cộng2!4!6 không thểm thêm vào dãy tạo thành cấp số cộng3!5!7 khơng thể thêm vào dãy tạo thành cấp số cộng2!5!8 khơng thể thêm vào dãy tạo thành cấp số cộng1!5!9

Chúng ta thêm 10, 11, lại thêm 12 Thêm 13, 14, tiếp tục ta thêm vào số gần 28,29

1 Ta dừng lại phân tích có số khoảng trống xuất hiện:2!4; 5!10; 14!

28 Tức số gấp lần xuất

2 Ngoài cịn có4 !10 ! 28, điều viết lại31C1 ! 32C1 ! 33C1 Từ ta nhận thấysố đóng vai trị quan trọng, đặc biệt lũy thừa 3. Điều gợi ta đến việc xây dựng dãy số theo số 3, trừ từ số Do dãy ta

S D1C f03; 13; 103; 113; 1003; 1013; 1103; 1113; 10003; : : : ; 111111111113g gồm tất số số gồm hai chữ số

Ta cójSj D211 D2048số

Phần tử lớn trongS là1C111111111113 < 100:000;

Gọix; y; zlà ba phần tử phân biệt tùy ý trongS, giả sửxCy D2z Do2z gồm hai chữ số Mà có phân tích nhất0C0D0; 1C1D2nênx; yphải có chữ số trùng vị trí, tứcx Dy, vơ lý Tức ba phần tử tùy củaS ba phần tử liên tiếp cấp số cộng

Câu 8. Chứng minh với số nguyên dươngn, ln biểu diễn dạngduy nhất thành tổng số lũy thừa phân biệt của2

Phân tích: Đầu tiên ta chứng minh biểu diện dụng thuật tốn tham lam Ta chọn lũy thừa lớn nhất, gọi là2k, cho2k n NếunD2k thì tốn kết thúc Ngược lại, ta áp dụng thuật toán cho sốn 2k Chắc chắn lũy thừa của2sẽ phân biệt bước chọn, theo cách xây dựng cho2k n < 2kC1, dẫn đếnn 2k < 2k Do bước ta chọn được2k 1:2k n 2k thì2k < 2k Tuy nhiên ta viết lời giải viết dạng quy nạp.

Lời giải. VớinD1, ta có1D20

Giả sử toán cho với sốnmà1n < 2k, vớik Ta chứng minh toán cho với mọinmà1n < 2kC1 Như ta biết khoảng1n < 2k, biểu diễn củansẽ có số hạng lớn là2k Cộng thêm vào biểu diễn cho số2k, ta biểu diễn cho sốnmà

(117)

Ngoài ranD 2k thân biểu diễn lũy thừa Do đó, tốn cho với mọi1n < 2kC1 Để chứng minh nhất, theo hướng quy nạp, ta phải chứng tỏ với mỗi2k n < 2kC1,phải có2k trong biểu diễn nó Thật vậy, giả sử ngược lại, biểu diễn củan, tổng lớn

n20C21C22C C2k D2k 1 < n;

mâu thuẫn

Câu 9. Cho A1; A2; : : : ; An tập phân biệt f1; 2; : : : ; ng jAij D 3;8i D

1; 2; : : : ; n Chứng minh ta tơ màu 2n

3

phần tử củaf1; 2; : : : ; ngsao cho tập

Ai cóít nhấtmột phần tử tơ màu

Phân tích: Ta thực tơ màu phần tử trongf1; 2; : : : ; ngtheo thuật toán tham lam như sau: Mỗi bước, ta tô màu phần tửxthuộc nhiều tập tốt Giả sửxthuộc vào tập

A1; A2; : : : ; Ak Đến bước thứ hai ta loại bỏ phần tử thuộc vào tậpA1; A2; : : : ; Ak, rồi ta lại tiếp tục tô màu phần tửy thuộc nhiều tập lại tốt, tiếp tục ta sẽ có điều phải chứng minh.

Lời giải. GọiAlà tập hợp tập số tậpA1; A2; : : : ; Anmà tập chưa có phần tử tơ màu sau thực hiệnklần thuật toán tham lamở Ta lưu ý sau thực hiệnkbước, tất tập hợp trongAđều "rời nhau" Khi rõ ràng thuật toán dừng thực tiếpkC jAjbước (vì tập hợp trongAphải tơ màu phần tử, tập rời nhau) Ta ý rằng:

k n

2 bước thực thuật tốn, số tập hợp giảm hai lần klần thực thuật toán đầu tiên, ta tập trung vào tô màu phần tử thuộc vào hai tập trở lên

jAj n k

3 sau thực thuật tốnklần, ta tơ màuk phần tử, tức

loại khỏi làkphần tử củaf1; 2; : : : ; ng, lại nhiều nhấtn kphần tử lại chia thành tập rời kích thước3:

Do

kC jAj kC n k

3 D n C

2k

n C

n D

2n

3 )k 2n

3 :

Vìkngun dương, chứng tỏ thuật tốn kết thúc ta thực nhiều 2n

3

lần Bài toán chứng minh

Câu 10(China TST 2015). ChoX tập khác rỗng vàA1; A2; : : : ; Anlàntập củaX cho

1 jAij 3;8i D1; 2; : : : ; n;

2 Bất kỳ phần tử củaX nằm tập sốA1; A2; : : : ; An

Chứng minh chọn

3n

(118)

Lời giải. Kết luận toán yêu cầu chọn số tập hợp, hợp lại bằngX, ta

1 Chọn tập có phần tử, giả sử A1;jA1j D 3 Sau đó, ta chọn tiếp tập A2 jA2j D3A2\A1 D ; Sau chọn tậpA2, ta chọn tiếp tậpA3jA3j D3

A3\A1D ;; A3\A2 D ;, tiếp tục đến chọn thêm tập nào nữa vào hệ.Trong tất cách cách lựa chọn tập hợp trongA1; A2; : : : ; An, ta chọn hệ tập hợpS3cực đại, giả sửS3 D fA1; A2; : : : ; Aig(i n) (tức họS3chứa nhiều tập hợp có) màjAtj D3;8t D1; 2; : : : ; i vàAr \As D ;;81r < si (điều có nghĩa, lần bổ sung tập hợp vàoS3thì tập X3 có số lượng phần tử tăng lên 3)

ĐặtX3 D [

Ar2S3

Ar Do tính tối đại tậpS3, nên với tập hợpAj (j > i)

jAj \.XnX3/j

vì khơng ta tiếp tục bổ sungAj vào tậpS3, mâu thuẫn với tính tối đại củaS3 Và

jX3j D3i:

2 Bây ta tiếp tục chọn họS2 cực đạichứa tậpAj lại sốAiC1; : : : ; An, chomỗi lần thêm tập hợp vào họS2, số lượng phần tử hợp chúng

tăng lên 2 Khơng tính tổng qt, giả sử

(119)

Đặt

X2 D [

Ar2S2

Ar\.XnX3/

thì theo cách xác định củaS2 ta cójX2j D2j theo tính tối đại tậpS2thì jAt \.Xn.X2[X3//j 1;8t Dj C1; : : : ; n:

3 Bây ta tiếp tục chọn họS1chứa tậpAscòn lại số

AjC1; AjC2: : : ; An;

sao cho mỗi lần thêm tập hợp vào họ S1, số lượng phần tử hợp của

chúng tăng lên 1dĩ nhiên tập hợp họS1chứa hết tất phần tử của

Xn.X3[X2/DX1 Không tính tổng quát, giả sử

S1D fAjC1; AiC2; : : : ; Akg:

4 Khi đójXj D jX1j C jX2j C jX3j D3i C2j Ck,X DX1[X2[X3và jS3j C jS2j C jS1j DiCj Ck Dm:

Ta cần chứng minhm 3n

7

Vì phần tử trongX1nằm tập hợp, dojAr\X1j 1;8r D

j C1; : : : ; nnên

niCj C4k: 1/

Mỗi phần tử trongX1[X2xuất tập hợp, dojArj\.X1[X2/

2;8r Di C1; : : : ; nnên

niC 4.2j Ck/

2 Di C4j C2k: 2/

Mỗi phần tử trongX xuất tập hợp, jAr \Xj 3;8r D

1; 2; : : : ; n,

n 4.3iC2j Ck/

(120)

Lấy20.1/C12.2/C27.3/ta có

59n 140.i Cj Ck/D140m)m 59n

140 < 3n

7 :

Bài tốn chứng minh hồn tồn

Câu 11(IMO 2014). Đồng xu gọi "may mắn" giá trị

n n2Z

C

Một

sưu tập đồng xu “may mắn” có tổng giá trị khơng q99C

2 Chứng minh chia

các đồng xu vào 100 nhóm cho nhóm có giá trị khơng q

Lời giải. Ta chứng minh toán tổng quát hơn: "Giả sử tổng giá trị đồng xu không

N

2 ln chia vàoN nhóm cho nhóm có tổng giá trị khơng q 1."

Giả sử sưu tập có đồng xu mệnh giá

1;1 2I

1 3I: : : ;

1

m (có thể có nhiều đồng xu mệnh giá):

Bước 1.Ghép đồng xu mệnh giá: Ta ghép đồng xu có mệnh giá lại thành một đồng xu (đồng xu "may mắn") , ghép khơng thể ghép dừng lại

Chẳng hạn lúc đầu tay ta có tập đồng xu may mắn sau

S D

1I1

2I 2I 2I 3I 3I 3I 3I 4I 4I 6I :

Ta ghép hai đồng xu

6 thành đồng xu

3, đồng xu

4 thành đồng xu

2 Khi ta

có năm đồng xu

3, bốn đồng xu

2, đồng xu

Tiếp tục ghép năm đồng xu

3 đồng xu 1, hai đồng xu

3 bốn đồng xu ghép

lại thành hai đồng xu

Cuối có

S0 D 1

3I

3I1I1I1

không ghép tiếp được, tổng không đổi so vớiS Kết thúc q trình ghép đồng xu ta có:

Cóp đồng xu mệnh giá

Nếukchẵn cịn lại nhiều đồng xu mệnh giá

k k > 1/

Nếuklẻ cịn lại nhiều nhấtk 1đồng xu mệnh giá

(121)

Khi đópđồng xu mệnh giá ta chia vềp nhóm cịn đồng tiền mệnh giá khác có tổng khơng qN

2 p ta chia vàoN pnhóm sau (với ý: với mỗik 2Z

C thì đồng

tiền mệnh giá

2k nhiều đồng đồng tiền mệnh giá

2k nhiều 2k 1/ 1đồng, trừ trường hợpkD1)

Bước 2.Chia đồng tiền mệnh giá lớn bằng

2.N p/vàoN pnhóm Với các

đồng tiền mệnh giá

2kI

2k 1,k D1I2I: : :IN pta chia nhómGk k D1I2I: : :IN p/

thì tổng số tiền nhiều nhómGk

.2k 2/: 2k C

1 2k < 1:

Nếu đồng tiền mệnh giá nhỏ

2.N p/ tiến hành bước sau:

Bước Ta thấy tổng số tiền chia vàoN p nhóm khơng lớn hơnN

2 p, nên tồn

tại nhóm có tổng số tiền nhỏ

1 N p

N p

D1 2.N p/:

Ta lấy đồng xu có mệnh giá nhỏ

2.N p/ bỏ vào nhóm nhóm có tổng

số tiền khơng vượt q

Vì số đồng tiền hữu hạn nên sau số lần kết thúc

Ta kết thúc chuyên đề ví dụ, cho thấy thuật tốn tham lam khơng cho ta tối ưu toàn cục, nhiên chúng xấp xỉ với nhau

Câu 12(IMO Shortlist 2014). Cho dãy số thựcx1; x2; : : : ; xn, gọi "giá" dãy số cho đại lượng

max

1injx1Cx2C: : :Cxij:

Cho trước số thựcn, Dave George muốn xếp thành dãy có “giá” nhỏ Chăm nên Dave kiểm tra tât cách tìm “giá”Gnhỏ Theo cách khác, George chọnx1sao chojx1jnhỏ nhất, số số lại chọnx2sao chojx1Cx2j nhỏ nhất, tiếp tục đến bước thứi chọnxi số số lại cho jx1Cx2C: : :Cxijnhỏ nhất, bước có số hạng chọn Cuối nhận dãy “giá”D Tìm sốc nhỏ cho với số ngun dươngnta ln có:

G cD:

Lời giải. Xét dãy số gồm số thực:1; 2; 1; Khi ta có xếp Dave George sau

(122)

GeorgeW1; 1; 2; 1; 1; 2; 2:Khi đóWG D2:

Từ ta thu được: c Ta chứng minhc D giá trị cần tìm, tứcG 2D ˙Giả sử

x1; x2; : : : ; xnlà dãy số ban đầu, d1; d2; : : : ; dn vàg1; g2; : : : ; gn dãy mà Dave George thu Đặt

M D max

1injxij; S D jx1Cx2C: : :Cxnj: Nhận xét:D S (hiển nhiên cách xác địnhD) Giả sửjdij DM ta có:

M D jdij D j.d1Cd2C: : :Cdi/ d1Cd2C: : :Cdi 1/j jd1C: : :Cdij C jd1C: : :Cdi 1j 2D: Do để chứng minhG 2D ta cần chứng minh

G maxfM; Sg:

ĐặtN DmaxfM; Sgta chứng minhG N Đặthi Dg1C: : :Cgi Khi

G N , jhij N; 8i D1; n: Ta chứng minh:jhij N quy nap Vớii D1ta có

jh1j D jg1j M N:

Giả sửjhi 1j N ta chứng minhjhij N Ta có

jhij D j.g1C: : :Cgi 1/Cgij D jhi 1Cgij:

Nếu số gi; giC1; : : : ; gn có số khác dấu nhau, giả sử j i số mà

hi 1:gj Khi jhi 1Cgij

ˇ

ˇhi 1Cgj ˇ

ˇmax ˚

jhi 1j; ˇ ˇgj

ˇ

ˇ N ) jhij N:

Nếu sốgi; giC1; : : : ; gncùng dấu, không tổng quát giả sử chúng số dương Khi đó, ta có

hi 1hi hiC1 : : :hn ) jhij maxfjhi 1j; jhnjg N ) jhij N

với8i D1; ntức làGN DmaxfM; Sg 2D Vậyc D2là giá trị cần tìm

Sau số tốn để đọc tham khảo, rèn luyện thêm

Câu 13(IMO Shortlist 2014). Chonlà số nguyên dương Tìm số nguyên dươngk nhỏ thỏa mãn: cho trước số thựca1; a2; : : : ; ad có

a1Ca2C Cad Dnvà0ai 1;8i D1; 2; : : : ; d

(123)

Câu 14 (USA TST 2003). Với cặp số nguyên dương a b với < a < b < 1000, tập

S f1; 2; : : : ; 2003gđược gọi "tập bỏ qua" cho.a; b/nếu với cặp phần tửs1; s22 S js1 s2j 62 fa; bg Đặtf a; b/là tập có kích thước lớn cho cặp.a; b/ Xác định giá trị lớn nhỏ hàmf x; y/, với0 < x < y < 1000

Câu 15. Chứng minh với số hữu tỉ dươngx, ln tìm số ngun dương phân biệta1; a2; : : : ; ansao cho

x D

a1 C

1

a2 C C

1 an

:

Câu 16 (IMO Shortlist 2001). Một ba phần tử số nguyên không âm x; y; z/ với

x < y < zđược gọi "đẹp" nếufz y; y xg D fa; bgvới0 < a < blà số nguyên dương cho trước Chứng minh tập hợpNcó thể viết thành hợp rời ba "đẹp"

Câu 17. Cho dãy an/ gồm số tự nhiên thỏa mãn: không tồn số i; j; k/ mà

i < j < k thìai < aj < ak Chứng minh phân hoạch dãy.an/thành hai dãy tăng

Tài liệu tham khảo

[1] Algorithms, Cody Johnson, Mathematical Reflection volume 4, 2015

(124)(125)

Lưu Bá Thắng

(Khoa Toán Tin, Đại học Sư Phạm Hà Nội)

Chúng ta biết dãy số nguyên tố2; 3; 5; 7; :::là vô hạn Để kiểm tra khoảng cách hai số nguyên tố liên tiếp không bị chặn, tức với số nguyên dươngktùy ý, ln tồn hai số ngun tố kề có hiệu vượt quák Để chứng minh điều này, ta cần tồn tạiksố tự nhiên liên tiếp không số nguyên tố Thật vậy, ta đặtN D2:3:5::::plà tích tất số ngun tố khơng vượt qkC2thì không số dãy gồmksố nguyên liên tiếp

N C2; N C3; :::; N C.kC1/

là số nguyên tố Nhưng có chặn cho khoảng cách dãy số nguyên tố Năm 1845, Bertrand đưa giả thuyết, thường gọi Định đề Bertrand (Bertrand’s postulate):

Cho số ngun dươngn1, ln có số ngun tốpvớin < p2n

Bertrand chứng minh giả thuyết chon < 3000000 Năm 1950, Tchebychev người đưa chứng minh giải tích cho định đề Năm 1932, Paul Erdos đưa một˝

chứng minh đẹp cho Định đề Bertrand dùng kiến thức Tốn sơ cấp, ơng 19 tuổi Tư tưởng Erdos việc chứng minh định đề Bertrand việc ước lượng giá˝

trị 2n

n

!

Trong viết này, giới thiệu đến bạn học sinh, sinh viên bạn

trẻ yêu Toán chứng minh Erdos vài số kết gần vấn đề này.˝

Ta thấy rằng, vớin2thì

2n n

!

n

2n: (0.1)

Thật vậy, 2n

n

!

là số hạng lớn trong2nsố hạng 2n

0

!

C 2n

2n

!

; 2n

!

; :::; 2n 2n

!

tổng

2n X

iD0

2n i

!

D.1C1/2n D4n:

Erdos chứng minh khơng có số nguyên tố˝ p chon < p2nthì suy

2n n

!

<

n

2n trừ số giá trịnnhỏ Điều chứng minh Định đề Bertrand cho sốnđủ

lớn Đối với giá trịnnhỏ, Erdos kiểm tra trực tiếp.˝

Để chứng minh ý tưởng trên, cần số bổ đề sau:

Bổ đề Định đề Bertrand vớin < 4000.

Chứng minh. Để chứng minh điều này, ta không cần kiểm tra tất cả4000trường hợp mà cần kiểm tra,

(126)

là dãy số nguyên tố mà số đứng sau không vượt quá2lần số đứng đằng trước Do đó, khoảngfm Wn < m2ngvớin < 4000đều chứa đựng trong14số nguyên tố dãy

Bổ đề (Định lý Legendre) Cho số nguyên tốp và số tự nhiênn Ta kí hiệuvp.n/là số mũ lớn củap sao chopvp.n/ jn, đó

vp.nŠ/D X

i1

Œ n pi:

Chứng minh. Trước hết ta thấy tổng thực chất gồm hữu hạn số hạng vớii đủ lớn

pi > nnên n

pi

D0:Mặt khác, tíchnŠcó n

p

thừa số bội củap Do ta viết

nŠDp

Œn

pn p

Šq1 đó.q1; p/D1:

Tương tự,

n

p

ŠDp

2 6 6 Œn p p 7 7 Œn p p

5Šq2trong đó.q2; p/D1:

Dễ kiểm tra Œn p p 5D n p2 nên n p

ŠDp

2 n p2 n p2

Šq2:

Suy

nŠ Dp

2 n p 5C n p2

5 n

p2

Šq1q2trong đó.q1; p/D1:

Tiếp tục q trình ta thu

nŠDp

2 n p 5C n p2 5C n p3 5C

Atrong đó.A; p/ D1:

Vậy số mũ số nguyên tốptrong phân tích tiêu chuẩn củanŠlà

p.nŠ/D X

i>1 n pi D n p C n p2 C n p3

(127)

Bổ đề Nếup j 2n

n

!

thìpvp 2nn//2n:

Chứng minh. Đặtr.p/là số tự nhiên thỏa mãnpr.p/ 2npr.p/C1 Ta có

vp

2n n

!

/D vp 2n/Š/ 2vp.nŠ/

D r.p/ X

iD1

Œ2n pi

r.p/ X

iD1

Œ n pi

D

r.p/ X

iD1

.Œ2n pi 2Œ

n pi/

r.p/;

và đópvp 2nn// pr.p/ 2n:

Từ Bổ đề ta thấy số nguyên tốp >p2nxuất phân tích 2n

n

!

nhiều

1 lần Hơn nữa, số nguyên tốp thỏa mãn 2n

3 < pnsẽ không ước 2n

n

!

vớin3:

Bổ đề Với số thựcx2, ta có

Y

px

p 4x (0.2)

trong tích chạy qua tất số nguyên tốp.

Chứng minh.

Trước hết ta ý nếuqlà số nguyên tố lớn thỏa mãnq xthì Y

px

p D Y pq

p và4q 14x 1:

Do ta cần chứng minh cho trường hợpxDq số nguyên tố quy nạp Vớiq D2, (0.2) hiển nhiên Bây ta xem xét (0.2) cho trường hợp số nguyên tố lẻq D2mC1 Giả sử (0.2) cho tất số tự nhiênx2m Choq D2mC1, ta có

Y

p2mC1

p D Y pmC1

p Y

mC1<p2mC1

p:

Ta có

Y

mC1<p2mC1

p 2mC1

m

(128)

vì 2mC1/Š

mŠ.mC1/Š số tự nhiên tất số nguyên tố vế trái ước tử số.2mC1/Š

nhưng ước mẫu sốmŠ.mC1/Š Lại có

2mC1 m

!

D 2mC1

mC1

!

2mC1 X

kD0

2mC1 k

!

D22mC1

nên 2mC1

m

!

22m Do đó,

Y

p2mC1

p 4m 2mC1 m

!

4m22mD42m:

Bây giờ, ta chứng minh Định đề Bertrand Trước hết ta đánh giá 2n

n

!

:Từ (0.1) Bổ đề 4, vớin3, ta có

4n 2n 2n n ! Y pp2n

2n Y

p

2n<p2n3

p Y

n<p2n

p;

hay

4n.2n/1C

p

2n Y

p

2n<p2n

p Y

n<p2n

p:

Kết hợp với Bổ đề 0.2, ta

4n.2n/1C

p 2n

42n3 Y n<p2n

p;

hay

4n3 2n/1C p

2n Y

n<p2n

p: (0.3)

Giả sử tồn số nguyên dươngnsao cho khơng có số ngun tốp thỏa mãnn < p 2n Theo Bổ đề 2, ta suy ran4000 Từ (0.3), ta có Y

n<p2n

p D1và

4n3 2n/1C p

2n

; (0.4)

hay

22n 2n/3.1C

p 2n/

: (0.5)

Dùng bất đẳng thứcaC1 < 2a; a2, ta có

2nD.p6 2n/6 < Œp6 2nC1/6 < 26Œ6

p 2n

266

p 2n;

22n.2n/3.1C

p 2n/

< 26

p

2n.18C18p2n/

< 2206

p 2np2n

D220.2n/2=3

vớin4000:Suy ra.2n/1=3 < 20hayn < 4000(mâu thuẫn) Vậy ta chứng minh xong Định đề Bertrand

(129)

Hệ Có sốc; C > 0sao cho với số dươngx, ta có

clnx

x x/C

lnx x

trong đó.x/là số số ngun tố khơng vượt quáx.

Việc chứng minh hệ tập dành cho bạn Một định lý tiếng ước lượng hàm.x/

lim x!C1

.x/ x=lnx D1

được chứng minh lần Hadamard de la Vallée-Poussin năm 1896 với công cụ từ giải tích phức Dùng Định lý này, chứng minh Định đề Bertrand Câu chuyện Định đề Bertrand chưa kết thúc có nhiều câu hỏi tương tự đặt Gần đây, năm 2006 El Bachraoui [2], đưa chứng minh sơ cấp cho việc ln có số nguyên tố nằm giữa2nvà3nvới số nguyên dươngn > Một kết tương tự năm 2011 Loo [5] chứng minh cho việc tồn số nguyên tố nằm giữa3nvà4n:Một câu hỏi đặt cho việc chứng minh toán tổng quát: Chok; nlà số ngun dương,k nvà

n > 1, ln tồn số nguyên tố nằm đoạnŒk nI.kC1/n Nếu trả lời câu hỏi này, có lời giải cho vấn đề tiếng mà chưa có lời giải: Giả thuyết.Ln tồn số nguyên tố giữan2.nC1/2với số nguyên dươngn? Sau số toán liên quan đến vấn đề mà đề cập

Bài tập ([4]) Cho số nguyên dươngn, tậpf1; 2; : : : ; 2ngcó thể chia thành cặp

fa1; b1g; : : : ;fan; bng

sao cho với mỗi1i n,ai Cbi là số nguyên tố.

Chứng minh. Ta chứng minh quy nạp theon VớinD1, kết tầm thường Vớin > 1, đặt

p số nguyên tố thỏa mãn2n < p 4n Từ4nkhông số nguyên tố nênp D 2nCmvới

1m < 2nvàmlà số lẻ Rõ ràng tậpfm; : : : ; 2ngđược chia thành2n mC1

2 cặp

f.m; 2n/; mC1; 2n 1/; : : : ; nC m

2 ; nC

mC1 /;

sao cho tổng cặp số nguyên tốp Theo giả thiết quy nạp, rõ ràng tậpf1; 2; : : : ; m 1g chia thành m

2 cặp cho tổng cặp số nguyên tố Từ suy điều phải

chứng minh

Bài tập Chứng minh với số nguyên dươngk;luôn tồn dãy số nguyên dương fa1; a2; :::; a2kgsao cho chia chúng thànhk cặp phân biệt mà tổng cặp số nguyên tố.

(130)

Bài tập Chop là số nguyên tố và

nDakpk Cak 1pk 1C Ca1pCa0

là khai triển củantrong sốp:Ký hiệusp.n/ Dak Cak 1C Ca0;chứng minh rằng

p.nŠ/D

n sp.n/

p :

Chứng minh. Áp dụng Định lý Legrendre, ta có

vp.nŠ/D X

i1

Œ n piDŒ

n pCŒ

n

p2 CŒ

n pk

D.akpk 1C Ca2pCa1/C.akpk 2C Ca2/C Cak Da1Ca2.1Cp/C Cak.pk 1C CpC1/

D n sp.n/

p :

Từ tập trên, ta thấy với số nguyên dươngnthìsp.n/1nên ln có bất đẳng thức: Với số ngun dươngn

p.nŠ/

n p 1:

Đẳng thức xảy khinDpr:

Từ đó, áp dụng bất đẳng thức vớip D2, dễ dàng giải toán sau:

Bài tập Với số nguyên dươngn, ta có2n nŠ

Bài tập Với số nguyên dươngn, ta có2n jnŠnếu nếunlà lũy thừa của2:

Ngoài ra, bạn đọc tham khảo [2,5] cho kết sau:

Bài tập Cho số nguyên dươngn2 Chứng minh tồn số nguyên tốpthỏa mãn

n < p < 3.nC1/ :

Bài tập Cho số nguyên dươngn3 Chứng minh tồn số nguyên tốpthỏa mãn

n < p < 4.nC2/ :

Bài tập 10 Với mỗi" > 0, tồn sốn0sao cho với mọin > n0thì có nhất

2 "/

n

(131)

Tài liệu tham khảo

[1] M Aiger and Găunter M Ziegler, Proofs from THE BOOK, Chapter 2, Fourth Edition, Springer 2010

[2] M El Bachraoui, Primes in the IntervalŒ2n; 3n, Int J Contemp Math Sci., Vol 1, 2006, no 13, 617-621

[3] P Erdos, Beweis eines Satzes von Tschebyschef,˝ Acta Sci Math. (1932), 194-198 [4] L.E Greenfield and S.J Greenfield, Some Problems of Combinatorial Number Theory

Related to Bertrand’s Postulate,J Integer Sequences, Vol 1, 1998.

(132)(133)

Kiều Đình Minh

(THPT Chuyên Hùng Vương, Phú Thọ)

1 Mở đầu

Chuỗi điều hoà

1 X

kD1

1

k D1C C

1 C: : :

là chuỗi vô hạn tiếng Tổng riêng thứn,

HnD1C

1 C

1

3C:::C n

được gọi số điều hoà Dãy.Hn/được gọi dãy số điều hoà (hay dãy điều hồ) Chuỗi có dạng

1 X

kD1

1

km D1C

1 2m C

1

3m C: : :

với m N cịn gọi chuỗi điều hồ bậc m Chuỗi điều hồ tổng qt chuỗi có dạng

1 X

kD1

1

akCb, viaÔ0; b l cỏc số thực Ngoài ra

1 X

kD1

1/kC1 k D1

1 C

1

1 C: : :

gọi chuỗi điều hồ đan dấu.

Trong cơng trình nghiên cứu nhiều nhà tốn học đơi có liên quan đến chuỗi điều hoà kỳ thi học sinh giỏi hay bắt gặp tốn khó Có nhiều hướng tìm hiểu khác chuỗi chẳng hạn toán đánh giá, toán về giới hạn hay toán số học Bài viết chúng tơi trình bày số kết bản về chuỗi điều hoà toán liên quan mà thường gặp kỳ thi Olympic.

2 Các toán đại số

Ví dụ (Đồng thức Catalan)Chứng minh với mọin 2N, ta có:

1 C

1

1 C: : :

1 2n D

1 nC1C

1

(134)

Chứng minh. Biến đổi vế trái ta có

V T D

1C1

2 C

3 C:::C 2n 2 C

4 C:::C 2n

D

1C1

2 C

3 C:::C 2n

1C1

2 C

3 C:::C n

DVP:

Đây đồng thức đơn giản, nhiên lại có nhiều ứng dụng giải tốn Chúng ta bắt gặp điều phần sau

Ví dụ Vớin2N, cho HnD1C

1 C

1

3 C:::C

nI TnDH1CH2C:::CHnI UnD T1

2 C T2

3 C:::C Tn

nC1

Chứng minh rằng

TnD.nC1/HnC1 nC1/UnD.nC2/HnC1 2nC2/. Chứng minh. Ta có:

TnD1C

1C

2

C

1C

2C

C:::C

1C1

2 C

3 C:::C n D n C n C n

3 C:::C n D

nC1 1

C

nC1

C:::C

nC1 n

D.nC1/

1

1C

2 C:::C n

n

D.nC1/Hn nD.nC1/HnC1 nC1/

UnD

T1

2 C T2

3 C:::C Tn

nC1 D.H2 1/C.H3 1/C:::C.HnC1 1/

DH2CH3C:::CHnC1 nD H1CTnCHnC1 n D.nC2/HnC1 2nC2/

:

Vậy toán chứng minh

Ví dụ (Canada MO 1998, Albania BMO TST 2014)Chon2N; n2 Chứng minh rằng

nC1

1C

3C:::C 2n > n 1 C

4C:::C 2n

:

Chứng minh. Ta giải toán theo hai cách sau: Lời giải 1.Bất đẳng thức cần chứng minh tương đương với

1 nC1

2n X

iD1

1 i >

2nC1 n.nC1/

n X

iD1

1 2i ,

2n X

iD1

1 i > 2n n n X

iD1

1 2i C n n X

iD1

1 2i

,

nC1 C

nC2 C: : :C 2n > n C

4 C: : :C 2n

(135)

Ta có

V T / > n 2n D > n: n

2 > VP /:

Vậy toán chứng minh

Lời giải 2.Bất đẳng thức cần chứng minh tương đương với

n

1C

3 C: : :C 2n

> nC1/

1

2 C

4C: : :C 2n

: /

Ta chứng minh./bằng quy nạp Thật vớinD2thì trở thành

3 >

4 (đúng)

Giả sử./đúng vớinDk 2, tức

k

1C

3 C: : :C 2k

> kC1/

1

2C

4 C: : :C 2k : Ta có

1C

3 C: : :C 2k

C kC1

2kC1 D

1 C

1

3C: : :C 2k

C

2C

kC1 2kC1 >

1 2C

1

4 C: : :C 2k

C

2 C

kC1 2kC1 >

1

2C

4 C: : :C 2k

C kC1

2kC2 C 2kC1 >

1

2C

4 C: : :C 2k

C kC2

2kC2:

Do kC1/

1C1

3C: : :C 2k 1C

1 2kC1

Dk

1C1

3C: : :C 2k

C

1C1

3C: : :C 2k

C2kkC1 C1 > k

1C1

3C: : :C 2k C 1 2C

4C: : :C 2k

C2kkC2 C2 > kC1/

1

2C

4C: : :C 2k C 1 2C

4C: : :C 2k

C kC2

2kC2

D.kC2/

1 2C

1

4C: : :C 2kC2

: Theo nguyên lý quy nạp thì./đúng với mọin2N; n2

Ví dụ (USA MO 1995)Choa1; a2; a3: : : ;là dãy số thực dương thoả mãn n X

jD1

aj p

nvới mọin1 Chứng minh với mọin1, ta có n

X

jD1

aj2 >

1C

2 C 3: : :C

1 n

:

Chứng minh. Ta bắt đầu chứng minh bất đẳng thức sau:

Nếua1; a2; : : : ; anlà dương vàb1 b2 : : :bn 0và với mọikn; a1Ca2C: : :C

ak b1Cb2C: : :Cbk,

(136)

Sử dụng công thức khai triển Abel, ta viết

a1b1Ca2b2C: : :CanbnDa1.b1 b2/C.a1Ca2/.b2 b3/C.a1Ca2Ca3/.b3 b4/C: : :C.a1Ca2C: : :Can/bn

b1.b1 b2/C.b1Cb2/.b2 b3/C.b1Cb2Cb3/.b3 b4/C: : :C.b1Cb2C: : :Cbn/bn

Db12Cb 2Cb

2

3C: : :Cb n

Hay ta có (*)

Áp dụng bất đẳng thức Cauchy – Schwarz, ta có

a21Ca22C: : :Ca2n

b21Cb22C: : :Cbn2

.a1b1Ca2b2C: : :Canbn/2 b12Cb

2C: : :Cb n

2

:

Trở lại toán, với ýpn pn >

2pn ChọnbnD

p

n pn 1thì n X

jD1

aj p

nD n

X

jD1

bj Từ./suy

n X

jD1

a2j n X

jD1 p

j pj

2

>

n X

jD1

1 2pj

2 D

4

1C

2C 3: : :C

1 n

:

Vậy tốn chứng minh

Ví dụ Chứng minh đẳng thức

1C

2 C: : :C n D n 1 n C1 n

C: : :C 1/nC11 n n n :

Chứng minh. Dựa vào đẳng thức

.1 x/nD n n xC n

x2C: : :C 1/n

n n

xn

1 x/nD n x n

x2C

n

x3 : : :C 1/n

n n xn : Ta có Z

1 x/n x dxD

1 Z n n xC n

x2 : : :C 1/n

n n

xn

dx D n 1 n C n

C: : :C 1/nC11 n

n n

Mặt khác, đặty D1 xthì Z

0

1 x/n x dxD

1 Z

0

1 yn y dy

D Z

0

1CyCy2C: : :Cyn 1dy

D

yC

2y

2

C: : :C

ny n ˇ ˇ ˇ ˇ

D1C

2 C: : :C n

(137)

3 Các toán giải tích

Euler đưa cơng thức giải tích Hn D Z

0

1 xn

1 x dxcho chuỗi điều hoà Trong mục này

chúng ta tìm hiểu tốn giải tích liên quan đến chuỗi điều hồ tốn liên quan đến số Euler.

Ví dụ Chứng minh rằng

1C

2 C

3C: : :C

n >ln.nC1/; 8n 2N

:

Chứng minh. Có nhiều cách chứng minh bất đẳng thức Sau giới thiệu hai cách chứng minh

Cách chứng minh thứ nhất: Sử dụng bất đẳng thức ln.1Cx/ < x;8x > 0, ta có

1C

2C

3 C: : :C

n >ln.1C1/Cln

1C1

2

C: : :Cln

1C

n

Dln.nC1/:

Cách chứng minh thứ hai:Xét hàm sốy D

x đoạnŒ1InC1 GọiS diện tích hình

thang cong giới hạn đườngx D1; x DnC1; y D0; y D

x Khi

S D nC1 Z

1

dx

x D lnxj

nC1

1 Dln.nC1/ 1/

GọiAi điểm với toạ độ

iI1

i

; i D 2; 3; : : : ; n Kí hiệu A1.1I0/; AnC1.nC1I0/ Gọi

Bi

iI

i

; i D2; 3; : : : ; nC1vàB1.1I1/

GọiS1là diện tích đa giácA1B1B2A2B3A3: : : BnAnBnC1AnC1 Khi

S1 D1C

1 2C

1

3 C: : :C

n 2/

Do hàm sốy D

x nghịch biến trênŒ1InC1nên S1 > S Do từ 1/; 2/suy điều phải

chứng minh

Nhận xét:Từ kết ta suy

lim

1C

2 C: : :C n

D C1: /

(138)

Ví dụ a) Chứng minh tồn giới hạn lim

n!C1.Hn lnn/D b) Chứng minh rằng < HnCHm Hnm 1;8n; m2N

c) Tính gần đúng với sai số chưa đến0; 1.

Chứng minh. a) Đặtan DHn ln.nC1/; bn DHn lnnthì.an/tăng

anan 1,

1

n Clnn ln.nC1/0, n ln

nC1

n ,e

1C

n

n

bất đẳng thức Tương tự thì.bn/giảm Mặt khácan< bnvà

lim

n!C1.an bn/Dn!C1lim

ln n

nC1

D0:

Do tồn lim

n!C1anDn!C1lim bnD b) Ta có

HnCHm Hnm D.HnCHm Hn/ H2n Hn/ H3n H2n/ : : : Hmn H.m 1/n

Hm n:

1 2n n:

1

3n : : : n: mn D1

Lại có

bmn< bn,Hmn ln.mn/Hn lnn,Hn HmnCln.mn/ lnn0: /

bm> ,Hm lnm > : / Cộng./và./ta

Hn HmnCln.mn/ lnnCHm lnm > ,HnCHm Hmn > :

c) lấynD100tính đượcan D0; 57I bn D0; 58 Suy 0; 57

Như thì1C1

2C: : :C

n DClnnC"n; "n!0khin! C1và gọi số Euler

– Mascheroni Chú ý Dlim.Hn lnn/D C1 Z

1

1 Œx

1 x

dx D0; 5772156649 : : : Cho

đến người ta chưa biết số hữu tỷ hay số vơ tỷ Tuy nhiên số có vai trị quan trọng tốn học

Ví dụ (Trường đơng Tốn học 2013)Cho dãy số.an/xác định bởia1 D

3

anC1 Dan

3nC2

2n.nC1/.2nC1/; nD1; 2; : : :

(139)

Chứng minh. Ta có đồng

3kC2

2k.kC1/.2kC1/ D k

1 2.kC1/

1 2kC1:

Do

anD

1 n C

1

nC1C: : :C 2n:

Suy

anDH2n Hn D.Cln2nC"2n/ Cln.n 1/C"n 1/Dln

2n

n 1C."2n "n 1/!ln2;

khin! C1

Chú ý:Có thể sử dụng bất đẳng thức

nC1 < ln

1C

n

<

n;8n 1và định lý giới hạn

kẹp để suy lim

n!C1an Dln2

Ví dụ Giả sửxn2.0I1/là nghiệm phương trình

1 x C

1

x C: : :C

x n D0:

Chứng minh dãy.xn/hội tụ Tìm giới hạn đó.

Chứng minh. Đặtfn.x/D

1 x C

1

x C: : :C

x n, thìfn.xn/D0

Ta thấy0 < xn< 1nên

fnC1.xn/Dfn.xn/C

1

xn n D

1 xn n

< 0:

Trong đófnC1 0C

> Theo tính chất hàm liên tục, khoảng.0Ixn/có nghiệm củafnC1.x/, nghiệm làxnC1 Suy raxnC1 < xn, tức dãy.xn/giảm Do dãy bị chặn bởi0nên hội tụ Ta chứng minh giới hạn bằng0

Giả sử lim xnDa > Khi đó, dãy.xn/giảm nên ta cóxn a;8n: Do

1C

2 C 3: : :C

1

n ! C1

khin! C1nên tồn tạiN cho với mọinN ta có

1C

2 C 3: : :C

1 n >

1 a:

Khi vớinN

0D

xn C

1 xn C

: : :C

xn n

< xn C

1 C

1

2C: : :C

n < a

1 a D0:

(140)

Ví dụ 10 Tồn hay khơng hai đa thứcf x/; g.x/2RŒxthoả mãn f x/

g.x/ D1C 2C

1

3 C: : :C

n; n2N

:

Chứng minh. Giá sử tồn hai đa thức f x/; g.x/ RŒx thoả mãn u cầu tốn Vì

lim n!C1

1C1

2 C

3 C: : :C n

D C1nên lim x!C1

f x/

g.x/ D C1, suy degf > degg Gọi a0; b0 hệ số dẫn đầu củaf x/; g.x/thì

lim x!C1 f x/ xg.x/ D " C1 a0 b0 1/

Mặt khác vớiN > 0tuỳ ý, ta có

0 lim n!C1

1 n

1C

2C: : :C N C

1

N C1 C: : :C n D lim n!C1 n

1C

2 C: : :C N C

N C1 C: : :C n lim n!C1 n

1C

2C: : :C N C lim n!C1 n 1

N C1 C: : :C n

0C lim n!C1

n N n.N C1/

1 N Do lim n!C1 n

1C

2C: : :C n

D0 2/

Từ.1/và.2/suy mâu thuẫn Vậy không tồn đa thứcf x/; g.x/2 RŒxthoả mãn yêu cầu

bài tốn

4 Các tốn số học

Ví dụ 11 (Brazil MO 1983)Chứng minh rằng

HnD1C

1 C

1

3C: : :C n

không số nguyên với mọin2 N; n > 1.

Chứng minh. Kí hiệuklà số tự nhiên thoả mãn2k n < 2kC1 vàM tích tất số lẻ khơng vượt qn Khi

Hn D1C

1 C

1

3 C: : :C

2kC1 C: : :C

1 n

Suy

2k 1:M:HnD2k 1M C2k 2M C2k 1:

M

3 C: : :C M

22 C: : :C

2k 1M

n …Z:

VậyHn D1C

1 C

1

3 C: : :C

(141)

Ví dụ 12 (IMO 1979)Chop; q 2Nsao cho p

q D1 C

1

1 C: : :

1 1318C

1 1319:

Chứng minh rằngpchia hết cho1979.

Chứng minh. Áp dụng đồng thức Catalan ta có

p q D

1 660 C

1

661C: : :C 1318C 1319 D 1 660 C

661 C: : :C 1319 C

1

6601C: : :C 1319 D 1 660C 1319 C 1 661C 1318

C: : :C 1 1319 C 660 D 1979 660:1319C 1979

661:1318C: : :C

1979 1319:660

D1979:A B

Ở đâyB tích số ngun khơng vượt q1319 Do1979là số ngun tố, vậy1979jp

Ví dụ 13 Chop > 3là số nguyên tố,mvà số nguyên nguyên tố cho

m n D

1 12 C

1

22 C: : :C

1 p 1/2:

Chứng minh rằngmchia hết chop

Chứng minh. Chú ý

p 1/Š/2:m

n D p 1/Š/

2

12 C

1

22 C: : :C

1 p 1/2

là số nguyên Cũng ý 1

1; 2; : : : ;

1 p

là hệ thặng dư đầy đủ modulop Theo định lý Wilson, ta có

p 1/Š/2

1

12 C

1

22 C: : :C

1 p 1/2

1/2

12C22C: : :C.p 1/2

p 1/p.2p 3/

6 0.modp/

Dop 5và gcd.6; p/D1 Vì vậypchia hết p 1/Š/

m n

Do gcd p 1/Š; p/D1, ta phải cópjm

Ví dụ 14 (Định lý Wolstenholme)Chop > 3là số nguyên tố Chứng minh rằng

p2j.p 1/Š

1C

2 C: : :C p

(142)

Chứng minh. Đặt

S D.p 1/Š

1C

2C: : :C p

thì

2S D.p 1/Š

p X

iD1 1

i C p i

D.p 1/Š

p X

iD1

p

i p i / Dp:T

ở đâyT D.p 1/Š

p X

iD1

1 i p i /

Do2S số nguyên vàplà số nguyên tố với mẫu số số hạng

T, thânT số nguyên Dop > 3;gcd.p; 2/D1vàpphải chia hếtS Ta cần rap

cũng chia hếtT Theo tốn trên, ta có

T p 1/Š

p X

iD1

1

i2

.p 1/Šm

n 0.modp/

dopjmvà gcd.m; n/D1

Ví dụ 15 (Định lý Euclide)Chứng minh có vơ hạn số ngun tố

Chứng minh. Định lý có khoảng gần20chứng minh chứng minh sau Euler Giả sử có hữu hạn số nguyên tốp1 < p2< : : : < pm Đặt

N D m Y

iD1

1C

pi C

: : :C

pki C: : :

! D

m Y

iD1

1 1

pi

:

Mặt khác, khai triển sử dụng phân tích chuẩn tắc số nguyên dương, ta

N D1C

2 C C: : :

Suy

m Y

iD1

1 1

pi

D1C

2 C

3C: : :! C1 )

m Y

iD1

pi

pi ! C1

:

Mâu thuẫn chứng tỏ điều giả sử sai Vậy tập số nguyên tố vơ hạn Nhận xét:Ta có

m Y

iD1

1 1

pi

D1C

2 C

3C: : :! C1 )

1

m Q iD1

1 pi

! C1 ) m Y

iD1 1 pi !0; suy C1 X

iD1

1

pi D C1

Paul Erdós đưa hai chứng minh đẹp cho phân kỳ chuỗi C1 X

iD1

1 pi

(143)

Ngày có nhiều hướng nghiên cứu liên quan đến chuỗi điều hoà, chẳng hạn như: Nghiên cứu các cách chứng minh phân kỳ chuỗi, mở rộng số Euler đồng thức, mở rộng định lý Wolstenholme, toán phân bố số nguyên tố, số P- adic,

Hy vọng bạn đọc tìm thấy ều bổ ích đọc viết tự tìm tịi thêm vấn đề nêu Vì viết hồn thành thời gian ngắn hiểu biết thân tác giả có hạn nên khơng tránh khỏi thiếu sót Tác giả mong nhận góp ý chân thành từ bạn đọc để viết hoàn chỉnh hơn.

Cuối chúng tơi xin nêu số tốn liên quan đến chuỗi điều hòa để cung cấp thêm tư liệu cho bạn đọc

1 (IMO SL 1989)Chứng minh

1C1

2 C C

6 C: : :C 478 C

1 479

2 480 D2

159 X

kD0

641

.161Ck/.480 k/

2 (Canada MO 1973)Chứng minh

nCH1CH2CH3C: : :CHn DnHn;8n2

3 (Rom Math Magazine, July 1998)Cho

AD

1:2C

3:4C: : :C

2011:2012; B D

1007:2012C

1008:2011C: : :C 2012:1007:

Tính A

B

4 (IMC, Senior Individual Contest 2013)Cho biểu thức

1C

2 C: : :C n C

1C

2C: : :C n C

2 C: : :C n

2

C: : :C

1 n

2

bắt đầu từ ngoặc đơn thứ hai, tổng bên nhận ta bỏ hạng tử tổng ngoặc đơn phía trước Tính giá trị biểu thức khinD2013

5 (APMO 1997)Cho

S D1C

1C

3

C

1C

3C

C: : :C

1C

3C

6 C: : :C 1993006

ở mẫu số chứa tổng riêng dãy nghịch đảo số tam giác Chứng minh rằngS > 1001

6 (T2/198 THTT) Đặt kn D 1C

1 C

1

5 C: : :C

2n Chứng minh với số

nguyên dươngn, ta có

1 k2 C 3k2 C 5k2

C: : :C

.2n 1/k2 n

(144)

7 Choa1; a2; : : : ; an số không âm cho a1a2: : : ak

1

.2k/Š;8k Chứng minh

rằng

a1Ca2C: : :Cak

1 nC1 C

1

nC2 C: : :C 2n

8 (IMO SL 1978)Cho ˆW N !Nlà hàm đơn ánh Chứng minh với số tự

nhiênnthì n X

kD1

ˆ.k/ k2

n X

kD1

1 k

9 (Germany MO 1980)Chứng minh với số tuỳ ý n; k Nlớn hơn1, ta có bất

đẳng thức nk X

jD2

1 j > k

n X

jD2

1 j

10 Chứng minh đẳng thức:

a) n 1:2 n 2:3 C n

3:4 C: : :C 1/

nC1

n n

n.nC1/ D C

1

3 C: : :C nC1

b) n 12 n

22 C

n

32 C: : :C 1/ n

n n

.nC1/2 D nC1

1C

2 C

3 C: : :C n

:

11 Tính giới hạn lim n!C1

n r

1C

2C: : :C n:

12 Cho dãy số.un/xác định bởiu1D1; unC1 DunC

1/n

nC1; n1

a) Chứng minh rằngu2n DH2n Hn;8n1

b) Chứng minh dãy.un/có giới hạn hữu hạn tìm giới hạn

13 Tìm giới hạn dãyanD

1 12 C

1

12C22 C: : :C

1

12C22C: : :Cn2; n1 14 Cho dãy số.an/có giới hạn lim

n!C1anDa Chứng minh

lim n!C1

a1

nC1C a2

nC2 C: : :C an

2n

Daln2:

15 (Olympic 30/4/2015)Cho dãy sốunD

e nC1 nC1 C

e nC2

nC2 C: : :C e

1 2n

2n ;8n1 Tìm

lim n!C1un:

16 Dãy số.xn/thoả mãn điều kiệnjxn xmj>

1

n;8n; m2N

(145)

17 (PTNK TST 2015)Cho dãy.xn/WxnD

1

ncosn; n1 Tìm giới hạn

limx1Cx3C: : :Cx2n

x2Cx4C: : :Cx2n

:

18 Chứng minh dãy.Rn/xác định

RnD1C

1

2 C: : :C n ln

nC

2C 24n

hội tụ tới

19 (VN TST 1999)Cho dãy số thực dương.un/1nD1 Với số nguyên dươngn, giả sửkn

là số nguyên dương nhỏ thoả mãn kn X

iD1

1 i

n X

iD1

ui Chứng minh dãy

knC1

kn

giới hạn hữu hạn dãy.un/có giới hạn hữu hạn

20 (ITOT, Junior A – Level, Fall 2013)Số1 2C

1

1

4C: : :C 2n

1

2nđược biểu

diễn dạng phân số tối giản Giả sử3nC1là số nguyên tố Chứng minh tử số phân số bội của3nC1

21 Chứng minh rằngH m; n/D

m C

mC1 C: : :C

n …Z; 1m < n

22 (IMO SL 1979)Chứng minh khơng có số nguna1; n1sao cho

1C

1CaC

1C2a C: : :C

1Cna 2Z:

23 (ARML 2002)Choalà số nguyên cho1C

2C

3 C: : :C 23 D

a

23Š Tìm phần dư

khiachia cho13

24 (Bulgaria MO 2004)Với mỗin2ZC, tổng1C

2C: : :C

n viết dạng pn

qn với

.pn; qn/D1

a) Chứng minh rằng3khơng chia hếtp67 b) Tìm tất cản2ZC cho3chia hếtpn

25 (AMM E1408)Tìmv2.A/, vớiAlà tử số của1C

1 C

1

5 C: : :C

2k 1; k

26 Cho p số nguyên tố Chứng minh C

2 C : : :C p D

a b p4j.ap b/

27 Chop số nguyên tố lẻ Đặt

a

.p 1/Š D1 C

1

3 : : :C p

1 p 1:

Chứng minh rằnga 2 p

(146)

28 Chứng minh chuỗiX 8…n

1

n hội tụ Ở đây8…ntức lànkhông chứa chữ số8trong

biểu diễn thập phân

29 (VN TST 2005)Một số nguyên dương gọi “ số Kim cương2005” biểu diễn thập phân có2005số9đứng cạnh liên tiếp Dãy.an/; nD1; 2; 3; : : :là dãy tăng ngặt số nguyên dương thoả mãnan< nC C số thực dương đó) Chứng minh dãy số.an/; nD1; 2; 3; : : :chứa vô hạn “số Kim cương2005”

30 (VN TST 1998) Cho số nguyên dương m > Giả sử p1; p2; : : : ; pk tất số nguyên tố không vượt quám Chứng minh

k X

iD1 1

pi C

1 pi2

>ln.lnm/

31 (APMO 2006)Chop 5là số nguyên tố vàr số cách đặtp quân cờ đam bàn cờpp ô cho tất quân cờ không hàng ( cột) Chứng minh rằngr chia hết chop5

32 (Ibero American 2005)Chop > 3là số nguyên tố Chứng minh p X

iD1

1 ip D

n

m với.n; m/D1thìp

3 jn

33 Chop số nguyên tố lẻ ĐặtT p; k/D p X

iD1

1

ik; k2N

Xét số dư củaT p; k/khi chia

chop

34 (Canada MO 1973)Cho m > 1là số nguyên dương Chứng minh

m tổng

của số hạng liên tiếp chuỗi X

jD1

1 j j C1/

35 Chứng minh lim n!C1

nŠ/

nŠ D C1

36 (Brazil MO 1992) Đặt d n/ D X 0<djn

1 Chứng minh với số tự nhiênn > 1, ta có

X

2in

1 i

X d i /

n

X

1in

1 i

37 Cho hàm f W N ! N thoả mãn f 1/ D 1; với k

h

3d 1I3d; d

f k/Dkf d / ĐặtxnD n X

kD1

1

f k/ Chứng minh dãy.xn/không bị chặn

38 (USA MO 2010)Choq D 3p

2 vớip số nguyên tố lẻ, đặt

Sq D

1 2:3:4 C

1

5:6:7C: : :C

1

(147)

Chứng minh

p 2Sq D m

n vớim; n2Z, thìm nchia hết chop

39 Cho dãy.an/tăng ngặt gồm số nguyên dương thoả mãn dãy anC1 an/ bị chặn Chứng minh tập ước nguyên tố dãy.an/là vô hạn

40 a) Chứng minh với số nguyên tốp > 3thì

2p p

1 modp3

b) Chứng minh với số nguyên tốp > 3thì

ap bp

a b

modp3I a; b 2N

c) Chứng minh vớiplà số nguyên tố vànDpl; l 2thì

2n n

2p p

modp4

41 (Argentina MO 2014)Một số nguyênn 3được gọi “Đặc biệt” khơng chia hết n 1/Š

1C1

2 C

3 C: : :C n

Tìm tất số “Đặc biệt” khoảng

Œ10I100

42 Chop > 3là số nguyên tố, p X

kD1

Hk

p3

3 Bp pC1 modp

4

Trong

Bnlà số Bernoulli xác định

B0D1; B1 D

1 2; B2 D

1

6 vàBnD nC1

n X

kD0

nC1 k

Bk; n1

Tài liệu tham khảo

[1] T -W Leung, “Harmonic Series,"Mathematical Excalibur, vol 15, no 16, 2011.

(148)(149)

Yimin Ge, Vienna, Áo

(Người dịch: Nguyễn Tất Thu, Đồng Nai)

1 Mở đầu

Trong thời gian gần đây, số vấn đề lý thuyết số trở nên phổ biến kì thi Cụ thể vấn đề tìm số dư củaax Cbxtheo modulo số nguyên dươngm Trong viết này, chúng tơi đưa tốn tổng qt cho toán

Chúng ta bắt đầu với định lí tồn thặng dư bậc hai Mặc dù tốn ta khơng cần đến nó, ý tưởng chứng minh định lí áp dụng nhiều vấn đề chúng tơi sử dụng suốt viết

Định lý Choplà số nguyên tố lẻ,klà số nguyên dương vàalà số nguyên không chia hết chop Khi đóalà thặng dư bậc hai theo modulopk khi khialà thặng dư bậc hai theo modulop.

Chứng minh. Ta thấy nếualà thặng dư bậc hai theo modulopk hiển nhiênalà thặng dư bậc hai theo modulop Do đó, ta cần chứng minhalà thặng dư bậc hai theo modulopk vớialà thặng dư bậc hai theo modulop Ta chứng minh vấn đề cách quy nạp theok

Giả sửalà thặng dư bậc hai theo modulopk, ta chứng minhalà thặng dư bậc hai theo modulo

pkC1 Thật vậy:

Vìalà thặng dư bậc hai theo modulopk nên tồn số tự nhiênxsao cho

x2 a modpk/

hay tồn số nguyênl cho

x2 DaCl:pk

vớixkhông chia hết chop

Đặtx0DxCy:pk, vớiylà số nguyên Ta chứng minh tồn số nguyêny cho

x02a mod pkC1/:

Ta có

x02 DxCy:pk

2

Dx2C2xypkCy2p2k DaC.lC2xy/pkCy2p2k aC.lC2xy/:pk mod pkC1/:

Ta chứng minh tồn tạiy cho

lC2xy mod p/:

Rõ ràng phương trình đồng dư tuyến tính và.2x; p/D1nên phương trình ln có nghiệm nguny

(150)

Ý tưởng quan trọng chứng minh kĩ thuật hữu ích: Sử dụng giả thiết quy nạp theo modulom, ta xây dựng nghiệm mớix0theo modulom0dựa vào nghiệmx theo modulo

mbằng cách thêm vào biến mớiy Chú ý rằngx0vẫn bất biến theo modulomkhiy thay đổi Nó cịn dùng để chứng minh vấn đề khác chọn giá trịythích hợp

Bài tốn sau xuất kí thi Olympic Brazil năm 2005

Bài toán (Brazil 2005)Cho số nguyên dươnga,b c Chứng minh tồn số nguyên dươngxsao cho

ax Cxb mod c/:

Một trường hợp đặc biệt toán xuất đề IMO Shortlist năm 2006

Bài toán (IMO Shortlist 2006)Chứng minh với số nguyên dươngnluôn tồn số nguyên dươngmsao chonlà ước của2mCm.

Một toán tương tự đưa kì thi USA TST năm 2007

Bài toán (USA TST 2007)Tồn hay không số nguyên dươnga,b sao choakhông là ước củabn nvới số nguyên dươngn.

Tất toán được tổng quát thành toán sau

Định lý ChoA; a; B là số nguyên vàM là số nguyên dương Khi đó, điều kiện cần đủ để với số nguyênC, tồn số nguyên dươngxsao cho

A:axCBx C mod M /

gcd.B; M /D1:

Lưu ý: Ta phát biểu định li theo cách khác

fA:ax CBx mod mjx 2ZCg D f0; 1; : : : ; M 1gkhi gcd.M; B/D1

Chứng minh. Ta chứng minh điều kiện cần: Giả sử.M; B/ > 1, gọi p ước nguyên tố chung củaM vàB Khi đó, nếupjAathì ta chọnC D 1, ngược lại ta chọnC D Khi đó, phương trìnhA:ax CBx C mod p/khơng có nghiệmx

Chứng minh điều kiện đủ: Ta chứng minh theo hai cách sau:

Cách 1.Giả sử.B; M / D GọiC số nguyên đặtM D mn vớim; n2 ZC

sao cho ước nguyên tố củan ước avà gcd.a; m/ D gcd.n; m/ D (điều có nghĩa a D p˛11 :p

˛2 : : : p

˛k

k :m0và M D p ˇ1 :p

ˇ2 : : : p

ˇk

k :mvới pi jm0; m ta chọn

nDp1ˇ1:p2ˇ2: : : pkˇk) Khi đónjax vớixđủ lớn Ta có

A:ax CBx C mod M /, (

A:axCBx C modm/ A:axCBx C modm/ :

Từ.B; n/D1, suy tồn số nguyên dươngB0sao choBB01 mod n/ Khi đó, vớixđủ lớn thỏa mãnx B0C mod n/thỏa mãn phương trình

(151)

Đặtx Dy nCB0C, ta chứng minh tồn số nguyên dươngyđủ lớn thỏa mãn

A:ax CBxC mod m/: 2/

Điều tương đương với

A:anyCB0C CB.nyCB0C /C mod m/

,A:aB0C:any C.Bn/yC.BB0C C /0 mod m/

c:eyCbyCt mod m/ 3/

vớic DA:aB0C; e Dan; b DBn; t DBB0C C Rõ ràng gcd.e; m/Dgcd.b; m/D1:

Đặtf y/DceyCbyCt Bây ta chứng minh toán phương pháp quy nạp theom VớimD1thì.3/hiển nhiên Giả sửm > 1và toán với mọim0 < m Gọiplà ước nguyên tố lớn củamvà giả sửm Dpk:p1k1:p

k2 : : : : p

kr

r :Đặtm0Dp k

:p1k1:p k2 : : : : p

kr r Khi đó, tồn số nguyên dươngyđể

f y/0 mod m0/

hay tồn số nguyênlsao chof y/Dlm0:Ta chọn

y0 DyCz:.p 1/pk

r Y

iD1

.pi 1/piki

vớizlà số nguyên dương Ta chứng minh tồn tạizsao chof y0/0 mod m/:Ta có

f y0

DceyCz.p 1/p

k Qr

iD1

.pi 1/piki

Cb yCz.p 1/pk

r Y

iD1

.pi 1/p ki i

! Ct

Dcey:ez.p 1/p

k Qr

iD1

.pi 1/piki

CbyCt Czb.p 1/pk

r Y

iD1

.pi 1/p ki i

.ceyCby Ct /Czb.p 1/pk

r Y

iD1

.pi 1/piki

lm0Cz:nm0.p 1/

r Y

iD1

.pi 1/ mod m/

(ở ta sử dụngez.p 1/p

k Qr

iD1

.pi 1/piki

1 mod m/theo định lí Euler’s) Do đó, ta chứng minh

lm0Czbm0.p 1/

r Y

iD1

.pi 1/0 modm/

có nghiệm nguyên dươngz, tức phương trình

lCzb.p 1/

r Y

iD1

(152)

có nghiệm nguyên dươngz

Điều ln phương trình đồng dư tuyến tính gcd.b.p 1/

r Y

iD1

.pi 1/; p/D

1 Do bước quy nạp hoàn tất

Ở ta chứng minh tồn số nguyên dươngy chof y/0 mod m/hay tồn

xthỏa mãn (2) Chúng ta cần chứng minh chọny lớn tùy ý thỏa mãn (3) Điều ln thực nếuythỏaf y/0 mod m/thìyC:m:.m/với2ZCcũng thỏa mãn

Vậy định lí chứng minh

Cách 2.Giả sử gcd.B; M /D 1vàC số ngun Ta có dãyAa1; Aa2; : : :là dãy tuần hoàn xét theo moduloM GọiT chu kì dãy, tức làT số nguyên dương nhỏ cho

A:axCkT A:ax mod M /

với mọixđủ lớn

Đặtd Dgcd.T; M / Trước hết ta chứng minhd < M vớiM >

VìdjM nênd M Giả sửd DM, đóMjT nênM T Tuy nhiên,T chu kì dãy

Aai xét theo moduloM nênT M, đóT DM Suy tồn số nguyên dươngX choA:aX mod M /và vậyA:ax mod M /với mọixX Dẫn đếnT D1, nên

M D1

Tiếp theo ta chứng minh toán phương pháp quy nạp theoM

VớiM D1là trường hợp tầm thường Ta xétM > 1và giả sử toán với mọim < M, dẫn đến toán vớid Tức là, tồn số nguyên dươngxsao cho

A:ax CBx C mod d /

hayA:axCBx DC Cld vớillà số nguyên Đặtx0DxCkT vớiklà số nguyên dương Ta có

A:ax0 CBx0DA:axCkT CB.xCkT /

.A:ax CBx/CkBT

DC Cld CkBT mod M /:

Tiếp theo ta chứng minh phương trình đồng dư

ld CkBT mod M /

có nghiệm nguyên dươngk Chia hai vế phương trình đồng dư chod ta phương trình

l CkBT

d mod M

d /

Phương trình ln có nghiệm ngun dươngkvì gcd

BT d;

M d

D1:

(153)

Trần Nam Dũng

(Trường Đại học Khoa học Tự nhiên, ĐHQG - TP.HCM)

LỜI GIỚI THIỆU

Chuyên mục lấy cảm hứng từ viết thầy Nguyễn Duy Liên Epsilon số3về toán số6trong kỳ thi IMO2001với5cách giải khác Mục để dành viết toán hay, lời giải đẹp câu chuyện thú vị xung quan tốn lời giải

Tên chun mục mượn từ tên nhóm người yêu toán Facebook anh Nguyễn Văn Lợi sáng lập “Bài toán hay – Lời giải đẹp – Đam mê toán học” Chuyên mục ghi nhận đề cử bạn đọc chọn đăng kỳ1;

bài tốn

Số chúng tơi giới thiệu với bạn đọc toán số đề thi toán quốc tế năm 1993

Bài toán (IMO 1993, toán 1) Chof x/DxnC5xn 1C3vớin > 1:Chứng minh rằng

f x/khơng thể phân tích thành tích hai đa thức khác với hệ số nguyên.

Bài tốn đặt vị trí số1;tức coi dễ ngày thứ (và kỳ thi) thực tế điểm trung bình thấp nhiều so với số4;số5

và xấp xỉ với bài2và bài6:Gần nửa thí sinh khơng làm bà khoảng20% thí sinh khác được1điểm (chắc ý tưởng phản chứng) Chính tình khó lý giải điểm chuẩn năm1993rất thấp.11cho huy chương đồng,20điểm cho huy chương bạc và30

điểm cho huy chương vàng)

Trong chương trình chun tốn phổ thơng, khơng có nhiều cơng cụ để chứng minh tính bất khả quy đa thức với hệ số nguyên Thực tế có tiêu chuẩn bất khả quy (điều kiện đủ) tiêu chuẩn Eisentein:

Tiêu chuẩn Eisenstein.Cho đa thứcP x/DanxnCan 1xn 1C Ca1xCa02 ZŒx:Giả

sử tồn số nguyên tốpsao cho điều kiện sau đồng thời xảy ra: i) ankhông chia hết chop:

ii) a0; a1; : : : ; an 1chia hết chop: iii) a0không chia hết chop2:

Khi đóP x/bất khả quy.

Ngồi ra, cịn có cách tiếp cận dùng đến nghiệm đa thức, với ví dụ kinh điển sau:

(154)

Lời giải toán IMO1993; thú vị, dựa việc phân tích kỹ cách chứng minh tiêu chuẩn Eisenstein lời giải tốn2nói Vì tiêu chuẩn Eisenstein tốn2đều tiếng, nên nhắc lại chứng minh lời giải mà chuyển sang phần lời giải cho tốn1:

Cách Ta khơng thể áp dụng trực tiếp tiêu chuẩn Eisenstein chop D 3vìan D 5không chia hết cho3(và áp dụng khơng phải đề thi tốn quốc tế) Tuy nhiên, ta dùng tư tưởng chứng minh tiêu chuẩn Eisenstein để giải toán

Cụ thể, lặp lại phép chứng minh tiêu chuẩn Eisenstein, ta chứng minh tiêu chuẩn Eisentein mở rộng sau:

Cho đa thứcP x/ DanxnCan 1xn 1C Ca1xCa0 2ZŒx:Giả sử tồn số nguyên tốp

và số nguyên dươngkthoả mãn đồng thời điều kiện sau: 1) ankhông chia hết chop:

2) a0chia hết chopnhưng không chia hết chop2: 3) a1; a2; : : : ; an k chia hết chop:

Khi đó, nếuP x/DQ.x/ S.x/vớiQ.x/; S.x/là đa thức với hệ số ngun trong hai đa thứcQ.x/; S.x/có bậc nhỏ hơnk:

Ở theo điều kiện đề thìk D2:Do nếuf x/khả quy vàf x/DQ.x/ S.x/thì áp dụng tiêu chuẩn trên, ta suy hai đa thứcQ.x/vàS.x/có bậc1:Từ suy raf x/

có nghiệm hữu tỷ Thế ta biết nghiệm hữu tỷ củaf x/DxnC5xn 1C3đều phải số nguyên (xem tập2ở mục1/;mà x ngun thìxnC5xn 1C3ln số lẻ, khơng thể bằng0:Mâu thuẫn Vậy điều giả sử sai, suy raf x/bất khả quy

Cách Giả sử x1; : : : ; xn nghiệm phức đa thức xnC5xn C3: Khi ta có

P x/DxnC5xn 1C3D.x x1/.x x2/ x xn/:Ta có với mọii D1; 2; : : : ; n

xin 1.xi C5/D 3; suy jxijn 1jxi C5j D3: 1/

Giả sửP x/DQ.x/S.x/vớiQ.x/; S.x/là đa thức với hệ số nguyên khơng tính tổng qt, ta giả sử

Q.x/D.x x1/ x xk/; S.x/D.x xkC1/ x xn/:

Ta cóx1x2 xk vàxkC1 xnlà số nguyên có tích bằng3:Khơng tính tổng qt, giả sử

jx1 xkj D3 jxkC1 xkj D1: 2/: Trong.1/choi chạy từ1đếnkrồi nhân lại, ý đến.2/ta

3n 1j.x1C5/ xkC5/j D3k:

Biểu thức dấuj jbằngjQ 5/jdo số nguyên dương, suy rak>n 1:Như vậyS.x/

(155)

Với cách giải áp dụng cách 2, ta làm mạnh kết sau, tốn xuất kỳ thi VMO 2014 (và gây khó dễ cho nhiều thí sinh)

Bài toán (VMO 2014) Cho đa thứcP x/ D x2 7xC6/2n C13; trong đónlà số nguyên dương Chứng minh rằngP x/khơng thể biểu diễn dạng tích củanC1đa thức khác có hệ số ngun.

Lời giải thức toán dựa vào ý sau:

1) P x/khơng có nghiệm thực, thừa số củaP x/có bậc chẵn

2) P x/có bậc4n;do nếuP x/phân tích thành tích củanC1đa thức khác với hệ số nguyên đa thức phải có bậc2:

Tiếp theo dùng đến việc xér tính chất số học hệ số suy mâu thuẫn

Tuy nhiên, xem xét kỹ cách chứng minh dùng ý tưởng cách giải thứ hai ta dễ dàng chứng minh kết mạnh sau

Bài tốn Cho đa thứcP x/ D.x2 7xC6/2nC13;trong đónlà số nguyên dương. Chứng minh rằngP x/không thể biểu diễn dạng tích của2đa thức khác có hệ số nguyên.

Bài tập xin dành cho bạn đọc Hãy đọc kỹ lại cách2của lời giải ý thêm chi tiết nếuQ.x/là đa thức với hệ số nguyên thìQ.1/ Q.6/chia hết cho5:

(156)(157)

Trần Nam Dũng

(Trường Đại học Khoa học Tự nhiên, ĐHQG - TP.HCM)

LỜI GIỚI THIỆU

Chuyên mục dành cho vấn đề cổ điển đại trình bày dạng tốn xâu chuỗi Đó chuỗi để giải toán đẳng chu, chứng minh đẳng thức Euler kỳ diệu1C

22 C

1

32 C D

2

6 ;một chuỗi toán

vận trù Cách trình bày xuất phát từ vấn đề đơn giản, dễ hiểu, khái niệm định nghĩa ln để đọc tương đối độc lập Và chuỗi nêu vấn đề định, giải toán kinh điển hay nêu giả thuyết mới, vấn đề Lời giải thảo luận toán đăng sốN C3:

Trong số này, tiếp tục đăng phần cuối tóm tắt lời giải toán đăng số2và bắt đầu đăng lời giải tóm tắt tốn đăng số3:

Bất đẳng thức Schapiro (tiếp theo kỳ trước)

Ta nói bất đẳng thức sau vớix1; x2; : : : ; xnlà số thực dương

x1

x2Cx3 C

x2

x3Cx4 C C

xn

xnCx1 C

xn

x1Cx2

n

2: 1/

Khi thi diễn ra, thí sinh giải tốt toán đề nghị mục1;và Ban tổ chức định bổ sung hai toán sau, thực chất chứng minh kết mạnh V.Drinfeld bất đẳng thức Schapiro (kết mạnh cho đánh giá dạng1:9/:

1.10.a) Với số nguyên dươngntồn sốqn> 1;sao cho với số thựcx1; x2; : : : ; xn thuộc đoạn

1

qn

; qn

bất đẳng thức.1/đúng.

b)* Tồn hay không sốq > 1;sao cho với số nguyên dươngnvà với mọixi

1 q; q

bất đẳng thức.1/đúng.

Chứng minh a) Kết thuộc V.Cirtoaje [9] Để cho gọn, ta đặtyk Dxk CxkC1:Khi bất đẳng thức viết lại thành

x1

y2 C

x2

y3 C C

xn

y1

n 2:

Ta biến đổi chút dạng n X

kD1

2q2

nxk ykC1

ykC1

(158)

Trong đóqnlà tham số chọn sau, cho tất bất đẳng thức mà ta xét Vì

2qn2xk ykC1D.qn2xk xkC1/C.qn2xk xkC2/0; nên theo bất đẳng thức Cauchy-Schwarz, ta có

n X

kD1

2qn2xk ykC1

ykC1 n

P kD1

2qn2xk ykC1

n P kD1

.2q2

nxk ykC1/ykC1

:

Như vậy, ta cần chứng minh

A2 D " n

X

kD1

.2qn2xk ykC1/ #2

n.qn2 1/

n X

kD1

.2qn2xk ykC1/ykC1Dn.qn2 1/B:

Vì n X

kD1

yk D2 n X

kD1

xk;ta có đẳng thức

AD.qn2 1/

n X

kD1

yk;

B D2qn2

n X

kD1

xkykC1 n X

kD1

yk2 D2qn2

n X

kD1

ykykC1 q2nC1/ n X

kD1

yk2:

Từ ta cịn cần chứng minh

.qn2 1/

n X

kD1

yk !2

n

"

2qn2

n X

kD1

ykykC1 qn2C1/ n X

kD1

yk2

#

: 2/

Biến đổi vế trái sử dụng đẳng thức

n X

kD1

yk !2

Dn

n X

kD1

yk2 X

i <k

.yi yk/2;

bất đẳng thức.2/có thể viết dạng

n

n X

kD1

.yk ykC1/2 1 q2 n X i <k

.yi yk/2:

Theo bất đẳng thức Cauchy-Schwarz

n X

kD1

.yk ykC1/2 k X

jDi

.yj yjC1/2

1 k j k X

jDi

.yj yjC1/

5

D

k j.yi yk/

2

n 1.yi yk/

2

(159)

Suy

n.n 1/

n X

kD1

.yk ykC1/2

1 n

X

i <k

.yi yk/2;

và ta chọn1 q2

n

D

.n 1/2;tức làqnD

n

p

n2 2n 1 > 1: Lưu ý.Khintăng giá trịqntìm dần đến1:

b) Hiện chưa có câu trả lời

1.11.GọiS Df x1; x2; : : : ; xn/là vế trái bất đẳng thức Schapiro Ký hiệua1; a2; : : : ; an là số x2

x1

; x3 x2

; : : : ; xn xn

; x1 xn

xếp theo thứ tự tăng dần. a) Chứng minh rằng

S

a1.1Can/ C

1

a2.1Can 1/C C

1 an.1Ca1/

:

b) Đặtbk D ˆ ˆ < ˆ ˆ : akanC1 k

nếu akanC1 k

2

akanC1 kCpakanC1 k

nếu akanC1 k <

;chứng minh rằng

2S >b1Cb2C Cbn:

c) Gọiglà hàm lồi lớn không vượt hàme x 2.ex Cex2/ 1:Chứng minh rằng

2S >gŒln.a1an/CgŒln.a2an 1//C CgŒln.ana1/>ng.0/:

d) Chứng minh với mọi > g.0/

2 tồn số nguyên dươngnvà số dươngx1; x2; : : : ; xn

sao choS 6n:

Chứng minh a) Đặtki D

xiC1

xi

;khi

S D

k1.k2C1/

C

k2.k3C1/

C C

kn.k1C1/

a1.anC1/C

1

a2.an 1C1/ C C

1 an.a1C1/

:

b) Bất đẳng thức

1

ai.anC1 i C1/

C

anC1 i.ai C1/

D 1C

aianC1 i 1Cai/.1CanC1 i/

aianC1 i

bi;

trong bất đẳng thức cuối do.1Cai/.1CanC1 i/.1CpaianC1 i/

:

c) Bất đẳng thức

(160)

đúng vìg.x/nhỏ cảe x và2.exCex2/ 1: Cịn bất đẳng thức

gŒln.a1an/CgŒln.a2an 1//C CgŒln.ana1/>ng.0/; theo bất đẳng thức Jensen, vìglà hàm lồi

d) Xem [2]

3.3 Các bất đẳng thức hữu dụng liên quan

Chứng minh bất đẳng thức với điều kiện tất sốxk dương Hãy kiểm tra số in đậm thay số lớn (với mỗin/:

2.1.Bất đẳng thức Mordell.

a) Với số không âmx1; x2; : : : ; xnta có bất đẳng thức n

X

kD1

xk !2

minnn

2;

o

n X

kD1

xk.xkC1CxkC2/:

b) Hãy xác định với giá trị không âm củax1; x2; : : : ; xnbất đẳng thức Mordell trở thành đẳng thức.

Chứng minh Bất đẳng thức chứng minh [20]

a) VớinD3vànD5;sau bỏ dấu ngoặc ta thu bất đẳng thức

.n 1/.a1Ca2C Can/2 2n X

i <k

aiak: 3/

Bất đẳng thức dễ dàng chứng minh dựa vào bất đẳng thức Cauchy-Schwarz cho số

.a1; a2; : : : ; an/và.1; 1; : : : ; 1/

n.a21Ca

2C Ca

n/.a1Ca2C Can/2: Tiếp theo ta ý

n.a1Ca2C Can/2Dn.a21Ca

2 C Ca

n/C2n X

i <k

aiak

.a1Ca2C Can/2C2n X

i <k

aiak;

từ suy ra.3/:VớinD4ta cần kiểm tra bất đẳng thức

.x1Cx2Cx3Cx4/22x1x2C2x2x3C2x3x4C2x4x1C4x1x3C4x2x4: Khai triển rút gọn số hạng đồng dạng, ta thu bất đẳng thức hiển nhiên

(161)

Bây ta chuyển sang trường hợpn>6:Hoán vị vịng quanh số cần, ta giả sử rằngx3 >x1vàx3 >x2ví dụ làm chox3 lớn Vớir D1; 2hoặc3ta ký hiệuar tổng tất sốxk chokr mod 3/; k 6n:Khi đóx1Cx2C CxnDa1Ca2Ca3và theo bất đẳng thức.3/ta suy

.x1Cx2C Cxn/2 D.a1Ca2Ca3/2 3.a1a2Ca2a3Ca3a1/ D3 X

i kÔ0 mod3/

xixk:

tAD X i kÔ0 mod3/

xixk vB D n X

kD1

xk.xkC1CxkC2/và kiểm tra rằngA>B:

Thật vậy, vớin0 mod3/tất số hạng tổngB có tổngA:Vớin0 mod 3/

trong tổngAso với tổngBchỉ thiếu số hạngxnx1:Nhưng số hạng lại không vượt quáxnx3 số hạng có trongAnhưng khơng có trongB;và cuối cùng, vớin2 mod3/thì tổng

Aso với tổngB thiếu số hạngxn 1x1vàxnx2:Nhưng số hạng tương ứng lại nhỏ số hạngxn 1x3 vàxnx3:

Như trường hợpA>B:Do

.x1Cx2C Cxn/2D.a1Ca2Ca3/23A3B D3 n X

kD1

xk.xkC1CxkC2/:

Tính tốt số minnn

2;

o

là hiển nhiên Vớin6ta lấy tất biến bằng1

còn vớin6;ta lấyx1 Dx2Dx3 D1vàx4 Dx5D DxnD0:

b) Trường hợpn < 6hiển nhiên VớinD6dấu xảy khix1Cx4 Dx2Cx5Dx3Cx6: Vớin>6dấu xảy tập hợp có dạng.t; 1; 1; t; 0; 0; : : : ; 0/trong đót 2Œ0; 1

và hốn vị vịng quanh chúng

2.2.Với số không âmx1; x2; : : : ; xnhãy chứng minh bất đẳng thức n

X

kD1

xk !2

min

n 3;

8

n X

kD1

xk.xkC1CxkC2CxkC3/:

Chứng minh Bất đẳng thức chứng minh báo [20] Ta bắt đầu từn68:

VớinD4vànD7đây trường hợp riêng bất đẳng thức.3/:

VớinD5bất đẳng thức tương đương với bất đẳng thức X

.xk 2xkC2CxkC4/2 0:

VớinD6sau bỏ dấu ngoặc rút gọn ta thu bất đẳng thức hiển nhiên

x12Cx22C Cx622x1x4C2x2x5C2x3x6:

Để chứng minh bất đẳng thức trường hợpnD8;ta phá ngoặc hệ bất đẳng thức Cauchy-Schwarz

(162)

để

3.x12Cx 2 Cx

2 3Cx

2

4/2.x1x2Cx1x3Cx1x4Cx2x3Cx2x4Cx3x4/: Suy

3.x1Cx2Cx3Cx4/2 8.x1x2Cx1x3Cx1x4Cx2x3Cx2x4Cx3x4/; 4/ bất đẳng thức cần chứng minh vớin D8:Để chứng minh trường hợpn > 8;ta giả sử x4 lớn đặtar; r D0; 1; 2; 3là tổng cácxk vớikr mod 4/; k 6n:Khi

x1Cx2C Cxn Da1Ca2Ca3Ca4 theo.3/ta có

3.x1Cx2C x4/2D.a1Ca2Ca3Ca4/2 8.a1a2Ca1a3Ca1a4Ca2a3Ca2a4Ca3a4/ X

i kÔ0 mod4/

xixk:

Bõy gi tA D X i kÔ0 mod4/

xixk vB D n X

kD1

xk.xkC1CxkC2CxkC3/;bằng lý luận hoàn

toàn tương tự bài2:1ta suy raA>B từ suy điều phải chứng minh

2.3.a) Vớin68hãy chứng minh bất đẳng thức

x1

x2Cx3Cx4

C x2

x3Cx4Cx5

C C xn

xnCx1Cx2

C xn

x1Cx2Cx3 n

3:

b)* Bất đẳng thức với giá trị khác n hay không ?

Chứng minh a) Bất đẳng thức chứng minh báo [11], sử dụng phép biến đổi Fourier Ta đưa cách chứng minh sơ cấp Theo bất đẳng thức Cauchy-Schwarz ta có

n X

kD1

xk

xkC1CxkC2CxkC3

n P kD1

xk

n P kD1

xk.xkC1CxkC2CxkC3/ n

3;

trong bất đẳng thức cuối suy từ2:2:

b) Chưa rõ câu trả lời

2.4.Chứng minh với mọin>4;thì

.x1Cx2C Cxn/2 >4.x1x2Cx2x3C Cxn 1xnCxnx1/:

Chứng minh Bài toán B.Ginzburg [1, toán 187], đề xuất cho đề thi tồn Liên Xơ năm1972:Bằng cách hốn vị vịng quanh số, giả sửx1 6x2:Đặt

S Dx1Cx2C Cxn; S1 Dx1Cx3C ; S2 Dx2Cx4C

Khi đóS12CS 2

.S1CS2/2

2 D

S2

2 ;suy S2

2 S

2

S12 S 2 D2

X

i kÔ0 mod2/

(163)

Nếunchẵn tổng cuối chứa tất số hạng dạngxkxkC1;cịn nếunlẻ tổng thiếu số hạngxnx1nhưng lại có số hạngxnx2:Như

S2

2 2.x1x2Cx2x3C Cxnx1/:

Ta có điều phải chứng minh

2.5.Chứng minh rằng

n X

kD1

xk

xkC1CxkC2 n X

kD1

xkC1

xkCxkC1

:

Chứng minh Xem lời giải toán1:3:

2.6.Với mọin4;thì

x1

xnCx2 C

x2

x1Cx3 C C

xn

xn 2Cxn C

xn

xn 1Cx1

2:

Chứng minh Đây toán A.Prokopiev, thi thành phố1981 1982; Œ4;

cơng bố tạp chí Kvant,1982;số6;bài toánM 749:

Ta gọi vế trái bất đẳng thức làLn:VớinD4;ta có

L4 D

x1Cx3

x2Cx4 C

x2Cx4

x1Cx3 D

aC

a 2:

Vớin >4 ta lý luận quy nạp Vì bất đẳng thức hốn vị vịng quanh nên ta giả sử

xnC1nhỏ tất số Khi bỏ số hạng cuối củaLnC1;sau giảm hai số hạng khác, ta có

LnC1

x1

xnC1Cx2 C C

xn

xn 1CxnC1

x1

xnCx2 C C

xn

xn 1Cx1 D

Ln:

Để chứng minh số vế phải tốt nhất, ta chọnx1 D x2 D 1; x3 D t; x4 D t2; : : : ;

xnDtn 2rồi chot !0:

Sử dụng bất đẳng thức Cauchy-Schwarz lời giải tốn đây, bạn đọc tìm lời giải khác cách đưa bất đẳng thức2:4:

2.7.Chứng minh với mọin4;thì

x1Cx2

x1Cx3

C x2Cx3

x2Cx4

C Cxn 1Cxn

xn 1Cx1

CxnCx1

xnCx2 4:

Chứng minh Chúng lấy đề từ báo [10] ĐặtS Dx1Cx2C Cxn:Áp dụng bất đẳng thức Cauchy-Schwarz cho số

x

k CxkC1

xk CxkC2

vàf.xk CxkC1/.xk CxkC2/gta

x1Cx2

x1Cx3 C

x2Cx3

x2Cx4 C C

xn 1Cxn

xn 1Cx1 C

xnCx1

xnCx2

4.x1Cx2C Cxn/2 n

P kD1

.xk CxkC1/.xk CxkC2/

(164)

Như vậy, để chứng minh bất đẳng thức cho, ta cần xác lập bất đẳng thức

S2 n X

kD1

.xk CxkC1/.xk CxkC2/D n X

kD1

xk2C2

n X

kD1

xkxkC1C n X

kD1

xkxkC2;

Bất đẳng thức cách phá ngoặc vế phải, vớin>4thì số hạngxkxkC1và

xkxkC2vớikD1; 2; : : : ; nlà khác

Ta chứng tỏ số thay số lớn Thật vậy, đặt xk D ak với

k D1; 2; : : : ; n 1vàxn Dan 2:Khia! 1thìn 3số hạng đầu dần đến0;còn số hạng cuối dần đến1; 2; 1:

2.8.Chứng minh với mọin>6;thì

x1

xnCx3 C

x2

x1Cx4 C C

xn

xn 2Cx1 C

xn

xn 1Cx2

3:

Chứng minh Chúng tơi lấy đề tốn từ báo [6] Bài chứng minh cách áp dụng bất đẳng thức Cauchy -Schwarz tương tự bài2:7;sau áp dụng bất đẳng thức Mordell (bài tốn2:1/:

Ta chứng tỏ số3khơng thể thay số lớn Thật vậy, đặtxk D ak với

kD1; 2; : : : ; n 2vàxn 1DxnD1:Khia!0thì số hạng đầu số hạng cuối dần đến1; số hạng khác dần đến0:

2.9.Chứng minh nếun6;thì

x2Cx3

x1Cx4

C x3Cx4

x2Cx5

C C xnCx1

xn 1Cx2

C x1Cx2

xnCx3 6:

Chứng minh Chúng lấy đề toán từ báo [5] Ta cần cộng hai bất đẳng thức

2:8lại (bất đẳng thức ở2:8và bất đẳng thức cho số đảo ngược)

Ta chứng tỏ số6không thể thay số lớn Thật vậy, đặtxk D ak với

kD1; 2; : : : ; n 2vàxn Dxn D1:Khia!0thì bốn số hạng cuối bằng1; 2; 2; tương ứng, số hạng khác dần đến0:

2.10.Chứng minh rằng

x1Cx2

x1Cx4 C

x2Cx3

x2Cx5 C C

x2004Cx1

x2004Cx3

6:

Chứng minh Trong báo [19], khẳng định nêu dạng giả thuyết Chứng minh thuộc thí sinh thi P.Milosevic M.Bukic

Bất đẳng thức cần chứng minh tổng hai bất đẳng thức vớinD2004- bất đẳng thức từ

2:8và bất đẳng thức

x1

x1Cx4 C

x2

x2Cx5 C C

xn

xnCx3

3:

Ta chứng minh bất đẳng thức VớinD3mnó tổng ba bất đẳng thức

x1

x1Cx4

C x4

x4Cx7

C C xn

(165)

x2

x2Cx5

C x5

x5Cx8

C C xn

xn 1Cx2 1; x3

x3Cx6 C

x6

x6Cx9 C C

xn

xnCx3

1:

Mỗi bất đẳng thức có dạng

1 1Ca1 C

1

1Ca2 C C

1 1Cam

1;

trong vớia1a2 am D1:

Bất đẳng thức chứng minh quy nạp theom:Cơ sở quy nạpmD2;thì

1 1Ca1 C

1 1Ca2 D

1 1Ca1 C

1

1C a11 D1:

Để thực bước quy nạp, ta ý

1 1Cb C

1 1Cc

1 1Cbc:

Ta chứng tỏ số thay số lớn Lấy số

x1 Dx2 Dx3 D1; xk Dan kC1vớik D4; 5; : : : ; nthì khia!0;số hạng thứ thứ hai dần đến2;số hạng thứ ba số hạng cuối dần đến1;các số hạng lại dần đến0:

2.11.Chứng minh rằng

x1

xnCx4 C

x2

x1Cx5 C C

xn

xn 2Cx2 C

xn

xn 1Cx3

4;

trong đónlà số chẵn lớn hơn7:

Chứng minh Đây chứng minh A.Khrabrov Đặt S D x1 C x2 C Cxn T D X

i kÔ0 mod2/

xixk:Theo bt ng thc Cauchy-Schwarz cho

x

k

xk 1CxkC3

vàfxk.xk CxkC3/g

ta thấy vế trái bất đẳng thức không nhỏ

.x1Cx2C Cxn/2

.x1x2Cx2x3C Cxnx1/C.x1x4Cx2x5C Cxnx3/

:

Như cần chứng minh

S24.x1x2Cx2x3C Cxnx1/C4.x1x4Cx2x5C Cxnx3/: Trong lời giải bài2:4ta chứng minh đượcS2 >4T:Như vậy, cần chứng minh

T x1x2Cx2x3C Cxnx1/C.x1x4Cx2x5C Cxnx3/: 6/ Vìnchẵn nên số hạng tổng bên phải chứa tổng bên trái

Ta chứng tỏ số thay số lớn Ta xét số xk D ak với

(166)

2.12.Chứng minh rằng

n X

kD1

xk2

xk2C1 xkC1xkC2Cxk2C2 n

C1

:

Chứng minh Chúng lấy từ báo [14]

Chú ý rằnga2 abCb2 6maxfa; bg2:

Giả sửxi1 số lớn số x1; x2; : : : ; xn; xi2 số lớn hai số tiếp sau

xi1; xi3 số lớn hai số tiếp xi2; : : : Ta xây dựng dãy số gặp số k cho số lớn hai số tiếp sauxik xi1:Rõ ràng k

n

2 k

nC1

:

n X

kD1

xk2

x2kC1 xkC1xkC2Cxk2C2

k X

jD1

xi2

j

xi2

jC1 k:

Ở đánh giá cuối ta dùng bất đẳng thức AM-GM

Hằng số

nC1

không thể thay số lớn ta thayxk D1vớiklẻ vàxk D0

vớikchẵn vế trái

nC1

:

Phát triển tư tổ hợp qua toán đếm

Đếm số đoạn thẳng, số tam giác, số hình vng khơng phải vui mà thực hình thành tư tổ hợp quan trọng: Biết chia trường hợp, biết phân loại, biết dự đoán quy luật, biết tổng quát hóa, biết loại trừ Dưới chuỗi toán đếm dành cho học sinh lớp lớn tiểu học lớp Trung học sở Với tư tổ hợp hình thành mức độ trực giác, trang bị công cụ tổ hợp, học sinh dễ dàng hiểu nhanh, hiểu sâu khái niệm có khả áp dụng

1.5điểm nằm đường thẳng Hỏi có điểm tạo thành?

Đáp số.(10điểm) Ta đếm theo kiểu liệt kê, dùng tổng hợp chập2của5phần tử

(167)

Đáp số.(10tam giác) Một tam giác gồm1đỉnh và1cạnh đáy Liên hệ với tốn1:

3.4điểm trên1đường thẳng và5điểm đường thẳng khác Hỏi có tam giác có đỉnh điểm đó?

Đáp số.(70tam giác) Có 2loại tam giác: Loại1là đỉnh trên, đáy loại2là đáy dưới, đỉnh Với loại1ta có4cách chọn đỉnh và10cách chọn đáy (liên hệ bài2/:Với loại

2ta có5cách chọn đỉnh và6cách chọn đáy Vậy có410C56D70tam giác Trong toán ta làm quen với2quy tắc đếm quan trọng quy tắc cộng (phân trường hợp) quy tắc nhân (phân công đoạn)

(168)

Đáp số..114tam giác) Có4loại tam giác: Loại1; 2; 3là có cạnh nằm trên1trong3cạnh tam giác đỉnh nằm trên1trong2cạnh cịn lại Loại4là loại có3đỉnh nằm trên3cạnh Ta tính thử loại1là loại có cạnh nằm cạnh bên bên trái Có6cách chọn cạnh và6cách chọn đỉnh cịn lại, suy có66D36tam giác loại1:Lý luận tương tự, với tam giác loại2có cạnh nằm cạnh bên bên phải, có3cách chọn cạnh và7cách chọn đỉnh loại3cũng Cuối cùng, có433D36cách chọn tam giác loại4:Vậy đáp số

66C37C37C433D114:

Ta thể đếm cách lấyC103 (là số bộ3đỉnh lấy từ10đỉnh) trừ số tam giác suy biến (là4C1C1D6/ta kết là120 6D114:

5.9điểm tạo thành1lưới vng33:Hỏi có tam giác có đỉnh điểm đã cho?

Đáp số.(76tam giác)

Cách 1.Tương tự bài4;ta chia tam giác làm2loại Loại1là tam giác có2đỉnh nằm

1đường ngang đỉnh nằm hai đường ngang lại Loại2là loại có3đỉnh nằm trên3đường ngang Có3cách chọn đường ngang chứa2đỉnh Với đường ngang chọn, có

3cách chọn2đỉnh Có6cách chọn đỉnh cịn lại Vậy số tam giác loại1bằng336D54:

Với loại2;rõ ràng số cách chọn hàng ngang1điểm bằng333D27:Tuy nhiên, số cách chọn có cách chọn tạo tam giác suy biến Dễ thấy có5tam giác suy biến Vậy số tam giác loại2là27 5D22:Và tổng số tam giác cần tìm

54D22D76:

Cách 2.Đơn giản làC93 8D84 8D76(bạn đọc thử giải thích lời giải này)

(169)

Đáp số.(90tam giác) Ký hiệu hai đỉnh đáy làB; C.B bên trái,C bên phải) Ta thấy1tam giác phải nhậnB hoặcC làm đỉnh

Ta đếm số tam giác nhậnB làm đỉnh Khi hai đỉnh cịn lại phải nằm trên1trong4đường thẳng kẻ từC:Có4cách chọn đường thẳng Trên đường thẳng có6đỉnh, có15

cách chọn2đỉnh Suy số tam giác nhậnBlàm đỉnh bằng415D60:

Tương tự, số tam giác cóC làm đỉnh bằng510D50:

Trong số tam giác này, có số tam giác đếm2lần, tam giác nhận cảB

vàC làm đỉnh Có54D20tam giác (đỉnh có lại giao của5đường thẳng kẻ từB

và4đường thẳng kẻ từC /:Suy có60C50 20D90tam giác

7.Trong hình sau có hình vng?

Đáp số.(30hình vng) Có4loại hình vng:11; 22; 33và44:Do đáp số

16C9C4C1D30:

(170)

Đáp số.1; 9; 36hình vng

9.Thử dự đốn xem hình có hình chữ nhật? Kiểm tra dự đoán bạn bằng cách đếm trực tiếp.

Đáp số.(100hình)

10.Có hình bình hành hình sau?

Đáp số.(45hình)

11.Vẽ9điểm mặt phẳng cho có đúng74tam giác có đỉnh điểm này. Cách giải.Hãy vẽ có10tam giác suy biến!

Bộ tổng đôi

K.Kokhas đề nghị cho Hội nghị mùa hè, thi toán thành phố2002:

Giả sử ta có số thực A D fa1; a2; : : : ; ang (trong sốai có số nhau) Bộ tổng đơi số có dạngai Caj vớii < j:Ta ký hiệu làA.2/:Ví dụ vớiAD f1; 1; 2; 7gthìA.2/D f2; 3; 3; 8; 8; 9g:

1.Từ sốa1; a2; a3; a4; a5ta tạo thành10tổng đôi, ký hiệu chúng làx1; x2; : : : ; x10: Chứng minh biết sốx1; x2; : : : ; x10 (nhưng, dĩ nhiên, khơng biết số tổng số nào), ta xác định sốa1; a2; a3; a4; a5:

2.Biết sốAcó tổng đôi làf2; 2; 2; 3; 3; 3; 3; 4; 4; 4; 4; 5; 5; 5; 6g:Hãy tìm tổng bình phương số bộA:

3.a) Nêu ví dụ hai bộ, có4số (trong cho phép có số nhau) có tổng đôi trùng

(171)

4.Chứng minh vớinD2k; k D1; 2; : : :tồn hai n số thực có tổng đơi

5.Chng minh rng vi minÔ2k;b gmns hon ton c xỏc định tổng đơi

6.a) Chỉ ví dụ ba số (có nhiều hơn2số) có tổng đơi b) Chứng minh khơng tồn ba bộ4số có tổng đôi c) Chứng minh không tồn bốn bộ8số có tổng đơi

7.Học trị Sasha nghĩ ra3số thực ghi lên bảng3tổng đôi,3tổng đôi với dấu trừ tất hiệu đôi một, tổng cộng là12số Hỏi học trị giỏi Nadia biết trị tuyệt đối số mà Sasha nghĩ không?

8.Chứng minh từ bộfa1; a2; : : : ; angcác số thực phân biệt, số tổng phân biệt dạng

ai Caj số nằm giữa2n 1và

n.n 1/ :

Tài liệu tham khảo

[1] Васильев Н Б., Егоров А А Задачи Всесоюзных математических олимпиад М.: Наука, 1988

[2] Дринфельд В Г Об одном циклическом неравенстве // Мат заметки 1971 Т № С 113–119

[3] Курляндчик Л Д., Файбусович А История одного неравенства // Квант 1991 № С 14–18

[4] Толпыго А К Тысяча задач Международного математического Турнира городов М.: МЦНМО, 2009

[5] Чимэдцэрэн С Нэгэн орчилт нийлбэр // Математикийн олимпиадын цуврал 1999 Т 22 (На монгольск яз.)

[6] Чимэдцэрэн С., Адъяасурен В., Батболд С Оценка в одной циклической сумме // Монгол улсын их сургууль, Эрдэм шинжилгээний бичиг 2000 Т (168) С 79–84

[7] Bushell P J Shapiro’s Cyclic Sum // Bull London Math Soc 1994: Vol 26: No 6: P

564–574

[8] Bushell P J., McLeod J B.Shapiro’s cyclic inequality for evenn // J Inequal & Appl.,

2002:Vol.7.3/:P.331–348

[9] Cirtoaje V Crux Mathematicorum.2006:Vol.32:No.8:Problem3195:

[10] Daykin D E Inequalities for certain cyclic sums // Proc Edinburgh Math Soc

.2/ 1970=71:Vol.17:P.257–262:

[11] Diananda P H.Extensions of an inequality of H S Shapiro// Amer Math Monthly1959:

(172)

[12] Diananda P H.On a conjecture of L J Mordell regarding an inequality involoving quadratic forms// J London Math Soc.1961:Vol.36:P.185–192:

[13] Diananda P H.Inequalities for a class of cyclic and other sums// J London Math Soc

1962:Vol.37:P.424–431:

[14] [14] Diananda P H.Some cyclic and other inequalities// Proc Cambridge Philos Soc

1962:Vol.58:P.425–427:

[15] Diananda P H.Some cyclic and other inequalities, II // Proc Cambridge Philos Soc.1962:

Vol.58:P.703–705:

[16] Diananda P H.On a cyclic sum// Proc Glasgow Math Assoc.1963:Vol.6:P.11–13:

[17] Elbert A On a cyclic inequality // Period Math Hungarica 1973 Vol No 2–3 P 163–168

[18] Malcolm M A.A note on a conjecture of L J Mordell// Math Comp.1971:Vol.25:P

375–377:

[19] Mitrinovic D S., Pecaric J E., Fink A M Classical and new inequalities in analysis, Kluwer Academic Publishers Group, Dordrecht,1993:(Mathematics and its Applications (East European Series), Vol.61/:

[20] Mordell L J.On the inequality n X

rD1

xr

xrC1CxrC2

n

2 and some others// Abh Math Sem

Univ Hamburg.1958:Vol.22:P.229–240:

[21] Nowosad P Isoperimetric eigenvalue problems in algebras// Comm Pure Appl Math

1968:Vol.21:P.401–465:

[22] Shapiro H S., Northover F H Amer Math Monthly 1956 Vol 63 No P 191–192

[23] Tanahashi K., Tomiyama J Indecomposable positive maps in matrix algebras // Canad Math Bull 1988 Vol 31 No P 308–317

[24] Troesch B A Full solution of Shapiro’s cyclic inequality // Notices Amer Math Soc 1985 Vol 39 No P 318

[25] Vukmirovic J A note on an inequality for the cyclic sums introduced by D E Daykin // Math Balk.1978:Vol.8:P.293–297:

[26] Yamagami S Cyclic inequalities // Proc Amer Math Soc 1993 Vol 118 No P 521–527

[27] Zulauf A Note on a conjecture of L J Mordell // Abh Math Sem Univ Hamburg.1958:

Vol.22:P.240–241:

(173)

THE THIRD MILLENIUM

Ban Biên tập Epsilon

Kể từ số tạp chí thứ 5, Ban biên tập dành chuyên mục để giới thiệu kỳ thi nước giới Số thứ vừa điểm qua số đề thi, lời giải bình luận cho đề chọn đội tuyển số trường chuyên lớn nước, chuẩn bị cho kỳ thi Học sinh giỏi toàn quốc tới Trong số lần này, tiếp tục giới thiệu Formula of Unity - The Third Millennium, kỳ thi khuôn khổ International Townament of the Towns (ITOT), kỳ thi Toán thành phố, tổ chức vào tháng 10 Hà Nội vừa qua Về đề thi thức lời giải ITOT, giới thiệu số báo khác

Đây cuối Epsilon số kỳ này, hẹn gặp lại độc giả Epsilon số 7!

1 Đề thi dành cho Khối lớp R5

Bài 1. Peter, Basil Anatoly góp tiền tiết kiệm để mua bóng giá la Biết rằng, số tiền góp người khơng nhiều nửa tổng số tiền góp hai người cịn lại Hỏi Peter đóng góp tiền?

Bài 2. Pauline viết hai sốAvàB lên bảng Victoria xóa hai sốA; Bvà viết tổngC tíchD

của chúng Sau đó, Pauline lại xố hai sốC vàD;thay tổngE tíchF chúng Biết hai sốE vàF số lẻ Hỏi số nào?

Bài 3. Ta nói học sinhAhọc tốt học sinhBnếu điểm củaAcao điểm củaBtrong phần lớn kiểm tra Sau3bài kiểm tra, thầy giáo nhận xét học sinhAhọc tốt học sinh B, học sinhB học tốt học sinhC học sinhC lại học tốt học sinhA:Liệu điều xảy khơng?

Bài 4. Nếu Leon bị điểm trường, cậu ta dành buổi tối để nói dối mẹ Cịn ngược lại, cậu ta ln nói thật Leon có em gái ln mẹ cho kẹo hơm bé khơng bị điểm Một buổi tối, Leon nói với mẹ: "Hơm bị nhiều điểm em" Hỏi cô em gái Leon có mẹ cho kẹo hay khơng?

Bài 5. Một tờ lịch ma thuật ngày tất ngày chẵn tháng sai ngày vào tất ngày lẻ Hỏi số lớn ngày liên tiếp ghi ngày lịch bao nhiêu? Và tháng?

(174)

2 Đề thi dành cho Khối lớp R6

Bài 1. Có14người ngồi quanh vòng tròn Peter, Victoria, Anatoly Genghis ngồi cạnh theo thứ tự bạn có đồng xu mệnh giá1; 2; 5và10rúp Một người đưa đồng xu cho người bên trái bên phải có đúng3người ngồi họ Sau lúc chuyển vậy, bạn Peter, Victoria, Anatoly Gengis lại nhận đồng xu Hỏi lúc này, bạn cầm đồng xu nào? Hãy tất khả chứng minh khơng cịn trường hợp khác

Bài 2. Pauline viết hai sốAvàB lên bảng Victoria xóa hai sốA; Bvà viết tổngC tíchD

của chúng Sau đó, Pauline lại xố hai sốC vàD;thay tổngE tíchF chúng Biết hai sốE vàF số lẻ Hỏi số nào?

Bài 3. Ta nói học sinhAhọc tốt học sinhBnếu điểm củaAcao điểm củaBtrong phần lớn kiểm tra Sau3bài kiểm tra, thầy giáo nhận xét học sinhAhọc tốt học sinh B, học sinhB học tốt học sinhC học sinhC lại học tốt học sinhA:Liệu điều xảy không?

Bài 4. Nếu Leon bị điểm trường, cậu ta dành buổi tối để nói dối mẹ Cịn ngược lại, cậu ta ln nói thật Leon có em gái ln mẹ cho kẹo hơm bé khơng bị điểm Một buổi tối, Leon nói với mẹ:"Hôm bị nhiều điểm em". Hỏi em gái Leon có mẹ cho kẹo hay không?

Bài 5. Một tờ lịch ma thuật ngày tất ngày chẵn tháng sai ngày vào tất ngày lẻ Hỏi số lớn ngày liên tiếp ghi ngày lịch bao nhiêu? Và tháng?

Bài 6. Có số có biểu diễn thập phân gồm 10 chữ số khác có chứa đoạn0123

hoặc đoạn3210?

Bài 7. Hình vng88được vẽ tờ giấy kẻ ô vuông dọc theo đường kẻ Alex cắt hình vng88theo đường kẻ thành7phần với chu vi Hãy cách cắt Alex

3 Đề thi dành cho Khối lớp R7

Bài 1. Một tờ lịch ma thuật ngày tất ngày chẵn tháng sai ngày vào tất ngày lẻ Hỏi số lớn ngày liên tiếp ghi ngày lịch bao nhiêu? Và ngày ngày nào?

Bài 2. Hãy điền vào ô bảng vuông55các số nguyên dương phân biệt cho tổng số hàng, cột nhỏ Biết rằng, số1; 2; 3; 4; 2015

đã điền trước đường chéo

(175)

Bài 4. Có27con gián tham gia chạy đua Trong vịng có ba gián chạy Mỗi gián chạy với tốc độ cố định, không đổi vòng đua, tốc độ gián đơi khác Sau vịng, người ta ghi lại thứ tự đích gián tham gia vịng đua Hỏi14vịng đua có đủ để xác định xác theo thứ tự hai gián chạy nhanh khơng?

Bài 5. Ta nói học sinhAhọc tốt học sinhBnếu điểm củaAcao điểm củaBtrong phần lớn kiểm tra Sau3bài kiểm tra, thầy giáo nhận xét học sinhAhọc tốt học sinhB;học sinhBhọc tốt học sinhC học sinhC lại học tốt học sinhA:Liệu điều xảy khơng?

Bài 6. Ta nói số ngun dương đẹp tích giai thừa số nguyên tố (không thiết phải phân biệt) Ta gọi số hữu tỉ dương tốt tỉ số hai số nguyên dương đẹp Chứng minh tất số hữu tỉ dương tốt

Bài 7. Ta gọi số nguyên dương đẹp dãy chữ số tăng thực sự, ví dụ1589là số tăng cịn447thì khơng Hãy tìm số nhỏ số nguyên dương đẹp với tổng là2015:

4 Đề thi dành cho Khối lớp R8

Bài 1. Hãy điền vào ô bảng vuông55các số nguyên dương phân biệt cho tổng số hàng, cột nhỏ Biết rằng, số1; 2; 3; 4; 2015

đã điền trước đường chéo

Bài 2. Có27con gián tham gia chạy đua Trong vịng có ba gián chạy Mỗi gián chạy với tốc độ cố định, khơng đổi vịng đua, tốc độ gián đôi khác Sau vòng, người ta ghi lại thứ tự đích gián tham gia vịng đua Hỏi14vịng đua có đủ để xác định xác theo thứ tự hai gián chạy nhanh không?

Bài 3. Hãy tìm số nguyên dương cho tích ước tự nhiên là1090:

Bài 4. John có12que gỗ với độ dài que số nguyên dương không vượt quá56:Chứng minh John có3que tạo thành tam giác

Bài 5. Ta nói số nguyên dương đẹp tích giai thừa số ngun tố (không thiết phải phân biệt) Ta gọi số hữu tỉ dương tốt tỉ số hai số nguyên dương đẹp Chứng minh tất số hữu tỉ dương tốt

Bài 6. Cho tam giácABC với∠B D 30ı;∠C D 105ıvàD trung điểm đoạn thẳngBC:

Tìm góc∠BAD?

(176)

5 Đề thi dành cho Khối lớp R9

Bài 1. Các đỉnh đa giác đều12cạnh tô màu xanh đỏ Biết trong3đỉnh tạo thành tam giác đều, có nhất2đỉnh màu đỏ Chứng minh ta chọn4đỉnh tạo thành hình vng với nhất3đỉnh đỏ

Bài 2. Ta nói số nguyên dương đẹp tích giai thừa số ngun tố (không thiết phải phân biệt) Ta gọi số hữu tỉ dương tốt tỉ số hai số nguyên dương đẹp Chứng minh tất số hữu tỉ dương tốt

Bài 3. Có27con gián tham gia chạy đua Trong vịng có ba gián chạy Mỗi gián chạy với tốc độ cố định, không đổi vòng đua, tốc độ gián đơi khác Sau vịng, người ta ghi lại thứ tự đích gián tham gia vịng đua Hỏi14vịng đua có đủ để xác định xác theo thứ tự hai gián chạy nhanh không?

Bài 4. Cho tam giácABC với∠B D 30ı;∠C D 105ıvàD trung điểm đoạn thẳngBC:

Tìm góc∠BAD?

Bài 5. John có12que gỗ với độ dài que số nguyên dương khơng vượt q56:Chứng minh John có3que tạo thành tam giác

Bài 6. Hãy tìm số nguyên dương cho tích ước tự nhiên là1090:

Bài 7. Tất biết 32 C42 D 52 Bên cạnh đó, khơng phải biết

102C112C122 D132C142:Liệu có tồn hay không2015số nguyên dương liên tiếp cho tổng bình phương của1008số tổng bình phương của1007số sau đó?

6 Đề thi dành cho Khối lớp R10

Bài 1. Hai thỏ Bugs Roger cá cược xem nhanh Để xác định người chiến thắng, hai bạn định tổ chức thi Mỗi bạn thỏ nhảy50mét theo hướng sau quay lại để nhảy ngược lại Biết rằng, độ dài bước nhảy Bugs là50cm Roger là60cm, thỏ Bugs nhảy được6bước Roger nhảy được5bước Hỏi giành chiến thăng?

Bài 2. Với giá trị củanthì ta chia hình vng thànhnhình chữ nhật đồng dạng cho có hai số chúng khơng nhau?

Bài 3. Có tồn hay không số nguyên dươngavàbsao cho

lcm.a; b/Dlcm.aC2015; bC2016/‹

Ở đây,lcm.a; b/được kí hiệu cho bội chung nhỏ hai sốavàb:

Bài 4. Cho tam giácABC với∠B D 30ı;∠C D 105ıvàD trung điểm đoạn thẳngBC:

Tìm góc∠BAD?

Bài 5. Hãy điền vào ô bảng vuông1010các số nguyên dương phân biệt cho tổng số hàng, cột nhỏ Biết rằng, số1; 2; 3; 4; 5; 6; 7; 8;

(177)

Bài 6. Đường tròn nội tiếp tam giácABC tiếp xúc với cạnhAB; BC vàAC điểm

C1; A1vàB1tương ứng Chứng minh

AC AB1 C

CB CA1 C

BA BC1

> 4:

Bài 7. Tất biết 32 C42 D 52 Bên cạnh đó, khơng phải biết

102C112C122 D132C142:Khẳng định sau hay sai: Với số nguyên dươngk;có

2kC1số nguyên dương liên tiếp cho tổng bình phương củakC1số tổng bình phương củaksố cịn lại?

7 Đề thi dành cho Khối lớp R11

Bài 1. Hai thỏ Bugs Roger cá cược xem nhanh Để xác định người chiến thắng, hai bạn định tổ chức thi Mỗi bạn thỏ nhảy50mét theo hướng sau quay lại để nhảy ngược lại Biết rằng, độ dài bước nhảy Bugs là50cm Roger là60cm, thỏ Bugs nhảy được6bước Roger nhảy được5bước Hỏi giành chiến thăng?

Bài 2. Với giá trị củanthì ta chia hình vng thànhnhình chữ nhật đồng dạng cho có hai số chúng khơng nhau?

Bài 3. Có tồn hay khơng số nguyên dươngavàbsao cho

lcm.a; b/Dlcm.aC2015; bC2016/‹

Ở đây,lcm.a; b/được kí hiệu cho bội chung nhỏ hai sốavàb:

Bài 4. Cho tam giácABC với∠B D 30ı;∠C D 105ıvàD trung điểm đoạn thẳngBC:

Tìm góc∠BAD?

Bài 5. Tại điểm có tọa độ nguyên mặt phẳng tọa độ trồng với đường kính10 Một bác tiều phu đốn gốc tọa độ.0; 0/và đứng gốc Hỏi phần mặt phẳng mà nhìn thấy có bị giới hạn hay không? Ở đây, coi cột hình trụ vơ hạn với trục chứa điểm nguyên mặt phẳng tọa độ

Bài 6. Hãy bộ4số dương bán kính bốn hình cầu đơi tiếp xúc

Bài 7. Tất biết 32 C42 D 52 Bên cạnh đó, khơng phải biết

102C112C122 D132C142:Khẳng định sau hay sai: Với số nguyên dươngk;có

Marquis de Condorcet lý thuyết chọn lựa xã hội, “nghịch lý Condorcet” Kenneth Arrow định lý bất khả thi Arrow1 chứng minh tổ hợp ngắn gọn. phát kiến tuyệt vời ứng dụng cụ thể giải thuật Robinson-Schensted. tăng sờ bài Terry Tao đẳng thức Plancherel đẳng thức Paserval bài này chỉ số sức mạnhBanzhaf xấp xỉ Stirling đây. 1http://www.georgeboole.com http://www.maths.tcd.ie/pub/ims/nl14 http://thatsmaths.com Pet-recognition. Cut The Knot blog Tanya Khovanova,

Ngày đăng: 08/02/2021, 07:45

TỪ KHÓA LIÊN QUAN

w